Ευκλειδης Β 53

Page 1


ΕΚΔΟΣΕΙΣ

ΠΑΤΑΚΗ www.patakis.qr

ΕΚΠΑΙΔΕΥΤΙΚΑ ΒΙΒΛΙ

ΠΑΤΑΚΗ

για να ξεκινήσετε με προβάδισμα άλγε:�ρα

{Jfι')ιιΟ<.;

t

ΑΛΓΕΒΡΑ

Α' Ενιαίου Λυκείου Γιάννης Βιδάλης, Βασίλης Γκιμίσης

ΑΛΓΕΒΡΑ

ΓΕΩΜΕΥΡΙΑ

Α' Ενιαίου Λυκείου

Γιάννης Βιδάλης, Βασίλης Γκιμίσης

ΕΥΚΛΕΙΔΕΙΑ ΓΕΩΜΗΡΙΑ

ΦΥΙΙΚΗ

ΦΥΙΙΚΗ

Α' Ενιαίου Λυκείου

Α' Ενιαιου Λυκείου

Νεκτάριος Πρωτοπαπάς

Βαγγέλης Φωτεινόπουλος

ΦΥΙΙΚΗ

ΜΑθΗΜΑΥΙΚΑ

Β' Ενιαίου Λυκείου, Γενικής Παιδείας

Β' Ενιαίου Λυκείου, Γενικής Παιδείας

Β' Ενιαίου Λυκείου, Γενικής Παιδείας

Β' Ενιαίου Λυκείου, θετικής Κατεύθυνσης

Γιάννης Βιδάλης, Βασίλης Γκιμίσης

Μ. Γεωργάκης, Αθ. Μακρίδης κ.ά.

Νεκτάριος Πρωτοπαπάς

Κ. Α. Κυριακόπουλος

φυσική

μαθηματικά

Γ

Γ' Evιoiou Λυκεiου

ΜΑθΗΜΑΥΙΚΑ

Γ Ενιαίου Λυκείου, Γενικής Παιδείας Ματθαίος Τσιλπιρίδης

ΜΑθΗΜΑΥΙΚΑ

Ενιοιου

Λυκειου

!!I τόμοζ r:;ι

ΦΥΙΙΚΗ

ΒΙΟΛΟΓΙΑ

Γ' Ενιαίου Λυκείου,

Γ Ενιαίου Λυκείου,

Γ Ενιαίου Λυκείου, θετικής Κατεύθυνσης

θετικής και Τεχνολογικής Κατεύθυνσης

θετικής και Τεχνολογικής Κατεύθυνσης

Αμαλία Τ ένγκου

Ελευθέριος Πρωτοπαπάς

Τριαντάφυλλος Μελισσαρόπουλος

σε όλο το βιβλιοπωλείο

LETRINA S.A. CERτtFICATION θΟΟΥ

ΒΙΒΛΙΟΠΩΛΕΙΟ ΠΑΤΑΚΗ: ΑΚΑΔΗΜΙΑΣ 65, I 06 78 ΑΘΗΝΑ, ΤΗΛ. 210.38.11.850

KENTPIKH ΔΙΑΘΕΣΗ: ΕΜΜ. ΜΠΕΝΑΚΗ 16,106 78 ΑΘΗΝΑ, ΤΗΛ. 210.38.31.078 ΘΕΣΣΑΛΟΝΙΚΗ: Ν. ΜΟΝΑΠΗΡΙΟΥ 122, ΤΗΛ. 2310.70.63.54-5

l't'Mil'i>o!lς;

ISO

9001

'" 12. Ι.'.�,\ 1.17

Cerιifίcacc Nu. 081054


ΕΛΛΗΝΙΚΗ ΜΑΘΗΜΑΤΙΚΗ ETAIPEIA Τεύχοc;

53 -

ΙούΑιοc; - Αύyουστοc; - Σεπτέμβpιοc; e-mail: info@hms.gr www.hms.gr

2004

Eupώ:

2,50

ΜΑΘΗΜΑη κο ΠΕΡΙΟΔΙΚΟ ΠΑ ΤΟ ΛΥΚΕΙΟ

Υπεύθυνοι Έκδοσης

ΠΕΡΙΕΧΟΜΕΝΑ

Ευσταθίου Ευάγγελος, Μπαραλής Γεώργιος.

Συvτακηκή Ομάδα:

2

Ανδρουλακόκης "Νίκος Βακαλόπουλος Κώστας Βιοκαδουρόκης Βασίλης Βλάχου Αγγελική Γιούσrις Θεόδωρος Γράψα.ς Κωνσταντίνος Δούvαβης Αντώνης Ευθύμογλου Πέτρος θεοδωρέλος Χρίστος Καλίκα.ς Σταμάτης Καρακάοογλου Αλέξανδρος . Καρακατοάνης Βασίλης Καρκάνης Βασίλης Κατοούλης Γιώργος Κερασαρίδης Γιάννης Κηπουρός Χρήστος Κόντζιας Νίκος Κυριακόπουλος Ανιώvrις Κυριακόπουλος θανάοης Κυριακοπούλου-Κυβερvίμου Χρυστ. Λο{,αρίδrις Χρίιστος Λουρίδας Σωτήρης Μαλαφέκας θανάσης ΜαλιδάκrιΕλένη Μεταξάς Νίκος Μυλωνάς Δημψρης Μώκος Χι:ιιίστος Ντζιώp � Ηλίας Παπ�ιάν Μαριλένα Παππάς Γιώργος Ρήγας θεόδωρος Σακελλάριις Βασίλης ΣαiτηΕύα Σια.θόπουλος Γιώργος Στάϊκος Κωνσταντίνος Στάϊκος Παναγιώτης Στρατής Ιωάvvrις Ταπεινός Νικόλαος Τασσόπουλος Γιώργος Τριάντος Γεώργιος Τσικαλουδάκrις Γιώργος Τσιούμα.ς θανάσιις Τυρλής Ιωάννης Χαραλaμπίδης Γιώργος Χαραλaμποπούλου Λίνα Χαραλάμπους θάνος Χριστόπουλος Παναγιώτης

5 7

ΑΡΧΑΙΑ ΕΛΛΗΝΙΚΑ ΜΑΘΗΜΑτΙΚΑ

ΤΟ ΒΗΜΑ ΤΟΥ ΕΥΚΛΕΙΔΗ ΗΟΜΟ MATHEMAfiCUS

I Μα8ημαηκά για την Α' Τάξη του Λυκείου I 11

Για να κατανοήσουμε καλύτερα τους αριΟμούς 20 Ισότητα τριγώνων

I Μα8ημαηκά για την Β' Τάξη ιου Λυκείου I 25 Τριγωνομετρία

29 Αναλογίες- Μετρικές σχέσεις 33 45η ΔιεΟvής ΜαΟηματική Ολυμπιάδα 50 Γεωμετρικά συμπεράσματα σε τρίγωνο όταν δύο yωνίες του

διαφέρουν κατά 90°

53 Διανυσματικές πράξεις- Συντεταγμένες στο επίπεδο - Εσωτερικό

γινόμενο διανυσμάτων

Συvcρyάιcς:

Μα8ημαηκά για rην Γ' Τάξη ιου Λυκείου

Φελλούριις Ανόρyυρος,Ελευθέριος Πρωτόπαπας, Γιάννης Στρατήyης, Δημήτρης Μυλωνάς

ΕΚΔΟΣΗΤQΣ ΕΛΑΗΝΙΚΗΣ . ΜΛΘUΜΛΤΙΚΗΣΕΤΑΙΡΕΙΑΣ.

ΠΑΝΕJΙI.ΣΊΗΜΙΟΥ3:4 ·10679ΑθΗΝΑ

63 Θέματα μιγαδικών αριΟμών

. Εκδότης: Αλε(αvδρqςΝμrόλ(].Ος ΔιeυΟυvιής: Ι{ι:φκ:οuλιαςΤiώρyος .

76 Προβλήματα απ' την καΟημεριvή ιωή

Τηλ; 210-36

17 784; 210'36 .16 532 FΒΣ 210c36 41 025 . · . · .· . • . . . . .

.

-

. .

.

ISSN: 1105- 7998

ΣΥΝΔΡΟΜΕΣ:

2,50 ευρώ Τεύχος Ετήσιαουvδρομίι 10,00 ευρώ (Σχολ) Συνδρομητές 10,00+2,00 ευρώ (Tax.) Οργανισμοί: 18,00 ευρώ Ταχ. Επιιαyός Τ. Γραφείο ΑΟήvα 54, Τ.θ. 30044

70 ΜαΟηματικά Γενικής Παιδείας- Ανάλυση 77 Η στήλη του μαΟητή

Σιοιχcιοθεσία - Σελιδοποίηση

ΕκτύπωοιιΙΝΤΕΡΠΡΕΣ Α.Ε., Iερά οδός 81 - 83

Ν. Αλεξόπο�λος

Ακαδημίας 43, τηλ 210-3606826

Υπευθ. Τυποyραφc(οιr.

Τηλ.:3474654


1

λ.ΡΧλ.Ιλ. θλλΗΝΙΚλ. Μλ.ΘΗΜλ.ΤΙΚλ. ΜΕΛΕΤΗ ΤΗΣ ΑΞΙΟΠΙΣΤΙΑΣ ΤΟΥ ΕΜΠΕΙΡΙΚΟΥ ΗΡΩΝΙΚΟΥ ΤΥΠΟΥ

Χρήστος Κηπουρός Στο 47° τεύχος του Ευκλείδη Β ' , στο άρθρο μας «Ε ισ αγωγή στη ν Έρευνα ιστορικών θ εμ άτ ων}}' είχαμε υποδείξει τον τρόπο έρευνας της αξιοπιστίας του τύπου ( 1 ), που μας δίνει το μήκος του τόξου ......---...

ΑΓΒ,

όπου β και υ είναι, αντίστοιχα, η χορδή ΑΒ και υ το βέλος ΔΓ του τόξου γωνία την φ :ς<1 80°.

......---...

ΑΓΒ,

που έχει επίκεντρη

Είχαμε χωρίσει τη διαδικασία σε 5 βήματα Επειδή δεν λάβαμε καμία εργασία, υποθέτουμε ότι ή δεν είχατε τον απαιτούμενο χρόνο για να ασχοληθείτε, ή δεν καταλάβατε καλά την διαδικασία. Για το λόγο αυτό σας δίνουμε την πορεία της έρευνας, ώστε να ξαναπροσπαθήσετε. Αν δεν έχετε τη δυνατότητα πρόσβασης στο πρόγραμμα excel, δεχθείτε τις τιμές του πίνακα που ακολουθεί, για τις συναρτήσεις που εμπλέκονται, καθώς και τα γραφήματά τους και παρακολουθήσετε προσεκτικά τη διαδικασία. Αν βρείτε δυσκολίες, γράψτε μας και θα σας απαντήσουμε. Η διαδικασία είναι η παρακάτω: Γ

υ

(

Από το σχήμα βρίσκουμε για τα β και υ: Q. = rημ φ και υ = r 1 - συν φ 2 2 2 φ άρα: β = 2rημ και το lo βήμα γίνεται: 2

Το 2ο βή μα γίνεται: S1(φ) = 2r ημ 2

�+

(

1 - συν

)

) � 2

= 2r 2 - 2συν

(2)

� = 2r

�(

2 1 - συν

φ φ φ = 4rημ = 40ημ , ( r = lO ) . 4 4 4 φ φ φ φ Ώστε: S1 ( φ ) = 40ημ . και S 2 ( φ ) = � 1 - συν = �ημ 2 = 5ημ 2 . 4 4 2 2 4 4 = 2r 2.2ημ 2

(

)

ΕΥΚΛΕΙΔΗΣ Β' λη ' τ.V2

)

=

ιι


------- Αρχαία Ελληνικά Μαθηματικά

3ο βήμα:

-------

πr

Το μήκος του τόξου δίνεται από τον τύπο Σ ( φ ) - φ = Ο, 1745φ. 1 80 όπου η γωνία φ μετρείται σε μοίρες. =

4ο βήμα:

Σχηματίζουμε τη συνάρτηση : F(φ) S(φ)-Σ(φ) [Sι(φ)+Sz(φ)]-Σ(φ) =

=

5ο βήμα: ·

Σχηματι' ζουμε τη συναρτηση: γωνίας φ. Και είναι:

=

φ φ 40ημ + 5ημ 2 - 0, 175φ. και, τέλος, 4 4

F( φ ) ' της ' ' το σχετικο' σφ άλμα για τις διαφορες που μας δινει τιμες Σ( φ) F( φ ) Σ( φ)

;

40ημ + 5ημ 2 =

Γενική παρατήρηση : τις συναρτήσεις: S( φ ) , Σ ( φ ) , F( φ ) και

; - 0, 1745

0, 1745

�i:; για τις διάφορες τιμές της γωνίας φ, δεν χρειάζεται να

τις υπολογίσουμε γιατί θα μας τις δώσει το πρόγραμμα excel, (εφόσον το έχουμε). Για τους τριγ. αριθμούς χρησιμοποιούμε, πάντοτε, την λατινική ορολογία.( π.χ. αντί ημθ βάζουμε sinθ, κ.ο.κ.). Περνάμε αυτές τις συναρτήσεις σε ένα πρόγραμμα excel και παίρνουμε ένα πίνακα των τιμών τους, για τις διάφορες τιμές της ανεξάρτητης μεταβλητής φ, από 0<ψ::180. Από τον πίνακα της επόμενης σελίδας παρατηρούμε τα εξής: lη Παρατήρηση

Οι τιμές της συνάρτησης F( φ ), όπως δείχνει ο πίνακας, είναι μικρότερες, από 10,21. στα διαστήματα 0°<φ<55° και 140°<φ0< 160° που σημαίνει ότι ο εμπειρικός τύπος �εν διαφέρει πολύ του πραγματικού τύπου Σ( φ). 2η Παρατήρηση

Η συνάρτηση. F( φ )!Σ( φ) που παριστάνει το σχετικό σφάλμα, είναι μικρότερη από 10,021 στα διαστήματα 0<φ<55° και 100°<φ<1 80, δηλ. το σχ. σφάλμα είναι, στα διαστήματα αυτά, μικρότερο από 2%. Αυτό επιβεβαιώνεται και από τα γραφήματα των συναρτήσεων, που ακολουθούν. Πράγματι:

Το γράφημα της συνάρτησης F(φ) (κόκκινη καμπύλη). πλησιάζει τον οριζόντιο άξονα των τιμών της γωνίας φ, στα διαστήματα της 1 ης παρατήρησης και της συνάρτησης F(φ )!Σ( φ) (άσπρη καμπύλη) συμπί­ πτει, σχεδόν, με τον οριζόντιο άξονα στα διαστήματα της 2ης παρατήρησης. Συμπέρασμα:

Ο τύπος ( 1 ), μπορεί να χρησιμοποιηθεί, ικανοποιητικά, για τόξα με επίκεντρες γωνίες φ, περιεχόμενες στα παραπάνω διαστήματα. Στον παρακάτω πίνακα περιέχονται οι τιμές των συναρτήσεων S1, S2, (S1+S2), Σ(φ), F(φ) και F(φ)!Σ(φ). που πήραμε από το πρόγραμμα excel. ΕΥΚΛΕΙΔΗΣ Β ' λη ' τ.l/3


------- Αρχαία Ελληνικά Μαθηματικά

φ 1 5 10 15 20 25 30 35 40 45 50 55 60 65 70 75 80 85 90 95 100 105 110 115 120 125 130 135 140 145 150 155 160 165 170 175 180

-2 -3 -4

rad 0,017453 0,087266 0,174532 0,261799 0,349065 0,436332 0,523598 0,610865 0,698131 0,785398 0,872664 0,959931 1,047197 1,134464 1,221730 1,308996 1,396263 1,483529 1,570796 1,658062 1,745329 1,832595 1,919862 2,007128 2,094395 2,181661 2,268928 2,356194 2,443460 2,530727 2,617993 2,705260 2,792526 2,879793 2,967059 3,054326 3,141592

81(φ) 0,17453237 0,87259540 1,74477549 2,61612516 3,48622971 4,35467499 5,22104768 6,08493544 6,94592710 7,80361288 8,65758455 9,50743569 10,3527618 11,1931605 12,0282319 12,8575786 13,6808057 14,4975215 15,3073372 16,1098675 16,9047304 17,6915476 18,4699445 19,2395507 20 20,7509303 21,4919843 22,2228093 22,9430574 23,6523859 24,3504571 25,0369388 25,7115043 26,3738326 27,0236083 27,6605222 28,2842712

82(φ) 9,51923Ε-05 0,002379446 0,009513255 0,021387847 0,037980617 0,059259982 0,085185434 Ο, 115707623 Ο, 150768448 0,190301169 0,234230532 0,282472917 0,334936491 0,391521385 0,452119889 0,516616649 0,584888892 0,656806658 0,732233047 0,811024481 0,893030976 0,978096427 1,066058909 1 '156750979 1,25 1,345628467 1,443454346 1,543291419 1,644949642 1,748235501 1,852952387 1,958900965 2,065879556 2,173684519 2,282110643 2,390951532 2,5

81+82 0,174627 0,874974 1,754288 2,637513 3,524210 4,413934 5,306233 6,200643 7,096695 7,993914 8,891815 9,789908 10,68769 11,58468 12,48035 13,37419 14,26569 15,15432 16,03957 16,92089 17,79776 18,66964 19,53600 20,39630 21,25 22,09655 22,93543 23,76610 24,58800 25,40062 26,20340 26,99583 27,77738 28,54751 29,30571 30,05147 30,78427

-------

Σ(φ) 0,1745 0,8725 1,745 2,6175 3,49 4,3625 5,235 6,1075 6,98 7,8525 8,725 9,5975 10,47 11,3425 12,215 13,0875 13,96 14,8325 15,705 16,5775 17,45 18,3225 19,195 20,0675 20,94 21,8125 22,685 23,5575 24,43 25,3025 26,175 27,0475 27,92 28,7925 29,665 30,5375 31,41

-F(φ) F(φ)/Σ(φ)

-5 -6

ΕΥΚΛΕΙΔΗΣ Β' λη ' τ.l/4

F(φ) 1,27564Ε 0,002474 0,009288 0,020013 0,034210 0,051434 0,071233 0,093143 Ο, 116695 0,141414 0,166815 0,192408 0,217698 0,242181 0,265351 0,286695 0,305694 0,321828 0,334570 0,343392 0,347761 0,347144 0,341003 0,328801 0,31 0,284058 0,250438 0,208600 0,158007 0,098121 0,028409 0,051660 0,142616 0,244982 0,359281 0,486026 0,625728

F(φ)/Σ(φ) 7,31024Ε 0,002836 0,005323 0,007645 0,009802 0,011790 0,013607 0,015250 0,016718 0,018008 0,019119 0,020047 0,020792 0,021351 0,021723 0,021906 0,021897 0,021697 0,021303 0,020714 0,019929 0,018946 0,017765 0,016384 0,014804 0,013022 0,011039 0,008854 0,006467 0,003877 0,001085 0,001909 0,005108 0,008508 0,012111 0,015915 0,019921


π

I

I

Μαθηματικά «Ολισθήματα» Ελευθέριος Πρωτοπαπάς Το ποιύτο θέuα της επιστολής μου, έχει ως αφορμή της μία κουβέντα που είχα τους συναδέλ­ φους κ. Κεφάλα Δημήτρη και κ. Χαραλάμπους Πέτρο σχετικά με την τριγωνική ανισότητα. Στο 9° Κεφάλαιο, στην παράγραφο για τις γε­ νικεύσεις του Πυθαγορείου Θεωρήματος στις α­ σκήσεις όπου δίνονται τρία ευθύγραμμα τμήματα και ζητείται να εξετασθεί αν ορίζουν τρίγωνο και στη συνέχεια να βρεθεί το είδος του τριγώνου (σε­ λίδα 194: Ασκήσεις Εμπέδωσης 1 , 2 και αποδει­ κτική άσκηση 6), οι μαθητές καλούνται να χρησι­ μοποιήσουν την τριγωνική ανισότητα για να εξε­ τάσουν την ύπαρξη του τριγώνου. Όμως κοιτάζοντας προσεκτικά το βιβλίο σε επίπε δο θεωρίας δεν υπάpχει το αντίστοιχο κρι­ τήριο, αλλά υπάρχει μόνο το ευθύ της πρότασης, σύμφωνα με την οποία σε κάθε τρίγωνο ισχύει η τριγωνική ανισότητα. Αυτό σημαίνει πρακτικά ότι για να χρησιμοποιήσεις το αντίστροφο της πρότα­ σης πρέπει να το αποδείξεις. Κάτι τέτοιο βέβαια δεν ισχύει για το λυσάρι, όπου οι ασκήσεις που αναφέρονται στην ύπαρξη τριγώνου χρησιμο­ ποιούν ως προϋπόθεση την ισχύ της τριγωνικής α­ νισότητας. Στην σελίδα 57 στην 3η ερώτηση κατανόησης οι μαθητές ρωτούνται για την ύπαρξη ενός τριγώ­ νου δοθέντων τριών ευθυγράμμων τμημάτων. Το λυσάρι δεν δίνει απαντήσεις στις ερωτήσεις κατα­ νόησης, οπότε δεν μπορούμε να γνωρίζουμε ποιος είναι ο τρόπος λύσης κατά τους συγγραφείς. Ακολούθως παραθέτω την απόδειξη της πρό­ τασης που αναφέρω και «θεωρητικά» πρέπει να γράψουν και οι μαθητές στις εξετάσεις σε περί­ πτωση που ζητηθεί κάτι τέτοιο.

)ίο>

Αν για τα σημεία Α, Β, Γ ισχύει ότι: IAB - ΑΓΙ< ΒΓ< ΑΒ + ΑΓ, τότε τα σημεία Α, Β και Γ είναι κορυφές τριγώνου. Απόδειξη

Έστω ότι τα σημεία Α, Β, Γ είναι συνευθεια­ κά. Τότε θα ισχύει μία από τις ακόλουθες περι­ πτώσεις: •

Αν Α ανάμεσα από τα Β, Γ, τότε ΒΓ=ΑΒ+ΑΓ, που είναι ΑΤΟΠΟ, λόγω της δοσμένης σχέ­ σης. Αν Β ανάμεσα από τα Α, Γ, τότε ΒΓ=ΑΓ-ΑΒ, που είναι ΑΤΟΠΟ, λόγω της δοσμένης σχέ­ σης. Αν Γ ανάμεσα από τα Α, Β, τότε ΒΓ=ΑΒ-ΑΓ, που είναι ΑΤΟΠΟ, λόγω της δοσμένης σχέ­ σης.

Επομένως αφού έχουμε ΑΤΟΠΟ σf: κάθε δυ­ νατή περίπτωση, δεν θα είναι τα σημεία συνευ­ θειακά, άρα θα αποτελούν πλευρές τριγώνου. Επιπλέον των παραπάνω το λυσάρι στην επί­ λυση αυτών των ασκήσεων χρησιμοποιεί αποκλει­ στικά τη σχέση Ιβ - γΙ< α< β + γ, όπου η α είναι πάντα η μεγαλύτερη πλευρά. Δεν γνωρίζω αν αυτό είναι τυχαίο, αλλά προφανώς η τριγωνική ανισότη­ τα, ως προϋπόθεση ύπαρξης τριγώνου, δεν ισχύει αποκλειστικά και μόνο όταν στην μέση βρίσκεται η μεγαλύτερη πλευρά. Αυτό γίνεται εύκολα κατανοητό γιατί στην παραπάνω πρόταση, δεν αναφερόμαστε στο ΒΓ ως το μεγαλύτερο ευθύγραμμο τμήμα. Τέλος κοιτάζοντας τα παλιότερο βιβλίο της Γεωμετρίας Α ' Λυκείου των Θωμαϊδη, Ξένου, Πα-

ΕΥΚΛΕΙΔΗΣ Β' λη ' τ.l/5


Το

Βήμα του Ευκλείδη

ντελίδη, Πούλου, Στάμου διαπίστωσα ότι υφίστα­ ται το ίδιο πρόβλημα, κάτι το οποίο δεν συνέβαινε με το προηγούμενο των Αλιμπινίση, Δημακού, Εξαρχάκου, Κοντογιάννη, Τασσόπουλου.

Σε επικοινωνία που είχα με το Υπουργείο την ημέρα της διεξαγωγής των εξετάσεων η απάντηση ήταν κοινότυπη και αναμενόμενη: «ότι γράφει το σχολικό βιβλίο . . . ».

Και εδώ τίθεται ο προβληματισμός: κάθε λο­ Το δει5τεpο Οέuα της επιστολής μου έχει ως αφορμή ένα ερώτημα από τις περσινές Πανελλή­ γικά σκεπτόμενος μαθητής κατά τη διάρκεια των νιες εξετάσεις της Γεωμετρίας της Β ' Λυκείου και εξετάσεων θα προβληματίσθηκε πολύ έντονα για συγκεκριμένα από το 1 ο Θέμα το Β.δ στο οποίο το τι θα απαντήσει και προφανώς υπάρχουν σοβα­ διατυπωνόταν μία πρόταση και ζητούσαν από τους ρές πιθανότητες να απάντησε και λάθος ( σύμφωνα μαθητές να χαρακτηρισθεί ως Σωστή ή Λανθασμέ­ με το σχολικό βιβλίο και το Υπουργείο). Μήπως νη. Η πρόταση αυτή ήταν: πρέπει τελικά να είμαστε ως Μαθηματικοί πιο έ­ ντονοι στις αντιδράσεις μας σε τέτοια θέματα και «Η διαφορά των τετραγώνων δύο πλευρών πολύ περισσότερο να μην διαιωνίζουμε τα λάθη ενός τριγώνου ισούται με το διπλάσιο γινόμενο της του σχολικού βιβλίου; τρίτης πλευράς επί την προβολή της αντίστοιχης διαμέσου πάνω στην πλευρά αυτή». Το τοίτο θέuα της επιστολής μου έχει επίσης ως αφορμή ένα ερώτημα από τις περσινές εξετά­ Στο σχολικό βιβλίο διατυπώνεται η πρόταση: σεις της Γεωμετρίας της Β ' Λυκείου και συγκε­ «Η διαφορά των τετραγώνων δύο πλευρών ενός κριμένα από το 4° Θέμα το γ ερώτημα, στο οποίο τριγώνου ισούται με το διπλάσιο γινόμενο της τρί­ ζητούσαν από τους μαθητές να βρεθεί το εμβαδό της πλευράς επί την προβολή της αντίστοιχης δια­ του περιγεγραμμένου κύκλου ! ! ! Όλοι κατάλαβαν μέσου πάνω στην πλευρά αυτή». Σύμφωνα με αυ­ τι εννοούσε το θέμα και όλοι καταλαβαίνουμε ποιο είναι το λάθος. τή η δοσμένη πρόταση είναι σωστή (ΣΩΣΤΟ). Όμως η πρόταση αυτή χρησιμοποιώντας αυ­ στηρά Μαθηματικά είναι λανθασμένη (ΛΑΘΟΣ). αφού ενώ το πρώτο μέλος της ισότητας μπορεί να εί­ ναι θετικό ή αρνητικό ή μηδέν, το δεύτερο μέλος εί­ ναι πάντα μη αρνητικό (θετικό ή μηδέν). Προφανώς αυτό σημαίνει ότι και το θεώρημα με τον τρόπο που διατυπώνεται στο βιβλίο είναι ΛΑΝΘΑΣΜΕΝΟ. Παράδειγμα:

Στο διπλανό τρίγωνο ΑΒΓ έχουμε ότι ΑΒ < ΑΓ, ΑΔl..ΒΓ και ΑΜ διάμεσος. Τότε ισχύει: ./ ΑΓ2 - ΑΒ 2 = 2·ΒΓ·ΜΔ ενώ είναι λάθος η πρόταση ΑΒ 2 - ΑΓ2 2·�Γ·ΜΔ, αφού ΑΒ 2 - ΑΓ2 <Ο και 2·ΒΓ·ΜΔ > Ο. =

Α

Το τέταοτο θέuα της επιστολής μου αφορά ένα ερώτημα για το οποίο θα ήθελα να τοποθετη­ θεί η συντακτική ομάδα του περιοδικού και να προβληματιστεί κάθε συνάδελφος Μαθηματικός. «Γιατί από την ύλη των Πανελληνίων εξετά­ σεων της Β' Λυκείου στη Γεωμετρία δεν αφαιρού­ νται οι δύο τύποι που έχουν σχέση με Τριγωνομε­ τρία;» Προφανώς αναφέρομαι στον νόμο των συ­ νημιτόνων και στον τύπο για το εμβαδό ενός τρι­ γώνου . Ορθώς, κατά τη γνώμη μου, στο βιβλίο ανα­ φέρονται οι δύο τύποι, ώστε οι μαθητές να κατα­ λάβουν ότι τα Μαθηματικά είναι ενιαία και όλα συνδέονται μεταξύ τους. Όμως η χρήση αυτών των τύπων ξεφεύγει από αυτό που πραγματικά είναι και προσφέρει η Γεωμετρία στον μαθητή, ξέχωρα από το γεγονός ότι έτσι κάθε μαθητής «εμμέσως νομιμοποιείται» να χρησιμοποιήσει Τριγωνομετρία σε μια άσκηση Γεωμετρίας.

Β

Γ ΕΥΚΛΕΙΔΗΣ Β' λη ' τ.l/6


�� 8 -���� ��·

Η'Ν' NATHeNAT/t'l/f

Η Homo Mathematicus είναι μια στήλη στο περιοδικό μας, με σκοπό την ανταλλαγή απόψεων και την ανάπτυξη προβληματι­ σμού πάνω στα εξής θέματα: l) Τι είναι τα Μαθηματικά, 2) Πρέπει ή όχι να διδάσκονται, 3) Ποιοι είναι οι κλάδοι των Μαθημα­ τικών και ποιο το αντικείμενο του καθενός, 4) Ποιες είναι οι εφαρμογές τους, 5) Ποιες επιστήμες ή κλάδοι επιστημών απαιτούν καλή γνώση των Μαθηματικών για να μπορέσει κάποιος να τους σπουδάσει. Για τυυς συνεργάτες της στήλης: παράκληση! τα κείμενα της στήλης αυτής, ως προς το περιεχόμενό τους και ως προς το επί­ πεδό τους, θα πρέπει να είναι συμβιβαστά με τα ενδιαφέροντα και το επίπεδο κατανόησης από μέρους των παιδιών. ·

Ι. 11 που και με ποιο τρόπο εφαρμόζονται τα Λ1αθιzματικά; 11 Τι είναι οι αi.γόριθμοι

Είναι κάθε διαδικασία σκέψης που ακολου­ θείται με σκοπό την επίλυση κάποιου προβλήμα­ τος, δηλ. κάθε μέθοδος που με κατάλληλα διαδο­ χικά βήματα οδηγεί στο να δημιουργηθεί κάτι, πχ. να επιλυθεί μια εξίσωση, f(x)=O, να εκτελεσθεί ένα έργο, να πραγματοποιηθεί ένας απολογισμός, να συναχθεί ένα λογικό συμπέρασμα κτλ. Σχεδόν κάθε δραστηριότητα, έργο, σκέψη, πράξη ή υπολογισμός, που μπορεί να ακολουθείται ακόμη και ενστικτωδώς, όπως στην παρασκευή ενός φαγητού ή ακόμη και στο χτίσιμο της φωλιάς του χελιδονιού ή στην κατασκευή του κελύφους ενός οστράκου. Θεωρητικά σχεδόν κάθε είδος της πνευματικής δραστηριότητας του ανθρώπου θα μπορούσε να αναχθεί στην εκτέλεση του άλφα ή του βήτα αλγορίθμου. Στην πράξη όμως είναι πολύ δύσκολο να βρεθούν οι κανόνες που θα αποτελέ­ σουν αυτούς τους αλγόριθμους. Υπάρχουν και τα λεγόμενα αλγοριθμικώς άλυτα προβλήματα., δηλ. υπάρχουν προβλήματα που για να λυθούν, χρειά­ ζονται άπειρο αριθμό διαφορετικών πράξεων. Από την ίδια την δημιουργία ενός επαρκώς αναπτυγμένου συστήματος μαθηματικών συμβό­ λων, το πρόβλημα της κατασκευής αρκούντως γε­ νικών και συγχρόνως σύντομων αλγορίθμων κατέ­ χει μείζονα θέση στην ιστορία των Μαθηματικών. Αλλά, μόνο τις τελευταίες δεκαετίες έχουν ανα­ πτυχθεί η γενική θεωρία των αλγορίθμων και η θε­ ωρία "αλγοριθμικής επιλυσιμότητας" των μαθημα­ τικών προβλημάτων. Οι πρακτικές προοπτικές αυτών των θεωριών είναι εμφανώς μεγάλες, ιδιαίτερα σε σχέση με την σύγχρονη ανάπτυξη των υπολογιστικών μεθόδων, οι οποίες καθιστούν δυνατή την αντικατάσταση πολύπλοκων μαθηματικών αλγορίθμων από υπο-

του Γιάννη Κερασαρίδη

λογιστικά προγράμματα. Με βάση τους νεότατους μηχανισμούς και συ­ στήματα της κυβερνητικής αναπτύσσεται με γοργό ρυθμό ο aυτοματισμός των διαφόρων ειδών της πνευματικής δραστηριότητας του ανθρώπου. Ο aυτοματισμός αγκαλιάζει ολοένα και πιο νέες πε­ ριοχές, οι δυνατότητες του είναι απεριόριστες. Οι επιτυχίες του αυτοματισμού προκαλούν σε μερι­ κούς φιλοσόφους και συγγραφείς ανησυχίες για το μέλλον της aνθρωπότητας. Αναρωτιούνται μήπως τα αυτόματα εκτοπίσουν τον άνθρωπο. Ωστόσο οι φόβοι αυτού του είδους είναι αποτέλεσμα της ά­ γνοιάς τους. Οι μηχανές, όσο τέλειες κι αν είναι, θα παραμένουν πάντα πιστοί βοηθοί του ανθρώ­ που, συντελώντας στην απέραντη άνθιση των υλικών και των πνευματικών δυνάμεων της αν­ θρώπινης κοινωνίας.

Η πρώτη γνωστή προσπάθεια αναζήτησης μιας ταξινόμησης των διαφόρων νοημάτων και των α­ ληθειών που τα διέπουν, καθώς και προσδιορισμού καθολικών μεθόδων για την επεξεργασία τους έγι­ νε από τον Αριστοτέλη (384-322 πΧ .) στο έργο του «Συλλογιστική». Η ονομασία «αλγόριθμος» εισήχθη από τον Άραβα μαθηματικό Μούσα αλ­ Χουαριζμί (Musa al-Kwaήzmi, 780-850 μΧ.). Οι ιδέες του επηρέασαν το έργο των μεταγενεστέρων και ειδικότερα των G. Cardano, R. Descartes, G. Leibniz, G. Bool. Ο σημερινός (μαθηματικός) ορι­ σμός του αλγόριθμου και η σημασία του για το λογισμό βασίζεται στις - ισοδύναμες μεταξύ τους - θέσεις των Church ( 1 9 36), Tuήng ( 1 9 37), Markov ( 1 947).

[Βιβλιογραφία: ΑΚΑΔΗΜΑΪΚΗ(τόμ. 2, σ. 443,492), ΠΑ­ ΠΥΡΟΣ-ΛΑΡΟΥΣ(τόμ. 3, σ. 72-74)]

Π. 1 Α δ 1 1 υτ το ξέρατε; 1

Ποια καμπύλη ονομάσθηκε «φιλάρεσκη>> ή «άνθος του γιασεμιού»; [Η απάντηση στο τέλος της στήλης]

ΠΙ. 11 Οι συνεργάτες της στιίλιιc: γράφουv-ερωτοι5ν11 Το θεώρημα του Bolzano κα ι η Μετεωρολογία

Ένα δροσερό βράδυ του Ιουνίου βρέθηκα να κουβεντιάζω με μια παρέα μαθηματικών, ανάμεσα στους οποίους ήταν κι ένας που είχε κάνει «Καλά μεταπτυχιακά» στη Μετεωρολογία. Ο καθένας μας βάλθηκε να ΕΥΚΛΕΙΔΗΣ Β' λη ' τ.ΙΠ


Homo Mathematicus

βρει επιχειρήματα, για ποιο λόγο τα Μαθηματικά «χώνουν τη μύτη τους, παντού>> κι ο μετεωρολόγος μας, όταν ήρθε η σειρά του, μας είπε: «Μετεωρολογία χωρίς Μαθηματικά μοιάζει με έναν επίδοξο δρομέα των 1OOm, χωρίς πόδιω> και συνέχισε παινεύοντας τα Μαθηματικά. Ήταν εντυπωσιακά αυτά που μας είπε· τό­ σο εντυπωσιακά, ώστε ο εντυπωσιασμός μας μετατράπηκε σε έκπληξη όταν μας είπε «μπορείτε να βρείτε τι σχέση έχει το θεώρημα του Bolzano με τη Μετεωρολογία;». Από τότε βάλθηκα να ψάχνω το θέμα με σκο­ πό να βρω κάτι για να μη . . . υστερήσω του φίλου μου. Μετά από αρκετές ημέρες, βρήκα αυτό που ζήταγα 1 (μετά από υπόδειξη ενός καλού φίλου από τα Γιάννενα2). Σας παραθέτουμε μερικές περικοπές του άρθρου, τις οποίες κρίναμε πως είναι του επιπέδου των μαθητών μας της Γ Λυκείου. Και, μεταξύ μας, . . . ας μη το μάθει ο μετεωρολόγος μας, για να τον εκπλήξουμε στην επόμενη σύναξη. Α '. Το μετ εωρολογικό θεώρ η μα Borsuk-Uiam

Το Μετεωρολογικό θεώρημα

«Κάθε χρονική στιγμή, υπάρχει ένα ζεύγος, διαμετρικά αντίθετων, σημείων της υδρογείου («αντίποδες») που έχουν την ίδια θερμοκρασία και πίεση»

Αν και διατυπώσαμε αυτή την πρόταση με μετεωρολογικούς όρους, στην πραγματικότητα δεν αφορά ιδιότητες της ατμόσφαιρας αλλά των συνεχών συναρτήσεων που ορίζονται στη σφαίρα. Το θεώρημα ανήκει στο πεδίο της Τοπολογίας,

θεώρημα ανήκει στο πεδίο της Τοπολογίας, ενός κλάδου των Μαθηματικών που, μεταξύ των άλ­ λων, ασχολείται με συναρτήσεις ή σύνολα συναρτήσεων που είναι συνεχείς σε συγκεκριμένα σύνο­ λα. Μερικές ιδιότητες τέτοιων συναρτήσεων κα­ θορίζονται από τη δομή του συνόλου επί του οποί­ ου ορίζονται.

Μαθηματικές διατυπώσεις του μετεωρολογικού θεωρήματος

Ας διατυπώσουμε το μετεωρολογικό θεώρημα συναρτήσεων δεν μπορεί να αλλάζουν ουσιαστικά συναρτήσει μαθηματικών όρων. Σε μια δεδομένη όταν η θέση του σημείου Ρ μεταβάλλεται ελαφρώς. χρονική στιγμή, σε κάθε σημείο Ρ της επιφάνειας S Θα συμβολίζουμε με Ρ' το άκρο της διαμέτρου της Γης μπορούν να προσαρμοστούν δύο αριθμοί - μιας σφαίρας με άλλο άκρο το σημείο Ρ. η πίεση f(P) και η θερμοκρασία g(P). Συνεπώς, στη Αφήνοντας κατά μέρος τη «μετεωρολογική» σφαίρα ορίζονται δύο συναρτήσεις. Υποθέτουμε ότι σημασία των συναρτήσεων f και g, θα διατυπώ­ είναι συνεχείς. Πραγματικά, οι τιμές αυτών των δύο σουμε το θεώρημα στη γενική του μορφή.

Θεώρημα Borsuk- Ulam.

«Αν οι συναρτήσεις f,g ορίζονται σε σφαίρα S και είναι συνεχείς, τότε υπάρχουν δύο αντιδιαμετρικά σημείαΡο καιΡΌ της σφαίρας τέτοια ώστε f(PΌ)=f(Po) και g(PΌ)=g(Po)»

Θα εισαγάγουμε δύο συναρτήσεις, την F(P)=f(P ' )-f(P) και την G(P)=g(P ')- g(P). Και οι δύο συναρτήσεις είναι συνεχείς και αντισυμμετρικές. Έτσι

F(P')=f((P') ')-f(P')= f( P)-f(P ')=-F(P). Για τα σημεία Ρ0ε:S τα οποία είναι τέτοια ώστε f(P' o)=f(P0) και g(P ' o)=g(Po) και μόνο γι ' αυτά, έχουμε F(Po)=O και G( P0)=0. Επομένως το θεώρημα Borsuk-Ulam προκύ­ πτει από το επόμενο θεώρημα:

Θεώρημα της κοινής ρίζας:

«Αν οι συναρτήσεις F και G είναι συνεχείς και αντισυμμετρικές σε μια σφαίρα S, τότε υπάρχει ένα σημείο Ρο στο οποίο και οι δύο αυτές συναρτήσεις παίρνουν τιμή ίση με το μηδέν. Δηλ. F(P0)=G(P0)=0» Β'.

Rene Descartes: ο φιλό σοφος π ου ά λλαξε την πορεία τ ω ν Μαθ η ματ ικών

Στο προηγούμενο τεύχος του «Ευκλείδη Β '», γράφαμε «Κάποιοι φίλοι της στήλης, μας ζήτησαν να γράψουμε "δυο λόγια" για τον "μέγιστο", όπως τον αποκαλούν, μαθηματικό Rene Descartes (γνω­ στότερος στους Έλληνες ως Καρτέσιος). Εμείς δεν έχουμε καμιά αντίρρηση και τους υποσχόμαστε

πως θα το φροντίσουμε» . Να που σ ' αυτό το τεύ­ χος, πραγματοποιούμε την υπόσχεσή μας. Προκα­ ταβολικά πρέπει να σημειώσουμε πως ο Rene Des­ cartes πριν απ' όλα και κατά κύριο λόγο, ήταν φι­ λόσοφος και μετά μαθηματικός . Για τον Descartes

έχουν γραφτεί "άπειρες" μελέτες. Ας δούμε, όμως,

1 Πρόκειται για μια εργασία των Μ. Κrein και Α. Nudelman με τίτλο «Το θεώρημα Borsuk-Ulam», περιοδικό «QUANτuM», τόμ. 7, τεύχ. 5, σ. 38-42). Ο καλός φίλος της στήλης από τα Γιάννενα, που μου 'στειλε τη σχετική περικοπή, μου ζήτησε σαν αντάλλαγμα, να μη δημοσι­ εύσω το όνομά του γιατί, όπως μου είπε, «δεν νομίζω να έκανα κάτι σημαντικό>> . Παρ' ότι η στήλη διαφωνεί μ' αυτή του την επιλογή, εντούτοις σεβόμενη την επιθυμία του, κρατεί το όνομά του στο αρχείο της.

2

ΕΥΚΛΕΙΔΗΣ Β' λη ' τ.l/8


Homo Mathematicus •

Στοιχεία β ιογρ αφ ία ς

Ο Rene Descartes, γνωστός και με το λατινικό του Γαλιλαίου και ο Descartes, παλαιός μαθητής όνομα Renatus Cartesius, εξ ου και το ελληνικό των Ιησουϊτών, αποφάσισε να παραιτηθεί της δη­ Καρτέσιος, γεννήθηκε στην La-Haye (Indre-et­ μοσίευσης του έργου του. Έτσι επιδόθηκε σε μια Loire, Touraine) στις 3 1/3/1 596 από εξέχουσα οι­ προσπάθεια να εκθέσει υπό μορφή λιγότερο επι­ κογένεια όχι της «grande» - όπως έλεγαν τότε - κίνδυνη τ ' αποτελέσματα των μακρών μελετών αλλά της «petit noblesse>> . Μόλις βγήκε από την του κι έτσι γεννήθηκαν τα τρία διάσημα δοκίμια: παιδική ηλικία, στάλθηκε (Πάσχα του 1604) στο «Διοπτρική» ( «Le Dioptrίque» ) «Μετέωρα» ( «Les Βασιλικό Κολλέγιο της La Fleche (Anjoue). Το Meteores» ) «Γεωμετρία» ( «Geometrie» ). Ενώ είχε τελειώσει τα δύο πρώτα και ασχολιό­ 1615-1616 στο Poitiers, γράφτηκε στην Νομική σχολή, αλλά αφιερώθηκε με πάθος στην Ιατρική ταν με τη συμπλήρωση του τρίτου, ο Descartes, με το όνειρο να συνεχίσει την μακρά οικογενειακή αναγνώρισε, ότι για να δώσει στα έργα του οργα­ παράδοση. Κατά τα έτη 1617-1620 ασχολείται με νική ενότητα έπρεπε να προτάξει ένα πρόλογο. Ο στρατιωτικά καθήκοντα. Το 1628 μετακόμισε πρόλογος αυτός, τελικά, έγινε ολόκληρο δοκίμιο κρυφά στην Ολλανδία (όπου συχνά άλλαζε κατοι­ με το διάσημο τίτλο «Λόγος περί μεθόδου δια να κία), προτιμώντας τη χώρα αυτή όχι μόνο γιατί οδηγεί κανείς καλώς το λογικό του και ν' αναζητεί είχε εκεί καλούς φίλους, αλλά κυρίως γιατί αισθα­ την αλήθεια στις επιστήμες» ( «Discours de la me­ νόταν τον εαυτό του ασφαλή από τις αρπάγες της thode pour bien condouίre sa raison et chercher la νerίte dans les sciences» ). Το έργο τούτο, συνο­ πανίσχυρης Ιεράς Εξέτασης. Ένα έκτακτο αστρονομικό φαινόμενο, το δευόμενο από τα τρία άλλα δοκίμια, δημοσιεύθηκε 1629, τον παρακίνησε να αφοσιωθεί σε μελέτες το 1637 στο Leiden, όπου ο Descartes είχε εντωμε­ ουράνιας φυσικής. Στις 16/4/1630, με επιστολή ταξύ εγκατασταθεί. Τα έργα αυτά είχαν τεράστια του προς τον Marin Mersenne, έκφραζε την πνευ­ απήχηση στις τάξεις των διανοουμένων κι ο Desματική του διάθεση με τη δήλωση ότι «έχω τόσο cartes έγινε γνωστός παντού. . Την 1/1 1/1649 ο Descartes γινόταν προσωπι­ βαρυνθεί με τα μαθηματικά, ώστε να μη επιτυγχά­ νω με αυτά πολλά πράγματα» . Οι σπουδές του κός δάσκαλος της βασίλισσας Χριστίνας της Σου­ στη φυσική του ουρανού διαρκούν μέχρι το 1633, ηδίας. Το δριμύ ψύχος κι η παράλογη απαίτηση καρπός δε τούτων υπήρξε ένα έργο του με τίτλο της Χριστίνας ώστε τα μαθήματα να αρχίζουν από «Περί του Κόσμου», όπου το Κοπερνίκειο σύστη­ τις 5 το πρωί, κλόνισαν την υγεία του και στις μα, που κατείχε κεντρική θέση, παρουσιαζόταν με 1 1/2/1650 πέθανε από οξεία πνευμονία στη Σουη­ ευνοϊκό φως. Όμως, ήδη, η Ιερά Εξέταση είχε δία. βγάλει την καταδικαστική απόφαση για τις ιδέες ,

,

Ο μαθ η ματ ικό ς Descartes

Η «Γεωμετρία» είναι το μοναδικό μαθηματι- ισάριθμες γεωμετρικές κατασκευές. Έχουμε την κό έργο που δημοσίευσε ο Descartes. Όλες οι άλ- καθιέρωση αυτού που σήμερα ονομάζουμε «σύ­ λες μαθηματικές απόψεις του βρίσκονται στην στημα αναφοράς» ή «άξονες συντεταγμένων» ή πλούσια αλληλογραφία του. Η «Γεωμετρία» του «καρτεσιανούς άξονες» ή . . . όπως αλλιώς στη μα­ χωρίζεται σε τρία βιβλία. Βιβλίο Ι. Αφιερώνεται θηματική φιλολογία αναφέρεται. Βιβλίο 11. Το στα προβλήματα που λύνονται με κανόνα και δια- κύριο αντικείμενο του βιβλίου αυτού είναι η σπου­ δή των καμπυλών που παριστάνονται από εξισώ­ βήτη. Περιέχει τα θεμέλια της διαδικασίας εκείνης, χάρις στην οποία ο Descartes μετασχημάτιζε το σεις με δύο άγνωστους. Βιβλίο 111. Εδώ ασχολεί­ γεωμετρικό πρόβλημα σε αλγεβρικό. Βάση αυτής ται με τη γραφική λύση των στερεών και υπερστε­ είναι η παραδοχή [που έγινε από τον Απολλώνιο ρεών καμπυλών. Ο Descartes είχε πλήρη συνείδηση της σπου­ τον Περγαίο-τέλος 3ου με αρχές 2ου αιώνα πχ.- και, αργότερα, από τον Bombelli σε σελίδες που παρέ- δαιότητας του έργου του. Σε μια επιστολή προς μειναν ανέκδοτες επί πολλά χρόνια] ενός μήκους, τον Marin Mersenne (τέλος Δεκέμβρη 1637), έ­ αυθαίρετου αλλά σταθερού, σαν μονάδας μέτρη- γραφε πως τα πράγματα τα οποία διδάσκει «θίγο­ σης όλων των μηκών, γνωστών και αγνώστων, των ντα τη φύση και τις ιδιότητες των καμπύλων εισερχομένων στο πρόβλημα, το οποίο υποτίθεται γραμμών και τον τρόπο της μελέτης τους, βρίσκο­ ήδη λυμένο, κατ ' εφαρμογή της αναλυτικής μεθό- νται τόσο πολύ πέραν της συνήθους Γεωμετρίας δου των αρχαίων. Ο Descartes απόδειξε ότι άμεσος όσο η ρητορική του Κικέρωνα απέχει από τους συνέπεια του συστήματος αυτού είναι ότι οι θεμε- παιδικούς φθόγγους a,b,c» λιώδεις πράξεις της αριθμητικής μεταφράζονται σε ΕΥΚΛΕΙΔΗΣ Β' λη ' τ.l/9


Homo Mathematicus •

Ο φιλό σο φ ος Descartes

Διάσημος Γάλλος σοφός. Ο Καρτέσιος παίρνει δυϊστική θέση πάνω στο βασικό ζήτημα της φιλο­ σοφίας, στο ζήτημα δηλ. της σχέσης ανάμεσα στη νόηση και το είναι. Παραδέχεται την ύπαρξη δύο ουσιών: την ουσία του σώματος του οποίου κατηγο­

στήμη την τυφλή πίστη. Καταφεύγει στην «αμφι­ βολία» σαν μέθοδο συλλογισμού. Χάρη στην αμ­ φιβολία ελευθερωνόμαστε από κάθε προεσκεμμέ­ νη ιδέα ή συνηθισμένη γνώση, και καθορίζουμε αλήθειες αναμφισβήτητες. Ισχυρίζεται ότι: αμφι­

ρούμενο (βασική ιδιότητα) είναι το ότι υπάρχει στο χώρο (έκταση) και την ουσία της ψυχής, της οποίας κατηγορούμενο είναι η σκέψη. Δηλ. δύο ανεξάρτη­ τες ουσίες, δύο αρχές: μια υλική και μια πνευματι­ κή. Η ύπαρξη του σώματος και της ψυχής καθορί­ ζεται από μια τρίτη ουσία (αρχή): το θεό .

βάλει για την αλήθεια, για την ακρίβεια των παρα­ στάσεων και των αντιλήψεών μας για τον κόσμο, αμφιβάλει ακόμη και για την ύπαρξη του ίδιου του κόσμου. ΑJ.λά ενώ αμφιβάλλει για όλα, παραδέχε­

Στη φυσική του ο Καρτέσιος υποστηρίζει υλι­ στικές θέσεις. Η φύση, κατά τη γνώμη του, είναι μια συνεχής συνένωση υλικών μορίων" η ουσία, η βασική ιδιότητα της ύλης, είναι η ύπαρξή της σε χώρο (έκταση). Η κίνηση του υλικού κόσμου, εί­ ναι αιώνια και πραγματοποιείται σύμφωνα με τους νόμους της μηχανικής: συντελείται δηλ. με την απλή μετακίνηση των μορίων, των ατόμων μέσα στο χώρο. Στη φυσική του, ο Καρτέσιος, απέδωσε στην ύλη μια ικανότητα αυτοδημιουργίας και υπογράμ­ μισε ότι η μηχανική κίνηση ήταν η ζωτική της εκ­ δήλωση ... Στα όρια της φυσικής του, η ύλη είναι η μοναδική ουσία, ο μοναδικός λόγος του είναι και της γνώσης. Ο Καρτέσιος απέρριψε τη φιλοσοφία του με­ σαίωνα, και αρνήθηκε την αυθεντία της Εκκλησί­ ας. Έχοντας βαθιά πίστη στη δύναμη της λογικής του ανθρώπου προσπάθησε να δημιουργήσει μια νέα, επιστημονική μέθοδο για τη γνώση του κό­ σμου, να αντικαταστήσει με τη λογική και την επι-

ται εντούτοις, είναι βέβαιος νός τουλάχιστον πράγματος:

για την ύπαρξη ε­

την ύπαρξη της ίδιας της αμφιβολίας του και του φορέως της, δηλ. του εαυτού του. Και ο Καρτέσιος καταλήγει σ ' αυτό το περίφημο ιδεαλιστικό συμπέρασμα: «νοώ, άρα υπάρχω» (Cognito, ergo sum). Έτσι, ξεκινώντας

από το γεγονός της ύπαρξης του ίδιου του «εγώ», φτάνει στο συμπέρασμα ότι και ο εξωτερικός κό­ σμος υπάρχει επίσης. Αυτή η θέση του Καρτέσιου

θα χρησιμοποιηθεί αργότερα από τον υποκειμε­ νικό ιδεαλισμό. Έτσι, πατέρας του ορθολογισμού (ρασιοναλι­

σμού) είναι ο Καρτέσιος. Πίστευε ότι οι αισθήσεις δεν μπορούν να μας δώσουν παρά μόνο μια συγκεχυμένη αντίληψη των αντικειμένων, και μ ' αυτό τον τρόπο μπορεί να μας οδηγήσουν σε λάθη. Η λογική κατακτά την αλήθεια με την ενόραση που είναι ιδιότητά της. Η ακρίβεια μιας αλήθειας επιβεβαιώνεται όχι από την πράξη και την εμπειρία, αλλά από την καθαρό­ τητα και την σαφήνεια των ιδεών μας. Το κριτή­ ριο, λοιπόν, της αλήθειας βρίσκεται μέσα στην ίδια τη λογική.

[Βιβλιογραφία: l)«Ιστορία των Μαθηματικών>>(GINO LORIA, τόμ. 2°ς, σελ. 234-251), 2) «Πλήρες Μικρό Φιλοσοφικό Λεξικό

>>(Μ. ΡΟΖΕΝΤΑΛ- Π. ΓΟΥΝ τΙΝ, σελ. 214), 3) <<01 ΜΑΘΗΜΑτΙΚΟΙ>> (Ε. Τ. BELL, τόμ. 11, σελ. 53-85), 4) <<Εγκυκλοπαί­

δεια ΠΑΠΥΡΟΣ-ΛΑΡΟΥΣ>>,(τόμ. 32, σ. 243-248), 5) <<ΛΕΞΙΚΟΝ ΦΙΛΟΣΟΦΙΑΣ>>(Χρ. Ανδρούτσου, σ. 203)]

Πα. "Αυτό το ξέρατε,·" [η α πά ντη ση ]

Να τι απαντά ο Gino Loήa στην «Iστορία των Μα­

έναν σε καθένα από τα άλλα τρία τεταρτοχώρια. Από

θηματικών» (τόμ. 2, σ. 242). Πρόκειται για την 3 3 « ...καμπύλη με εξίσωση x +y =nxy, την οποία ε­

δή θύμιζε το σχήμα του κόμπου μιας γραβάτας ή ενός

πινόησε ο ίδιος (ο Rene Descartes) και για τούτο ονο­ μάζεται "φύλλο του Καρτεσίου". Η καμπύλη αυτή ορί­

λουλουδιού, οδήγησε τον

κύκλου του να ονομάσουν την καμπύλη αυτή "baland'' ή

ζεται σε μια επιστολή του προς τον Mersenne (Ιανουά­

"fleur de jasmin". Τα ονόματα αυτά βρίσκονται στην

ριος 1638), συνοδευόμενη από ένα σχήμα εντελώς ε­

σφαλμένο. Φαίνεται πως η καμπύλη αυτή τράβηξε αμέ­

'δω ένα εξωφρενικό σύμπλεγμα γραμμών, το οποίο επει­

G.P. Roberval και άλλους του

υπό ημερομηνία 17 Ιουλίου 1638 επιστολή του Rene Descartes προς τον Marin Mersenne, απ' την οποία εξά­

σως την προσοχή του Gilles Personne Roberval, ο οποί­

γεται το συμπέρασμα, ότι ούτε ο μεγάλος φιλόσοφος

ος νόμισε ότι βρήκε το ακριβές σχήμα της (<< ένα είδος

γνώριζε με ακρίβεια το σχήμα της καμπύλης, πράγμα

ωοειδούς>> ). Επειδή, όμως, αγνοούσε τη σημασία των

που επικυρώνεται κι απ' την επόμενη επιστολή του της

σημείων των συντεταγμένων, πρόσθεσε στο βρόχο του πρώτου τεταρτοχωρίου των αξόνων άλλους τρεις, ανά

23 Αυγούστου 1638, όπου προσδιορίζεται το μήκος του

βρόχου ...>>

ΕΥΚΛΕΙΔΗΣ Β' λη ' τ.l/10


για �

Για

να

Μαθηματικά την Α'

κατανοήαουμε

τάξη

του

Λυκείου

καλύτεpα τοuς

αpιθμούς Γιάννη Στρατή

Στην εργασία που ακολουθεί γίνεται προσπάθεια μέσα από συγκεκριμένα παραδείγματα να βοηθήσουμε τα παιδιά στα πρώτα βήματά τους στο Λύκειο. Τα Μαθηματικά γενικά μας επιτρέπουν να ακονίζουμε το μυαλό μας. Είναι ωστόσο απαραίτητο να μπο­ ρούμε να διαγράφουμε μια πορεία αποτελεσματική για να απαντάμε με τις δικές μας δυνάμεις στα ζητήματα που τίθενται.'Και γι' αυτό επιβάλλεται να εξοικειωθούμε με τις έννοιες και τις μεθόδους.

Α'

Απάντηση

Ταυτότητες (σελ. 18 -19).

της υπόθεσης, μετά από απαλοιφ� πα­ Ενδιαφέρον έχουν οι ταυτότητες «υπό περιορι­ Η συνθήκη ρονομαστών, γίνεται: σμό»· συνήθως μπαίνουν με τη μορφή <<Vα αποδει(1) βγ + 2αγ + 3αβ = 0 χθεί η . ...... (ισότητα), αν είναι γνωστό ότι ισχύει η Έχουμε τώρα: παρακάτω .... (συνθήκη)» .. (6α + 3β + 2γ) 2 = Παράδειγμα: = 36α2 + 9β 2 + 4γ 2 + 36αβ + 12βγ + 24αγ Να αποδειχθεί ότι: (3α-4α3)2 +(4β3 -3β)2 = ι, ή (6α + 3β + 2γ) = αν είναι γνωστό ότι: α2 + β2 = ι . = 36α2 + 9β 2 + 4γ 2 + 12(βγ + 2αγ + 3αβ) Απόδειξη

(\)

Το πρώτο μέλος της αποδεικτέας γίνεται: Α = (9α2 - 24α4 + 16α6 ) + (16β6 - 24β4 + 9β 2 ) = 9(α2 + β 2 ) - 24(α4 + β4 ) + 16(α6 + β6 ) = 9 - 24 [ (α 2 + β 2 ) 2 - 2α2 β 2 ] + 16(α2 + β 2 ) = [ (αz + β z ) z _ 3αz β z ] = 9 - 24 ( 1 - 2α2 β 2 ) + 1 6(1 - 3α2 β 2 ) = 1 .

Δίνουμε παρακάτω μερικά παραδείγματα στις ταυ­ τότητες υπό περιορισμός: 1.

Αν για τους μη μηδενικούς αριθμούς α, β, γ ι 2 3 ο , , ει ισχυει: - + - + - = , τοτε να απο δειχθ' α β γ ότι: (6α + 3β + 2γ)2 = 36α2 + 9β2 + 4γ2 •

= 36α2 + 9β 2 + 4γ 2 •

2.

Αν α*β*γ*α και α3-3α2 +κα = β2-3β2 +κβ = γ3-3γ2+κγ, τότε να αποδειχθεί ότι: α + β + γ = 3 . Απάντηση:

α3 - 3α 2 + κα - β 3 + 3β 2 - κβ = 0 ή (α3 - β 3 ) - 3(α2 - β 2 ) + κ(α - β) = 0 ή (α - β) [ (α2 + αβ + β 2 ) - 3(α + β)+κ ] = Ο ή α 2 + αβ + β 2 - 3(α + β) + κ = Ο [αφού: α - β*Ο ] (1)

ΕΥΚΛΕΙΔΗΣ Β ' λη ' τ.Vl l

(2)


Μαθηματικά για την Α' Λυκείου

Αφαιρούμε κατά μέλη τις (1) και (2) και βρίσκου­

να αποδειχθεί ότι: «αν α α*β* γ :1:- α , τότε: z z β α ----=,...-2α2 +βγ 2β2 +αγ

με:

α 2 -γ2 + β(α-γ) -3(α-γ) = 0 ή (α-γ)[(α + γ) + β-3) = 0 ή α + β + γ-3 = 0 [αφού: α;t:γ ].

3.

+

γz 2γ2 +αβ

και

= ι ».

Απάντηση Για το πρώτο:

Αν για τους μη μηδενικούς αριθμούς κ, α, 6κ και β, γ, ισχύει: 2α 3β

+ +γ=

-3 + -2 + -6 = -ι α

β

γ

κ

,

(2), τότε να αποδειχθει ότι

είναι αληθής μια τουλάχιστον απ' τις ισό­ τητες: α = 3κ , β = 2κ, γ = 6κ.

Αρκεί να αποδειχθεί ότι: «το γινόμενο (α-3κ)(β-2κ)(γ-6κ) είναι μηδέν». Μετά από απλές πράξεις παίρνουμε: (α-3κ)(β-2κ)( γ - 6κ) = = αβγ-κ(3βγ + 2αγ + 6αβ) + 6κ 2 (2α + 3β + γ - 6κ) (2 ) = αβγ-αβγ = 0 (I) [αφού: κ(3βγ + 2αγ + 6αβ) = αβγ ] . Αν τα μήκη α, β, γ των πλευρών τριγώνου

(j + �) + (� + �) + (� + j) =

ΑΒΓ α

επαληθεύουν β

τη

γ

συνθήκη: 6,

τότε

να αποδειχθεί ότι: «το τρίγωνο είναι ισό­ πλευρο». Απάντηση

= α 2 (β-γ) + β 2 γ - αβ 2 + αγ 2 - βγ 2 = = α 2 (β - γ) - α(β 2 -γ 2 ) = (β - γ) [ α2 + βγ-α(β + γ) J = = (β - γ) [ α( α - β) - γ( α - β)] = (α - β)(β-γ)(α - γ) = -(α - β)(β-γ)(γ - α) π

Για το δεύτερο:

Απάντηση

4.

+

+β+ γ= Ο

Δείχνουμε κατ' αρχάς ότι οι παρονομαστές είναι διαφορετικοί απ' το μηδέν (και συνεπώς τα κλά­ σματα έχουν νόημα) 2α 2 + βγ = α2 + α2 + β(-α - β) = = α(α-β) + (α 2 - β 2 ) = (α - β) [α + (α + β)] = (α-β)(α - γ) = -(α-β)(γ - α);t:Ο (2) Όμοια εργαζόμαστε και δείχνουμε: (3) 2β 2 + αγ = -(α - β)(β - γ);t:Ο και 2γ 2 + αβ =-(β-γ)(γ - α) :1:- Ο. Άρα: Α =+ (α- -α) + (α - - γ) (β - -α)

[ α2β�;γ(β-γ) + β2 (γ -�;βα) + γ2 (α - β) γ�;γ ]

ή Α=

(α - β)(β-γ)(γ-α) = -(α - β)(β-γ)(γ-α) = 1 (α-β)(β - γ)(γ-α)

=

Μετά την απαλοιφή παρονομαστών η συνθήκη γί­ νεται: α 2 (β + γ) + β 2 (α + γ) + γ 2 (α + β)-6αβγ = Ο 6. Δίνεται η παράσταση: ή Π = 4α(4α -5) + 3β(3β + 5) -24αβ. β(α 2 +γ 2 -2αγ)+γ(α 2 -β 2 -2αβ)+α(β 2 +γ 2 -2βγ)=0 Να βρεθεί η αριθμητική τιμή της παρά­ ή στασης, αν είναι γνωστό ότι: 4α -3β = 7 (ι) β( α-γ) 2 + γ(β-α) 2 + α( γ - β) 2 = Ο. Και συνεπώς: α = β = γ. Απ άντη ση (Διαφορετικά η παράσταση του πρώτου μέλους Έχουμε: Π = 16α 2 -20α + 9β 2 + 15β - 24αβ θάτανε θετικός αριθμός). = (16α 2 + 9β2 -24αβ) - 5(4α - 3β) = (4α - 3β) 2 - 5(4α-3β) = 7 2 - 5 · 7 = 14 . 5. Αφού γίνει γινόμενο η παράσταση: Π = α2(β- γ) +β2(γ-α) +γ2(α -β) , ΕΥΚΛΕΙΔΗΣ Β' λη ' τ.l/12


Μαθηματικά yια την Α' Λυκείου

7.

Για τους αριθμούς α, β, γ που είναι διαφο­ ρετικοί του -1, δεχόμαστε ότι: αβγ = 1. Αν αβ +α +1 :;e 0 , να αποδειχθεί ότι η παράσταση

Π=

1 β β +1--α +1

+

1 γ γ +1--β +1

+

1 α+1-� γ +1

είναι καλά ορισμένη. Στη συνέχεια να υπο­ λογιστεί η αριθμητική τιμή της. Απάντηση

Ο παρονομαστής του πρώτου κλάσματος γίνεται: αβ + α + 1 και είναι προφανώς διαφορετικός απ ' α+1 το μηδέν. Όμοια ο παρονομαστής του δευτέρου κλάσματος γίνεται: βγ + β + 1 βγ + β + αβγ = β+1 β+1 = β(γ + 1 + αγ) = βγ(1 + αβ + α) :;e O β+1 β+1 Επίσης ο παρονομαστής του τρίτου κλάσματος γί­ αγ + γ + 1 γ( α + 1 + αβ) Ο = :;t: νεται: γ+1 γ+1 Τώρα για την αριθμητική τιμή της παράστασης έ­ χουμε: γ+1 β+1 π α+l = + + αβ + α + 1 βγ(αβ + α + 1) γ(αβ + α + l) α + 1 + α(β + 1) + αβ(γ + 1) αβ + α + 1 2α 2αβ + 1 + αβγ 2(α + αβ + 1) _ + =2 αβ + α + 1 αβ + α + 1 [αφού: αβγ = 1] _

.

Ο αριθμητής γίνεται: β 2 γ - βγ 2 + αγ 2 - α2 γ + αβ(α - β) = = γ \ α - β) - γ(α 2 - β 2 ) + αβ(α - β) = (α - β) [ γ 2 - (α + β)γ + αβ ] = (α - β)(γ - α)(γ - β) ( 1 ) Επίσης: α β Β = -- + + Υ = β -γ γ-α α-β α(γ - α)(α - β) + β(β - γ)(α - β) + γ(β - γ)(γ - α) _ (β - γ)(γ - α)(α - β) Ο αριθμητής γίνεται: (α - β) [ α( γ - α) + β(β - γ) ] + γ(β - γ)(γ - α) = = (α - β) [ γ( α - β) - (α2 - β 2 ) J + γ(β - γ)(γ - α) = (α - β) 2 [ y - (α + β) ] + γ(β - γ)(γ - α) = γ [ 2(α - β) 2 + βγ - αβ - γ 2 + αγ J =

= r [ 2(α + β) 2 - 9αβ + γ( α + β) - y 2 ] = -9αβγ* (2) (I)

Συνεπώς Π = ΑΒ = (2) -9αβγ -α( α - β)(β - γ)(γ - α) =9 αβγ (β - γ)(γ - α)(α - β) Αφού : α +β γ *

Β'

8.

Αν α :;t: β :;t: γ :;t: α και α + β + γ = Ο , να βρεθεί

[

)[

η τιμή της παράστασης γ-α

α β-γ Π= 7 + -+ β γ

α β-γ

+

β γ-α

Απάντηση

β-Υ Υ-α α-β = + + α γ β βγ(β - γ) + αγ(γ - α) + αβ(α - β) = ���--�--���� αβγ

'Εχουμε: Α =

+

γ α -β

)

=

Εξισώσεις της μορφής α χ + β = Ο (σελ. 23 - 24).

---

_

--

--

I . Ο ι σ υ ντ ελε στές είναι σ υγκεκριμένοι.

1.

Να επιλυθεί η εξίσωση: 1 5 1 1 - (1 - 4χ) - - (χ + 1) = - (χ-4) +5

4

w

4

Απάντη ση

Πολλαπλασιάζουμε και τα δύο μέλη της εξίσωσης επί 20 (το ελάχιστο κοινό πολλαπλάσιο των παρονομαστών) κι έχουμε: 4(1 - 4χ) - 5(χ + 1) = (χ - 4) + 25 <:::> 4 - 16χ - 5χ - 5 = χ - 4 + 25 <:::> -22χ = 22 <:::> χ = -1 . Δηλαδή η εξίσωση έχει μια μόνο λύση τη χ = - 1 .

ΕΥΚΛΕΙΔΗΣ Β ' λη ' τ.l/13


Μαθηματικά για την Α ' Λυκείου

2.

Να επιλυθεί η εξίσωση: ι

Π.

2

ι

-- + -- = -χ-ι

χ + ι χ2 - ι

Ε ξισώσεις με παράμετρο

Η διαδικασία είναι συνυφασμένη με διε­

ρεύνηση .

Απάντηση

Κατ' αρχάς παρατηρούμε ότι: «Ο άγνωστος χ δε μπορεί να πάρει τις τιμές ± 1 (μηδενίζεται κάποιος παρονομαστής· κι αυτό δεν επιτρέπεται)» Δηλαδή ο άγνωστος υπόκειται στον περιορισμό χ :;:. ± 1 . Κατόπιν αυτού η εξίσωση γίνεται (με απαλοιφή παρονομαστών) (χ + 1) + (χ - 1) = 2 ή 2χ = 2 ή χ = 1 . Η τιμή αυτή όμως απορρίπτεται (αντίκειται στον περιορισμό). Συμπεραίνουμε έτσι ότι η εξίσωση είναι αδύνατη.

3. Να επιλυθεί η εξίσωση: χ-ι

χ+2

χ+ι

χ-2

χ+2

χ-ι

χ-2

χ+ι

-- + -- = -- + -­ Απάντηση

Θέτουμε κατ' αρχάς τους περιορισμούς: χ :;:. ±1, χ :;:. ±2 . Η εξίσωση είναι ισοδύναμη με την εξίσωση: χ+1 χ-2 χ+2 χ -1 -- 1 + -- - 1 = -- - 1 + -- - 1 <::::> χ -1 χ-2 χ+1 χ+2 -3 3 3 -3 <=> -- + -- = -- + -- <=> χ +2 χ -1 χ-2 χ +1 1 1 1 1 <=> -- - -- = -- - -- <=> χ -1 χ +2 χ-2 χ +1 3 3 = <=> <=> (χ - 1)(χ + 2) (χ - 2)(χ + 1) <::::> (χ - 1)(χ + 2) = ( χ - 2)(χ + 1) <=> χ 2 + χ - 2 = χ 2 - χ - 2 <=> χ = - χ <=> χ = 0 .

ι.

Να επιλυθεί η εξίσωση: λ λ2 - (λχ + 3) = - + λχ + 2 2

είναι ο άγνωστος, λ η παράμετρος). Απάντηση

Η εξίσωση, μετά την απαλοιφή των παρονομα­ στών, γίνεται: 2λ(λχ + 3) = λ2 + 4λχ + 8 <=> <=>

2λ(λ - 2)χ = (λ - 2)(λ - 4)

Φυσικά η εξίσωση επιλύεται και με τη συνήθη μέ­ θοδο : με απαλοιφή παρονομαστών. Μετά από κα­ μπόσες πράξεις ξαναβρίσκουμε την ίδια λύση ( τα μαθηματικά είναι ισότροπα).

(1)

Για λ = Ο η (1) γίνεται: Ο · χ = 8 και είναι αδύνατη. Δηλαδή δεν υπάρχει τιμή του αγνώστου, που την επαληθεύει. Για λ = 2 ( 1 ) γίνεται: Ο · χ = Ο και είναι αόριστη. Δηλαδή έχει άπειρες το πλήθος λύσεις. Στην προ­ κειμένη περίπτωση η (1) επαληθεύεται για κάθε «πραγματική» τιμή του χ. (Δηλαδή εξίσωση είναι ταυτότητα). Για λ :;:. Ο και λ :;:. 2 η (1) έχει μία μόνο λύση: λ-4 Χ = -- . 2λ 2.

Να επιλυθεί η εξίσωση:

2χ + ι χ-λ

=

λ · όπου

λ ε IR. (παράμετρος). Απάντηση

Κατ ' αρχάς θέτουμε τον περιορισμό: χ :;:. λ ( 1 ) Η εξίσωση γίνεται: (λ - 2)χ = λ2 + 1 (2) Για λ = 2 η (2) γίνεται: Ο · χ = 5 και είναι αδύνατη. Για λ :;:. 2 η (2) έχει τη μοναδική λύση:

Δηλαδή η εξίσωση έχει μία μόνο λύση τη χ = Ο.

Σχόλιο :

4

(3) Η λύση, που δίνει ο τύπος (3), είναι δεκτή, σύμ­ φωνα με τον περιορισμό ( 1 ), μόνον όταν: λ2 + 1 :;:. λ η, λ-2 -1 ισοδύναμα: λ2 + 1 :;:. λ2 - 2λ <::::> λ :;:. - . 2

ΕΥΚΛΕΙΔΗΣ Β ' λη ' τ.l/14


Μαθηματικά για την Α ' Λυκείου

Τελικά η δοσμένη εξίσωση έχει τη μοναδική λύση, που δίνει ο τύπος (3), μόνον όταν: λ '* 2 και -1 λ -:f:. - . 2

Διάταξη

Γ' I.

Μόνιμες ανισότητες

Να αποδειχθεί ότι: «αν α 2 + αβ + β 2 = Ο , τότε: α = β = 0»

1.

Απ όδε ιξη

111.

Προβλή ματα

1.

Ένας ποδηλάτης διανύει μιαν απόσταση με (μέση) ταχύτητα 4o kmΛ . Αν όμως διατρέξει την ίδια απόσταση με (μέση) ταχύτητα 5o kmΛ , τότε φτάνει στο τέρμα ι ώρα γρηγορότερα. Ποια είναι η απόσταση;

Έχουμε:

� 2.

Απά ντ η ση

( %) 2 + % β 2 = 0 �

α 2 + αβ + β 2 = 0 � α +

{ }

α+� = Ο �α=β=Ο 2 β=Ο

Να αποδειχθεί ότι: «για κάθε χ ε JR ισχύει: 2χ2 - 4χ + 5 > ο ».

Ας είναι L η απόσταση. Τότε για τον χρόνο t της μετακίνησης ισχύουν οι εξισώσεις: Απάντη ση L L Έχουμε: 2χ 2 - 4χ + 5 = 2χ 2 - 4χ + 2 + 3 = t = - και t - 1 = - . 50 40 = 2(χ2 - 2χ + 1) + 3 = 2(χ - 1) 2 + 3 > 0 , για κάθε χ ε JR . Έτσι έχουμε να επιλύσουμε την εξίσωση: Δίνεται η παράσταση * : 3. � _:_ 1 = � � 5L - 200 = 4L � L = 200km . φ( χ) = αχ 2 + βχ + γ I α '* Ο 50 40 2.

ένα δοχείο υπάρχει υγρό μείγμα 260 ml περιεκτικότητας 25% μιας συγκεκριμένης ουσίας. Πόσα ml της ουσίας αυτής πρέπει να προ­ σθέσουμε στο μείγμα για να πάρουμε μείγ­ μα περιεκτικότητας 35% της 01;1σίας αυτής;

ί)

Σ'

Α πά ντη ση

Έστω ότι προσθέτουμε χ

της ουσίας. Τότε το μείγμα θα περιέχει: (260 + χ) · 35 της ουσίας. 100 Απ ' την άλλη στο νέο μείγμα περιέχονται τα 25 260 . της ουσίας συν τα χ ml, που προσθέσαμε. 100 Συγκροτούμε έτσι την εξίσωση: 35 25 (260 + χ) · - = χ + 260 · - η, 100 100 ml

7(260 + χ) = 20χ + 5 . 260 � � 13χ = 2 · 260 � χ = 40ml

*

Να αποδειχθεί η ταυτότητα: z z φ( x ) = x + � + 4αγ - β 4α 2α ii) Να αποδειχθεί ότι:

J

(

Αργότερα την παράσταση αυτή θα τη πούμε τριώνυμο 2ου βαθμού.

2 4αγ=-----=-β αν α > Ο, τότε φ( χ) 2:: 4α 4α γ - β2 • αν α < Ο, τότε φ( χ) $ _-=----=---4α (το « = » ισχύει μόνον όταν: χ = -β ) 2α •

Α πά ντη ση

Για το (i) : Εκτελούμε τις πράξεις στο δεύτερο μέ­ λος και φτάνουμε στο πρώτο. Για το (ii): Στην περίπτωση α > Ο η παράσταση φ(χ) έχει ελάχιστη τιμή: 4α - β 2 Είναι φ - 1._ = γ = min {φ(x) / x ε JR } . 4α 2α

( )

ΕΥΚΛΕΙΔΗΣ Β ' λη ' τ.l/15


Μαθηματικά για την Α ' Λυκείου

Όμοια στην περίπτωση α < Ο ισχύει: φ

(;�) = maχ {φ(χ) / χ ε �} .

3.

Αν α < β < Ύ και κ, λ, μ θετικοί αριθμοί, να ' , α < κα + λβ + μγ < Ύ αποδ ειχθει, οτι: κ+λ+μ •

Απάντηση 4. Να αποδειχθεί ότι: « 2( α 2 + β 2 ) � (α + β) 2 , Ύια οποιουσδήποτε πραΎματικούς αριθμούς Η αποδεικτέα είναι ισοδύναμη με τις ανισότητες: �+�+�<�+�+�<η+�+�� α και β». �

;...>._

Απάντηση

Η αποδεικτέα γίνεται: 2α2 + 2β 2 - α2 - β 2 - 2αβ � Ο ή (α - β) 2 � Ο «

»

και αληθεύει πάντοτε (το = ισχύει μόνον όταν: α = β). 5.

και εwαι προφανώς

αληθείς. 4.

Αν Ύια τους θετικούς αριθμούς α, β ισχύεt: ' 5α + 4β = 20 , να αποδειχθεί ότι:

( ) ( j)

(i) αβ ::; 5 και (ίί) : 1 + � 1 + � 9. Να αποδειχθεί ότι: « (α + β + Ύ) 2 � 3(αβ + βγ + Ύα) , Απάντη ση -yια οποιουσδήποτε πραΎματικούς αριθμούς Για το (i): Θεωρούμε την ταυτότητα: (5α + 4β) 2 = (5α - 4β) 2 + 8αβ . α, β, Ύ·

Οπότε παίρνουμε: (20) 2 � 80αβ .

Απάντηση

Η αποδεικτέα γίνεται: α 2 + β 2 + γ 2 + 2αβ + 2βγ + 2γα � 3(αβ + βγ + γα) ή α z + β z + γ z - αβ - βγ - γα � Ο ή 2α2 + 2β 2 + 2γ 2 - 2αβ - 2βγ - 2γα � Ο ή (α - β) 2 + (β - γ) 2 + (γ - α) 2 � Ο και αληθεύει πάντοτε (το ισχύει μόνον όταν: α = β = γ). « = »

11.

Ανισότητες υπό περιορισμό.

1.

Αν α+β > Ο, να αποδειχθεί ότι: α 3 + β 3 � β + αβ z .

(το

«

4 = )) ισχύει μόνον όταν: α = -β δηλαδή ' β

β = � και α = 2). Συνεπώς 5 � αβ . 2 Για το (ii): η αποδεικτέα γίνεται: 4 5 20 40 1 + - + - + - � 9 η, - � 8 η' 5 � α β ' α β αβ αβ που έχουμε ήδη αποδείξει. 5.

Απάντηση

Η αποδεικτέα γίνεται: α3 + β 3 - α 2 β - αβ 2 � Ο ή (α + β)( α - β) 2 � Ο και είναι προφανώς αληθής. (το = )) ισχύει μόνον όταν: α = β). «

2.

{λ(κ(αα -- β)γ) ++ μ(λ(βα -- γ)γ) << ΟΟ

Αν α > 2, β > 3, να αποδειχθεί ότι: 2αβ > 3α + 2β . Απάντηση

Η αποδεικτέα γίνεται: 2αβ - 3α - 2β > Ο ή

α(β - 3) + β( α - 2) > Ο και είναι προφανώς αληθής.

Αφού αποδειχθεί η ταυτότητα: (α z + β z )(β z + δ z ) = (αΎ + βδ) z + (αδ - βγ )� ' να αποδειχθεί ότι: «αν 3α - β = 4 τότε: α z + β z � -8 . 5 -2 ) , μονον , οταν: , α = -6 , β = (το « » ισχυει 5 5 =

·.

Α πάντη ση

Για το πρώτο: το πρώτο μέλος γίνεται: αzγz + β z + γz + αzδ z + β zδ z . Το δεύτερο μέλος γίνεται: α 2 γ 2 + 2αβγδ + β 2 δ 2 + α2 δ 2 - 2αδβγ + β 2 γ 2 και είναι προφανώς ίσα.

ΕΥΚΛΕΙΔΗΣ Β' λη ' τ.l/16


Μαθηματικά για την Α ' Λυκείου

Για το δεύτερο: Απ' το πρώτο μέρος έχουμε την ταυτότητα: [ 32 + ( -Ι) 2 J <α2 + β 2 ) = (3α - β) 2 + (3β + α) 2 ή ΙΟ(α 2 + β 2 ) 2 Ι6 . Συνεπώς: α 2 + β 2 2 Ι6 = � . (το « = » ισχ6ει μόνον 10 5 όταν: 3β + α = Ο · δηλαδή β = -2 ,α = � ). 5 5 Αν για τους θετικούς αριθμούς α, β ισχύει: 6. α + β = 3 , να αποδειχθεί ότι:

2 1)

2 1)

( + +( +β α

α

β

·

2 169 • 18

Απά ντ η ση

Η αποδεικτέα γίνεται Ι 69 α2 + -Ι + 2 + β + -Ι + 2 2 -

(

α2 (α2 + β z ) Ι +

2

β2

--f..τ ) 2 97 . 8 αβ

Ι8

και παρατηρούμε ότι: «Οι κ, λ, είναι προφανώς θε­ τικοί αριθμοί)). Με πρόσθεση κατά μέλη των παραπάνω σχέσεων παίρνουμε 4(α + β + γ) - (α + β + γ) = κ + λ + μ ή α+β+γ = κ+λ+μ ( 2) 3 Κι επειδή: 2( α + β + γ) - 3α = κ συμπεραίνουμε ότι: 3α = 3. (κ + λ + μ) - κ ή α = 2(λ + μ) - κ . 3 9 Παρόμοια εργαζόμαστε και βρίσκουμε: β = 2(μ + κ) - λ , γ = 2(κ + λ) - μ . 9 9 Έτσι το πρώτο μέλος της αποδεικτέας γίνεται: 2(λ-+ μ)-'---'- + 2(μ + κ) - λ + 2(κ + λ) - μ = ---'9λ 9κ 9μ

η,

[( Η ;)] - J · i = 3. [( � + � ) + ( μ + �) + ( μ + �)] - .!. ; 3. . 6 _ .!. 9 κ λ κ μ λ μ 3 9 3 �

Ι Κι επειδή 2(α 2 + β 2 ) 2 (α + β) 2 , συμπεραίνουμε ότι: α2 + β 2 2 2 . Και συνεπώς για να αποδείξουμε 2 την (Ι) είναι αρκετό να αποδείξουμε ότι: 97 Ι Ι8 = 97 � -Ι- >- Ι6 � Ι + -2 -2 >- Ji αβ 8Ι α2 β 2 8Ι

� � + � � + �Η: +

=Ι (το « = )) ισχ6ει μόνον όταν: κ = λ = μ δηλαδή ό­ ταν α = β = γ. Με άλλα λόγια όταν το τρίγωνο εί­ ναι ισόπλευρο). Αν p,q > O , τότε Ε. + 3. 2 2 . (το « = )) ισχ6ει q Ρ μόνον όταν: p = q). � 9 2 4αβ �(α + β) 2 2 4αβ , που είναι αληθής. (το « » ισχ6ει μόνον όταν: α = β = � ). 2 111. Α νισώσεις Έτσι έχουμε: (α 2 + β 2 ) Ι + --f..τ 2 2 · 97 = 97 . 3χ - 5 - -χ > 2 + 1 , Να επιλυθει, η ανισωση: -1. α β 2 8Ι Ι8 4 3 χ 7. Αν α, β, γ τα μέτρα των πλευρών τριγώνου Α πάντη ση ΑΒΓ, να αποδείξετε ότι: Η ανίσωση μετά την απαλοιφή των παρονομαστών α β γ 2 1 . γίνεται: 3(3x - 5) - 4x > I2(2x + l) ή Ι 9 χ < -27 ή + + 2(β + γ) - α 2(γ + α) - β 2(α + β) - γ 27 . χ < -Ι9 ( την έχει προτείνει ο συνάδελφος Γιώργος Α πο­ Τελικά το σύνολο λύσεων της ανίσωσης είναι το στολόπουλος απ' το Μεσολογγι). ανοικτό δι�στημα S = Απάντη ση Θέτουμε: 4χ + 3 :::; 2 • , 2(β+γ) - α = κ,2(γ+α) - β = λ, 2(α+β) - γ = μ (Ι) 2. Να επιλυθει, η ανισωση: χ-1 *

υποθ

=

( )

(

-οο, -

�;) .

--

ΕΥΚΛΕΙΔΗΣ Β' λη ' τ.l/17


Μαθηματικά για την Α ' Λυκείου

Α π άντηση

Κατ ' αρχάς πρέπει: χ * 1 (1) Η δοθεί�α ανίσωση γίνεται: (I) 2Χ + 5 4Χ + 3 - 2 � 0 � -- � 0� χ-1 χ -1 + 5)(χ 1) � (2χ ο [Θυμίζουμε ότι: � < Ο � αβ < Ο ] β Θέτουμε φ( χ) = (2χ + 5)(χ - 1) και συγκροτούμε τον παρακάτω πίνακα: [στην τρίτη γραμμή του πί­ νακα εφαρμόζουμε τον κανόνα των προσήμων]. --

Γ !

χ

Ι 2x+s -Ι I I

I

-α:;

5

-

2

ο

I

χ

φ(χ )

+

ο

I

-,

1

+ ωl

±____{ i - (�� - =1 I + +

Συνεπώς το σύνολο λύσεων της ανίσωσης είναι το διάστημα: S = 3.

[-%,ι).

S=

[ λ� 1 , +οο}

αν λ < -2 ή λ > 1

�.

αν λ = 2

(-οο, λ� 1 }

αν - 2 < λ<1

0,

αν λ = 1

αν λ < -2 ή λ > 1 αν λ = -2 αν -2 < λ < 1 αν λ = 1 Με το σύμβολο: 0 παριστάνεται το σύνολο που δεν έχει στοιχεία. Το λέμε κενό σύνολο. 4.

Να επιλυθεί η aνίσωση:

χ �1 2χ - λ

__

Απάντη ση

Η δοθείσα ανίσωση μετά τις πράξεις γίνεται: (λ2 + λ - 2)χ � λ2 - 2λ ή (λ - 1)(λ + 2)χ � λ(λ + 2) (1) [αφού: λ2 + λ - 2 = λ2 + λ - 1 - 1 = λ2 -1 + λ - 1 = = (λ - l)(λ + 2) ] Μελετάμε κατ ' αρχάς το πρόσημο του συντελεστή του χ. Γι ' αυτό συγκροτούμε τον πίνακα:

Γ�

Ι�-λλ:2-�-1�ι1 -ΟΟ--�---+��!---------1��!ι +

(1)

--

Α π ά ντηση

1

λ ' θ ετουμε αρχας τον περιορισμο:' χ * - . ' 2 Έτσι η ανίσωση γίνεται: -χ + λ � ο � (2χ - λ)(χ - λ) � ο 2χ - λ Για λ = Ο, χ * Ο η (1) είναι αδύνατη. κατ '

Να επιλυθεί η aνίσωση: λ2 (χ - 1) + λ( χ - 2) - 2χ � ο (χ: άγνωστος, λ: παράμετρος).

Όπου: Γ = (λ - l)(λ + 2)

Για -2 < λ < 1 η ( 1 ) γίνεται: χ � � (γιατί;) λ-1 Για λ = -2 η ( 1 ) γίνεται: Ο · χ � Ο και αληθεύει πάντοτε. Για λ = 1 η ( 1 ) γίνεται: Ο · χ � 3 και είναι αδύνατη. Τελικά το σύνολο λύσεων της ανίσωσης είναι:

---- ��

1

:__j +

Για λ < -2 ή λ > 1 η ( 1 ) γίνεται: χ � � (γιατί;) λ-1

Για λ > Ο ισχύει: !:_ < λ και συνεπώς κατασκευάζε2 ται ο πίνακας:

Γ-:2χ-λ

I

L

χ-λ Γ

-'Χ)

--

-

-

+

�-

ο

I

;::;

__]

----

-

-

+

λ

-

-

-::

I

+

ο

+

ο

+

1

Όπου Γ = (2χ - λ)(χ - λ) . Άρα το σύνολο αληθείας της ανίσωσης είναι το διάστημα:

(� , λJ.

Για λ < Ο ισχύει: !:_ > λ και συνεπώς το σύνολο 2 αληθείας της ανίσωσης είναι το διάστημα:

ΕΥΚΛΕΙΔΗΣ Β ' λη ' τ.l/18


Μαθηματικά για την Α ' Λυκείου

[ Τ) . Τελικά για το σύνολο αληθείας της

--------

Α πά ντ η ση

S = λ,

ανίσωσης έχουμε:

S=

IV. 1.

(� . λ] . 0,

[λ. �) .

αν λ > Ο αν λ = Ο αν λ < Ο

Έστω ότι έχει εγγραφεί το ορθογώνιο κι ας είναι χ, y τα μήκη δύο διαδοχικών πλευρών. Τότε το εμβαδόν του Ε = xy. Προβλ1) ματα Καθώς γνωρίζουμε ιχύει: Σε μια σχολική εκδρομή ο υπεύθυνος ενός x z + y z � 2xy (1)

γραφείου ταξιδίων έκανε την εξής προσφο­ ρά: «πρέπει να πάρουν μέρος τουλάχιστον 200 παιδιά. Οπότε κάθε παιδί θα πληρώσει 60 ε. Αν όμως συμμετάσχουν περισσότερα, τότε για κάθε επιπλέον μαθητή θα γίνει έκ­ πτωση 0,25 ε σ ' όλους τους εκδρομείς. Πόσοι επιπλέον μαθητές πρέπει να πάρουν μέρος για να μεγιστοποιηθεί το ποσό, που θα εισπράξει το γραφείο; Ποια είναι στην περίπτωση αυτή η είσπραξη; Απάντηση

Ας είναι χ το πλήθος των επιπλέον μαθητών. Τότε θα πάρουν μέρος 200+χ παιδιά. Κάθε παιδί θα πληρώσει: 60-0,25χ. Και συνεπώς το γραφείο θα εισπράξει: (200 + χ)(60 - 0, 25χ) = -0, 25χ 2 + 10χ + 1 2.000 €. Η παράσταση: φ( χ) = -0, 25χ 2 + 10χ + 12.000 παίρνει τη μέγιστη τιμή, κατά το 3° παράδειγμα των μονίμων ανισοτήτων αυτής της εργασίας, για την τιμή: -10 5 = = 20 . Χ= 2( -0, 25) Ο, 25

--

Τότε το γραφείο θα εισπράξει: φ(20) = 12. 100 2.

�""\

�------�� Δ

€.

Σ ' ένα δοσμένο κύκλο θα εγγραφεί ορθογώνιο μεγίστου εμβαδού.

Το «ίσο» ισχύει μόνον όταν: χ = y. Προφανώς η ΒΔ είναι διάμετρος. Κι αν είναι r η ακτίνα του κύκλου, τότε ΒΔ = 2r . Συνεπώς η ( 1) γίνεται: (2) Άρα η μέγιστη τιμή του εμβαδού είναι 2r2 και την παίρνει μόνον όταν: χ = y· δηλαδή όταν το ορθο­ γώνιο είναι τετράγωνο. Με άλλα λόγια όταν οι διαγώνιες ΑΓ και ΒΔ είναι κάθετες. Η κατασκευή είναι mα πολύ εύκολη. Παίρνουμε μια διάμετρο ΒΔ και την κάθετη σ ' αυτή διάμετρο ΑΓ. 3.

Αφού αποδειχθεί η ταυτότητα: (α z + β z + γ z )(x z + y z + z z ) _ (αχ + βy + γz) 2 = (βχ - αy) z + (γy - βχ) z + (γχ - αz) 2 =

Α /\\

/

\

�" ,// /y

p Y\ \

Β/ ------

--- -- - .

χ

· · -------�

--

\

\ \

\

---- ...\ - Γ

να βρεθεί στο εσωτερικό τρίγωνου ΑΒΓ σημείο Ρ έτσι ώστε «αν χ, y, z είναι οι απο­ στάσεις του Ρ απ ' τις πλευρές του τριγώνου και χ + y + z = c όπου c > Ο σταθερός, το άθροισμα χ 2 + y 2 + z 2 να είναι το μικρότερο δυνατό».

ΕΥΚΛΕΙΔΗΣ Β' λη ' τ.l/19


Μαθηματικά yια την Α ' Λυκείου

( το έχει προτείνει ο συνάδελφος Θανάσης Κυρια­ κόπουλος, μέλος της Συντακτικής Επιτροπής του περιοδικού).

Το « = » ισχύει μόνον όταν: χ = y = z. Άρα το ζητούμενο σημείο είναι το σημείο που συ­ Απάντηση ναντιούνται οι διχοτόμοι των γωνιών του τριγώ­ Για το πρώτο: νου. Εκτελούμε τις πράξεις και στα δύο μέλη της απο­ Θυμίζουμε ότι: «η διχοτόμος μιας γωνίας αποτε­ δεικτέας και οι παραστάσεις που προκύπτουν είναι λείται από όλα τα σημεία του Επιπέδου, που ισα­ ίδιες. πέχουν απ ' τις πλευρές της γωνίας». Για το δεύτερο: Σχόλιο: Το σημείο αυτό λέγεται έγκεντρο (κέντρο Απ' την ταυτότητα για α = β = γ = 1 παίρνουμε: του εγγεγραμμένου κύκλου). οπότε χ = y = c = ρ 3(x z + y z + z z ) - (χ + Υ + z) z = (ρ η ακτίνα του εγγεγραμμένου κύκλου. Συνεπώς = (χ -:- y) z + (y z) z + (z x) z c = 3ρ . ή x z +y z +z z = c 2 + (x-y) z +(y z) z + (z x) z J . _

� �[

_

_

_

� ΓΕΩΜ Ε ΤΡΙΑ για τη ν Α ' τάξη το υ Λ υκ ε ίο υ Ισό τη τ α τρ ιγώ ν ω ν

του Κώστα Βακαλόπουλου Σε μια μεγάλη ομάδα ασκήσεων στη Γεωμετρία χρησιμοποιούμε τα θεωρήματα (ΚΡΙΤΗΡΙΑ) ισότητας των τριγώνων. Τα θεωρήματα αυτά γνωστά και από το Γυμνάσιο κωδικοποιούνται στις φράσεις: • Π-Γ-Π (πλευρά - γωνία - πλευρά), • Γ-Π-Γ (Γωνία - πλευρά - γωνία) και • Π-Π-Π (πλευρά - πλευρά - πλευρά). Για την περίπτωση των ορθογωνίων τριγώνων ισχύει ότι είναι ίσα όταν έχουν: • Δύο ομόλογες πλευρές ίσες μια προς μια • Μια πλευρά και την προσκείμενη σ' αυτή οξεία γωνία αντίστοιχα ίση μια προς μία Ο τρόπος αντιμετώπισης των ασκήσεων συνίσταται: ]0v) Στον προσδιορισμό των τριγώνων που θα συγκρίνουμε. (Εδώ πολλές φορές φέρνουμε κατάλληλες ευθείες ώστε να σχηματιστούν τρίγωνα που να περιέχουν τα ζητούμενα στοιχεία). 2°v) Στην τεκμηρίωση των ισοτήτων που απαιτούνται για την στοιχειοθέτηση των προϋποθέσεων του κριτηρίου που χρησιμοποιούμε. (Στο βήμα αυτό, προσέξτε, ότι πολλές φορές χρειάζεται να συγκρίνουμε άλλα τρίγωνα προκειμένου να συμπεράνουμε ισότητες που χρειαζόμαστε στην αρχική μας σύγκριση. Συνήθως η σύγκριση αυτή αν και η ανάγκη της δημιουργείται στη πορεία της λύσης, προηγείται στη λύση μας). ΕΥΚΛΕΙΔΗΣ Β ' λη ' τ.l/20


Μαθηματικά για την Α ' Λυκείου

3°v) Στα συμπεράσματα που προκύπτουν από την ισότητα των τριγώνων που αποδείξαμε μεταξύ

των

ο­

πο ίων θα υπάρχει το ζητούμενο της άσκησης. Στη συλλογή των ασκήσεων που ακολουθεί σας δίνεται η ευκα ιρία από μεν τις λυμένες να δείτε τη διαδι­ κασία που αναφέραμε από δε τις άλυτες να την εφαρμόσετε μόνοι σας. 1.

Έστω γωνίες χΟψ και χΌ'ψ με κοινή διχοτόμο Οδ. Στις πλευρές Οχ και Οψ παίρνουμε σημεία Α και Β αντίστοιχα ώστε ΟΑ = ΟΒ. Επίσης στις πλευρές Οχ' και Οψ' σημεία Α' και Β' ώστε ΟΑ' = ΟΒ'. Δείξτε ότι: ΑΑ ' = ΒΒ '. (Για αρχή μια εύκολη άσκηση ! )

Υπ ό δ ε ιξη

Φέρτε την ΑΜ (. . . η κατάλληλη ευθεία που ανα­ φέραμε στην εισαγωγή) ώστε να δημιουργηθούν τρίγωνα που να περιέχουν τα τμήματα που θέλου­ με να αποδείξουμε ίσα. ψ

� Ε

Δ

χ

Λύση

Φτιάξτε το σχήμα.

ψ

��,,ο4=��----δ ��----Α' Α�-

χ' -.....___ χ

Τα τρίγωνα που θα συγκρίνουμε είναι τα ΟΑΑ' και ΟΒΒ '. Αυτά έχουν: 1) ΟΑ = ΟΒ (Από υπόθεση) 2) ΟΑ' = ΟΒ ' (Από υπόθεση) 3) ΑΟΑ' Β ΟΒ' (Ως διαφορά των ίσων γωνιών χόδ δΟφ και χ'Οδ δΟφ' ) Άρα τα τρίγωνα είναι ίσα (Π Γ Π) Επομένως ΑΑ' = ΒΒ ' =

=

Αξιοποιείστε τις ιδιότητες των ισοσκελών τριγώνων και εξηγείστε γιατί ισχύουν οι προϋποθέσεις ενός κριτηρίου ισότητας τριγώνων το οποίο και εφαρμόστε το. 3.

Δίνεται ισοσκελές τρίγωνο ΑΒΓ (ΑΒ = ΑΓ). Οι διχοτόμοι των εξωτερικών γωνιών Β και Γ τέμνονται στο Μ. Αν Κ, Λ είναι τα μέσα των ΑΒ και ΑΓ δείξτε ότι: ΜΚ = ΜΛ. Λ

=

-

-

.

Επισή μανση :

Σε ίσα τρ ίγωνα απέναντι από ίσες πλευρές υπάρ­ χουν ίσες γωνίες και αντίστροφα απέναντι από ίσες γωνίες υπάρχουν ίσες πλευρές. 2.

Δίνεται ισοσκελές τρίγωνο ΑΒΓ με ΑΒ = ΑΓ. Φέρνουμε Αχ .l ΑΓ και Αψ .l ΑΒ (έξω από το τρίγωνο). Παίρνουμε σημείο Δ στην Αχ και Ε στην Αψ ώστε ΑΕ = ΑΔ. Να δειχθεί ότι τα Δ και Ε απέχουν εξίσου από το μέσο Μ της ΒΓ.

Υπ ό δ ε ιξη

Εδώ τα πράγματα είναι εύκολα! Δεν χρειάζεται να φέρουμε εμείς ευθεία για να σχηματίσουμε τρίγωνα. Τα τρίγωνα είναι σχηματι­ σμένα! Είναι τα ΚΒΜ και ΛΓΜ. Αιτιολογείστε

ΕΥΚΛΕΙΔΗΣ Β' λη ' τ.l/21


Μαθηματικά για την Α ' Λυκείου =

έτσι ώστε: ΑΔ ΑΕ. Έστω Μ τυχαίο ση­ μείο της διχοτόμου της γωνίας Α. Αν Ζ, Η τα σημεία στα οποία οι ΜΔ και ΜΕ τέ­ μνουν την ΒΓ (ή τις προεκτάσεις της), να δείξετε ότι: ΒΖ ΓΗ.

απλά τις προϋποθέσεις του κριτηρίου που θα χρη­ σιμοποιήσετε. 4.

=

Δίνεται ισοσκελές τρίγωνο ΑΒΓ με ΑΒ ΑΓ. Πάνω στη πλευρά ΒΓ παίρνουμε τα σημεία Μ και Ν έτσι ώστε ΒΜ ΓΝ. Αν Δ, Υπ ό δειξη Ε οι προβολές των Β και Γ στις ΑΜ και ΑΝ αντίστοιχα δείξτε ότι το τρίγωνο ΑΔΕ είναι Θα συγκρίνουμε τα τρίγωνα: ΒΔΖ και ΓΕΗ ( . ) Α ισοσκελές. =

=

. .

Α

Γ

Μ

Λύ ση

Θα δείξουμε ότι: ΑΔ = ΑΕ και συγκεκριμένα θα δείξουμε ότι ΑΜ = ΑΝ ( 1 ) και ΔΜ = ΕΝ (2). Συγκρίνουμε τά τρίγωνα ΑΒΜ και ΑΓΝ, αυτά έ­ χουν: α) ΑΒ ΑΓ (αφού το ΑΒΓ είναι ισοσκελές) β) ΒΜ = ΓΝ (από υπόθεση) γ) Β = f (αφού το ΑΒΓ είναι ισοσκελές) Άρα τα τρίγωνα ΑΒΜ και ΑΓΝ είναι ίσα (Π-Γ-Π) οπότε: ΑΜ = Αν (1) και ΑΜΒ = ΑΝΓ . =

Συγκρίνουμε στη συνέχεια τα ορθογώνια τρίγωνα ΒΔΜ και ΓΕΝ, αυτά έχουν: α) ΒΜ = ΓΝ (από υπόθεση) β) ΒΜΔ = rNE (ως παραπληρώματα των ίσων γωνιών ΑΜΒ και ΑΝΓ , βλέπε προηγούμενη άσκηση) Άρα τα τρίγωνα ΒΔΜ και ΓΕΝ είναι ίσα οπόtε: ΜΔ = ΝΕ (2) Προσθέτοντας κατά μέλη τις σχέσεις ( 1 ) και (2) έχουμε: ΑΜ + ΜΔ = ΑΝ + ΝΕ <:::::> ΑΔ = ΑΕ

Προσέξτε όμως! Για τις γωνίες ΒΔΖ και ΗΕΓ . Θα συγκρίνουμε πρώτα τα τρίγωνα ΑΔΜ και ΑΕΜ ώστε να προκύψουν ίσα οπότε θα έχουν τις γωνίες ΑΔΜ και ΑΕΜ ίσες ώστε οι Β ΔΖ και ΗΕΓ να είναι ίσες ως παραπληρωματικές ίσων γωνιών. 6.

Έστω τεθλασμένη γραμμή ΑΒΓΔ αποτε­ λούμενη από τρία ευθύγραμμα τμήματα ΑΒ, ΒΓ, ΓΔ με ΑΒ ΓΔ και Β = Γ . Στο μέσο Ε της ΒΓ φέρνουμε κάθετη προς αυτήν και έστω Ζ τυχαίο σημείο της. Να δείξετε ότι: ΖΑ ΖΔ. =

Λ

=

Λί>ση

Φέρνουμε τις ΒΖ και ΓΖ Επειδή Ζ σημείο της μεσοκαθέτου της ΒΓ θα ισχύ­ ει: ΒΓ = ΓΖ, δηλαδή το τρίγωνο ΒΖΓ είναι ισο­ σκελές.

1

5.

Λ

ι

z Στις ίσες πλευρές ΑΒ και ΑΓ ισοσκελούς τριγώνου ΑΒΓ παίρνουμε σημεία Δ και Ε Άρα: ΖΒΓ = zfB . Όμως Β = f

ΕΥΚΛΕΙΔΗΣ Β' λη ' τ.l/22


Μαθηματικά για την Α ' Λυκείου

Λύση

Άρα: Β - ΖΒΓ Γ - zrB ς:} ΖΒΑ zrΔ =

=

Συγκρίνουμε τώρα τα τρίγωνα: ΑΒΖ και ΔΓΖ, αυ­ τά έχουν: α) ΑΒ = ΓΔ (από υπόθεση) β) ΒΖ = ΓΖ (αφού Ζ σημείο της μεσοκαθέτου) γ) ΖΒΑ zrA (απεδείχθη) Άρα τα τρίγωνα ΑΒΖ και ΔΓΖ είναι ίσα (Π-Γ-Π), οπότε ΖΑ = ΖΔ.

χ /

//

- � -- ----- . I

Α

=

Κυρτή πολυγωνική γραμμή χΑΒψ έχει τις γωνίες Α και Β ίσες. Από το μέσο Μ της ΑΒ φέρνουμε κάθετη στην Αχ που τέμνει την Βψ στο Ζ και άλλη κάθετο στην Βψ Έστω Δ το σημείο στο οποίο η κάθετος από το Ο που τέμνει την Αχ στο Δ. Δείξτε ότι: ΜΖ = τέμνει την ΒΑ και Μ το σημείο στο οποίο αυτή τέμνει την Β 'Α'. ΜΔ. ..

..

Υπόδειξη χ

κ

Για να σχηματίσουμε τρίγωνα που να περιέχουν τις πλευρές ΜΑ' και ΜΒ ' που θέλουμε ν ' αποδεί­ ξουμε ίσες φέρνουμε από τα Β ' και Α' κάθετες στην κάθετο από το Ο στην ΒΑ και έστω Κ και Λ που αυτές την τέμνουν. Θα συγκρίνουμε τελικά τα ορθογώνια τρίγωνα ΜΛΒ ' και ΚΜΑ'. Πριν όμως θα «μαζέψουμε» από άλλες συγκρίσεις τις ισότητες που χρειαζόμαστε. Συγκρίνουμε τα ορθογώνια τρίγωνα ΟΚΑ' και ΟΔΑ, αυτά έχουν: α) ΟΑ = ΟΑ' (από υπόθεση) β) ΟΑ 'Κ ΑΟΔ (οξείες γωνίες με πλευρές κάθε­ τες) Άρα τα τρίγωνα ΟΚΑ' και ΟΔΑ είναι ίσα οπότε: ΚΑ' = ΟΔ ( 1 ) =

Έστω Κ και Λ τα σημεία στα οποία οι κάθετες από το Μ τέμνουν τις Αχ και Βψ αντίστοιχα. Να συγκρίνετε πρώτα τα τρίγωνα ΜΚΑ και ΜΛΒ και στη συνέχεια τα ΜΚΔ και ΜΑΖ. 8.

Δίνονται δυο παραπληρωματικές γωνίες χΟψ και χΌψ' τέτοιες ώστε: Οχ ..l Οχ' και Οψ ..l Οψ' . Πάνω στις Οχ και Οχ' παίρνουμε αντίστοιχα σημεία Α και Α' ώστε ΟΑ = ΟΑ' και πάνω στις Οψ και Οψ' σημεία Β και Β' ώστε ΟΒ = ΟΒΌ Δείξτε ότι η κάθετος από το Ο στην ΒΑ διχοτομεί την ΒΆΌ

Συγκρίνουμε τα ορθογώνια τρίγωνα ΟΛΒ ' και ΟΒΔ, αυτά έχουν: α) ΟΒ = ΟΒ ' β) ΛΒ 'Ο Β ΟΔ (οξείες γωνίες με πλευρές κάθε­ τες) Άρα τα τρίγωνα ΟΛΒ' και ΟΒΔ είναι ίσα οπότε: ΛΒ' = ΟΒ (2) Από (1) και (2) προκύπτει ότι ΚΑ' = ΛΒ Ό Συγκρίνουμε τέλος τα τρίγωνα ΜΛΒ ' και ΜΚΑ', αυτά έχουν:

ΕΥΚΛΕΙΔΗΣ Β' λη ' τ.l/23

=


Μαθηματικά για την Α ' Λυκείου

α) ΛΒ ' = ΚΑ' (απεδείχθη) β) ΑΒ 'Μ = κλ 'Μ (ως συμπληρωματικές των ίσων γωνιών ΑΜΒ' και κΜΑ' που είναι ίσες ως κατακορυφήν) Σ η μείωση :

Για την τελευτα ία ισότητα χρησιμοποιήσαμε την γνωστή πρόταση από το Γυμνάσιο ότι οι οξείες γω­ νίες ενός ορθογωνίου τριγώνου είναι συμπληρωμα­ τικές. Την πρόταση αυτή θα την αποδείξουμε στο επόμενο κεφάλαιο. Άρα τα τρίγωνα ΜΛΒ και ΜΚΑ είναι ίσα, οπότε ΜΑ' = ΜΒ Ό ·

9.

·

Δείξτε ότι, αν δύο ορθογώνια τρίγωνα έ­ χουν μια από τις κάθετες πλευρές ίσες και τις περιμέτρους ίσες τότε είναι ίσα. Λύ ση

Έστω δύο ορθογώνια τρίγωνα ΑΒΓ και Α Ή 'Γ με β = β' και α + β + γ = α' + β' + γ'. Τότε θα ισχύει: α + γ = α' + γ'. Προεκτείνουμε τις πλευρές ΑΒ και Α Ή ' και στις προεκτάσεις τους παίρνουμε τμήμα­ τα ΒΕ = ΒΓ και Β Έ ' = Β 'Γ.

Συγκρίνουμε τα τρίγωνα ΓΒΕ και ΓΒ Έ', αυτά έχουν: α) ΓΕ = ΓΈ' (απεδείχθη) β) Ε = Ε ' (απεδείχθη) γ) ΒfΈ = B'f'E' (αφού τα τρίγωνα ΒΓΕ και Β Τ Έ ' είναι ισοσκελή και BfE = Ε και B'r'E' = E ) Άρα τα τρίγωνα ΓΒΕ και ΓΒ Έ' είναι ίσα οπότε ΒΕ = Β Έ '. Επομένως ΒΓ = Β 'Γ". Συγκρίνουμε τέλος τα αρχικά ορθογώνια τρίγωνα ΑΒΓ και Α Ή 'Γ, αυτά έχουν: α) ΑΓ = Α'Γ (από υπόθεση) β) ΒΓ = Β 'Γ (απεδείχθη) Άρα τα τρίγωνα ΑΒΓ και Α Ή 'Γ είναι ίσα. 1 0.

Δείξτε ότι, αν δύο τρίγωνα έχουν δυο πλευ­ ρές αν μία ίσες και τις μεταξύ των ίσων πλευρών περιεχόμενες διαμέσους ίσες, τότε είναι ίσα. Λ

/� j '�

�� Μ Α

\

1�Γ /

Γ

I

\\ I/ ν

γ

Δ

Υπ ό δ ε ιξη

Α

γ

Άρα ΑΕ = ΑΈ' Συγκρίνουμε τα ορθογώνια τρίγωνα ΑΕΓ = Α Έ 'Γ, αυτά έχουν: α) ΑΓ = Α'Γ (από υπόθεση) β) ΑΕ = ΑΈ' (απεδείχθη) Άρα τα τρίγωνα ΑΕΓ και Α Έ 'Γ είναι ίσα, οπότε: Ε = Ε ' και ΓΕ = ΓΈΌ

Έστω δύο τρίγωνα ΑΒΓ και ΑΉΤ' με β = β', γ = γ' και � = �'. Προεκτείνουμε τις διαμέσους ΑΜ(�) και Α'Μ'(�) κατά ίσα τμήματα: ΜΔ = ΑΜ και Μ'Δ'= Α'ΜΌ Με διαδοχικές συγκρίσεις τριγώνων αποδείξτε ότι ΜΓ = Μ'Γ α α' δηλαδή - = - <=> α = α' 2

2

οποτε τα τρίγωνα ΑΒΓ και Α Ή 'Γ θα είναι ίσα αφού θα έχουν και τις τρεις πλευρές τους ίσες μία προς μία.

ΕΥΚΛΕΙΔΗΣ Β' λη ' τ.l/24


Μαθ ηματ ι κά

για την Β

... τάξη

του

Λυκείου

(15) Τρ ι γω νομετρ ία

Νίκος Ταπεινός

Για την νέα σχολική χρονιά, ευχόμαι κάθε επιτυχία και κυρίως απόκτηση γνώσεων από όλους τους μαθη­ τές, που θα τους βοηθήσουν για την επίτευξη των στόχων τους, στην προσπάθεια να κατακτήσουν την ζωή. Οι λυμένες ασκήσεις της τριγωνομετρίας που παραθέτω, ελπίζω να βοηθήσουν στην καλύτερη κατανόη­ ση της ενότητας αυτής.

Επειδή

Λυμένες Α σκή σεις

1.

(1) και

Αν {4 - .J3) ημχ - 2συν2χ + 2 = 2.J3

χ τόξο του 2°υ τεταρτημορίου να υπολογι­ σθεί η τιμή της παράστασης. 2 · εφχ + 3σφχ - 6 · συνχ Α= . 6 · ημχ

<=> ( 4 - ../3) · ημχ-2 · ( ι-ημ2 χ ) + 2 - 2../3 ο <=>

Από τη σχέση < ι )

2ημ 2 χ + ( 4 - ..!3 ) · ημχ-2../3 = ο . Θέτουμε η μχ = y και τότε έχουμε: 2y2 + (4 - ../3) y - 2../3 = 0 με Δ = (4 - ../3) 2 - 4 · 2 · (-2../3) =

και

2·2

<=>

=

=-

{-�)-6{-υ 6 -

.J3 -ι 3

2.

Αν εφ

(�; ) (; ) : ( ) ( ) + χ - εφ

-χ =

2

να υπο-

λογίσετε την τιμή της παράστασης: π 3π Α = εφ 3 10 + χ - εφ 3 - x .

<=>

S

' η μχ = 2 (Δεκτη' ) Υ 2 η' y 2 . Τοτε ή ημχ = -2 < -ι (απορρίπτεται) .J3

=-

ι-

= .J3 · 2 ι - .J3 (ι - ..!3 )../3 .J3 - 3 = = = .J3 3 ../3 2

Α=

= ι6 + ../3 2 + 8../3 = = (4 + ../3) 2 .

=

εφχ = η μχ συνχ

�% �

.J3 ι . = -../3, σφχ = __ = 3 εφχ

Άρα

=

= ι 6 + ../3 2 - 8../3 + ι6../3 =

'

, π έχουμε:

συνχ =-�ι-ημ2 χ = -

=

-( 4 - ../3) ± ( 4 + ..!3 )

(% )

2 (--13) + 3

Λύ ση

Άρα Υ ι . 2 =

�ε

.J3

Λύ ση π

, , 3π π π Ά αρατηρουμε οτι: + χ + --χ . ρα

-

εφ

(�� χ) σ (�- χ) <=> 10

-

+

ΕΥΚΛΕΙΔΗΣ Β ' λη ' τ.l/25

= φ

5

= -

2


Μαθηματικά Β' Λυκείου

Από την ταυτότητα (α - β)3 = α3 - 3α 2 β + 3αβ 2 - β3 έχουμε: α3 - β3 = (α - β)3 + 3αβ(α - β) . Άρα: � (� � + · - εφ � - χ +

Η ) ( )]' +3 · εφ (� - χ }[ εφ ( �� + χ ) - εφ ( � - χ )] = 3 = ( 2J3 ) + 3 · 1 · 2J3 = 8 · 3 · J3 + 2J3 = 27 3 3

= g J3 + 2J3 = 26J3 . 9 3 3. Να αποδείξετε ότι: 4(συν8χ - η μ8χ) = συν2χ(3 + συν 4χ) . Λύ ση

4(συν 8χ - ημ8χ) = = 4(συν4χ - ημ4 χ) · (συν4 χ + ημ4χ) = = 4(συν 2 χ - ημ 2 χ) · (συν 2 χ + ημ 2 χ) . { (συνzχ + ημzχ)z - 2ημzχ · συνzχ J = = 4συν 2χ · 1 · 1 2 - 4ημ2χ · συν 2 χ 2 = 4συν2χ ι - ημ 2χ =

{

(

)

( )

_

_!_

Λύ ση

Από (1) � η μ 2 χ+ J3ημχσυνχ - ημχσυνχ-J3συν 2 χ=0 και επειδή χ =t- κπ + !: � συνχ =t- Ο διαιρούμε με 2 συν 2 χ και έχουμε: ημ 2 χ + J3 ημχσυνχ ημχσυνχ J3 · συν 2 χ ο συν 2 χ συν 2 χ συν 2 χ συν 2 χ � εφ 2 χ + J3 · εφχ - εφχ - J3 = 0 � εφ 2 χ + ( J3 - ι ) εφχ - J3 = 0 Θέτουμε εφχ = y και έχουμε: y 2 + ( J3 - 1 ) · y - J3 = Ο με Δ = ( J3 - 1 ) 2 + 4J3 = ( J3 + 1 ) 2 ·

--

)

= συν2χ · { 4 - 2ημ 2 2χ ) = = συν2χ · [ 4 - (1 - συν4χ)) = = συν2χ · (3 + συν4χ) . 4. Ν α λυθεί η εξίσωση: 2ημ2χ + 2 .J3ημχ + 2συνχ + .J3 = Ο ( 1). Λύ σ η

(1) � 4ημχσυνχ + 2J3 · ημχ + 2συνχ + J3 = Ο 1 J3 + -συνχ J3 = Ο � ημχσυνχ + -ημχ +4 2 2 � ημχ συνχ + + συνχ + �ο

( �) Η �) � ( συνχ + �Η ημχ + υ � ο

J3 = 0 (2) ή ημχ + -1 = ο (3). � συνχ + 2 2 Από (2) � συνχ = - J3 � συνχ = -συν!: 2 6 5π 5π � συνχ = συν- � χ = 2κπ ± - με κ Ε Ζ 6 6 Από (3) � ημχ = � ημχ = -ημ!: 2 6 � ημχ = ημ - � � χ = 2κπ - � με κ Ε Ζ 7 π με κ Ε z . η. χ = 2κπ + 6 π 5. Αν χ =t- κπ + ,κ Ε Ζ να λυθεί η εξίσωση 2 2 η μ x+ .J3 ημχσυνχ=ημχσυνχ+ .J3συν 2 χ ( 1 ).

Άρα

- ( J3 - 1 ) ± ( J3 + 1 ) :=:? Υ = 1 ή Υ = -J3 2 τ οτε, . εφχ = 1 � εφχ = εφ 4π � χ = κπ + 4π y1,2 =

( )

με Κ Ε Ζ ή εφχ = -J3 � εφχ = -εφ � � εφχ = εφ - � � Χ = ΚΠ - -π με Κ Ε Ζ . 3 2χ = .J3 · ημ 2χ 1) 6. Να λυθεί η εξίσωση συν ( 3 3 αν χ Ε (ο, π] .

ΕΥΚΛΕΙΔΗΣ Β ' λη ' τ.l/26


Μαθηματικά Β' Λυκείου

Λύση

2χ :;t: Ο . Επειδή χ Ε (ο, π] έχουμε η μ3 Διαιρούμε τα μέλη της (1) με ημ 2χ και έχουμε: 3 {:; 2χ 2χ π σφ - = ν3 <::::> σφ- = σφ- <::::> 6 3 3 2χ π 3κπ π - = κπ + - <::::> χ = - + - με κ Ε Ζ (2). 2 4 6 3 Αλλά χ Ε (ο, π ] <::::> Ο < χ :ς π και λόγω (2) 3κπ ::ς 3π 3κπ + -π ::ς π <::::> --π < <::::> 0 < 4 2 4 2 4 <::::> --1 < 3κ :ς -3 <::::> --1 < κ :ς -1 2 2 6 2 Επειδή κ Ε Ζ έχουμε κ Ο, οπότε λόγω της (2) χ = -π =

4

.

-ημ2 α - ημ 2 β :::;; Ο <::::> η μz α . ημ2 β + η μ 2 α . η μ2 β _ -2ημα · συνα · ημβ · συνβ - η μ 2 α - η μ 2 β :ς Ο <::::> (1 - συν 2 α)(l - συν2 β) + η μ2 α . η μ 2 β 2ημα · συνα · ημβ · συνβ-(1-συν 2 α)-(1 - συν 2 β) :ς Ο <::::> 1 - συν 2 β - συν 2 α + συν 2 βσυν 2 α + ημ2 α · η μ 2 β 2ημα · συνα · ημβ · συνβ - 1 + συν 2 α - 1 + συν 2 β :ς Ο <::::> συν 2 α · συν 2 β + ημ 2 α · ημ 2 β -2ημα · συνα · ημβ · συνβ :ς 1 <::::> ( συνα · συνβ - η μα · ημβ) 2 :ς 1 <::::> συν2 (α + β) :::;; 1 που προφανώς ισχύει. π 9. Σε τρίγωνο ΑΒΓ, είναι Β = . Να αποδεί4 ξετε ότι: σφ Α+ 1 σφ r+ 1 = 2

(

}(

)

Λύ ση Να αποδείξετε ότι: συν( α + β) · ημβ + ημ(α + γ) · συνγ = Γνωρίζουμε ότι: Α+ Β+ Γ = π και επειδή Β = -Π = συν(β - γ) · η μ(α + β + γ). 4 3π Λύ ση έχουμε Α+ Γ = 4 συν( α + β) · ημβ = συν [ (α + β + γ) - γ ] · ημβ 3π = [ συν(α + β + γ) · συνγ + ημ(α + β + γ) · ημγ ] · ημβ = σφ Α+ Γ = σφ 4 = συν(α + β + γ) · συνγ · ημβ + <::::> σφ Α· σφ Γ - 1 = _ 1 +ημ(α + β + γ) . ημγ . ημβ (1) σφΑ+ σφΓ ημ(α + γ) · συνγ = ημ [ (α + β + γ) - β ] · συνγ = <::::> σφΑ· σφ Γ- 1 = -σφΑ- σφΓ = [ημ(α + β + γ)συνβ - συν(α + β + γ)ημβ ] συνγ = = ημ(α + β + γ) · συνβ · συνγ <::::> σφ Α σφ Γ+ σφ Α+ σφ Γ = 1 -συν( α + β + γ) · ημβ · συνγ (2) Προσθέτουμε τις (1) και (2) κατά μέλη και έχουμε: <::::> σφΑ· σφ Γ+ σφΑ+ σφΓ+ 1 = 2 συν(α + β) · ημβ + ημ(α + γ) · συνγ = <::::> σφ Α· σφ Γ+ 1 + σφ Γ+ 1 = 2 = η μ( α + β + γ )η μγ · ημβ+ημ(α+β+γ)συνβ · συνγ <::::> συν( α + β) · ημβ + ημ(α + γ) · συνγ = <::::> σφ Α+ 1 · σφ Γ+ 1 = 2 . = ημ(α + β + γ) · [ συνβ · συνγ + ημβ · ημγ ] 5π <::::> συν( α + β) · ημβ + ημ(α + γ) · συνγ = 10 . Αν χ :;t: κπ ± - να απο δ ει'ξετε οτι: . 6 = συν(β - γ) · ημ(α + β + γ) σφ χ + 5π · σφ χ - 5π = 1 + συν2χ . 8, Ν α αποδείξετε ότι: 1 - 2συνχ 6 6 1 2ημ 2αημ2β--ημ2α · ημ2β :ς ημ 2α+η μ2 β (1) Λύ ση 2 Λύ ση (1) <::::> 2ημ2 α · η μ2 β - -1 · 2ημα · συνα · 2ημβ · συνβ 2 7.

Λ

Λ

Λ

Λ

Λ

( ) Λ

Λ

Λ

Λ

Λ

Λ

Λ

Λ

Λ

Λ

Λ

Λ

Λ

Λ

Λ

Λ

Λ

Λ

Λ

(

(

(

ΕΥΚΛΕΙΔΗΣ Β ' λη ' τ.l/27

)(

)(

)

) (

)

)


Μαθηματικά Β' Λυκείου

5π - 1 σφχ ο σφ5π + 1 σφχ ο σφ6 έχουμε --6�= ---5π 5π σφ- + σφχ σφ- - σφχ 6 6 σφ 5: = σφ π - � = -σφ � = -νJ . 'Ετσι

(

( )

)

ο Α = -ν'3σφχ - 1 ο -νJ σφχ + 1 = -νJ + σφχ -νJ - σ φχ ν] ο ο σφχ - 1 = σφχ + 1 ο ν] = ν] - σφχ ν'3 + σφχ , = 3σφ 2 χ 2- 1 εχουμε 3 - σφ χ εφ 2 χ = ι ..:.. συν2χ � σφ 2 χ = 1 + συν2χ . 'Ετσι 1 + συν2χ 1 - συν2χ Ι + συν2χ _1 3 Α - 0 1 - συν2χ = 3 + 3συν2 χ - 1 + συν2χ = 3 _ 1 + συν2χ 3 - 3συν2χ - 1 - συν2χ 1 - συν2χ = 2 + 4συν2χ = 1 + 2συν2χ 2 - 4συν2χ 1 - 2συν2χ

(

)

_

-------

11.

Να αποδείξετε ότι:

( ;} ( ι� - ι} ο ( ι - εφ z 3π ) ο ( εφ z 7π - ι ) = 48 ι2 S ι - εφ z

εφ z

Λύση

, Επειδη εφ 2 χ = 1 - συν2χ εχουμε: 1 + συν2χ ,

(ι - εφ2 �}(εφ2 1� - ι} ο ( Ι - εφ 2 3π ) ο ( εφ 2 7π - ι ) = 12 S

[

ο

[

[

1 - συν � Ι - συν � ο ι ο π π l + συν- l + συν4 6 1 - συν 3π 1 - συν Ί π 4 6 -1 = 13π 7π l + συν4 l + συν6

= ι-

J[ J

J J

3π -2 ο συν 7π 2συν-π -2συν-π 2συν 4 6 = 6 ___.:� _,_ = ----π --'4"-π 3π 7 l + συν-4 l + συν- l + συν- l + συν -π 4 6 6 _ __ __

_ __

= =

J2 J2 ν'3 ν'3 -2 ο -2 o 2ο 2 2 2 ο -2- =

1 + J2 l + ν] 1 - J2 1 - ν] 2 2 2 2 J2 J2 ο J2 J2 ι1+ 2 2

0

ν'3 ν'3 1+ 2

0

ν'3 ν'3 12

=

= -2- ο -3- = 4 ° 1 2 = 48 ι-! ι-l 4 2 Να αποδείξετε ότι:

1 2.

εφ z 4χ - 2συν z 4χ + 2ημ z 4χ = ο = 3 συν8χ + συνι6χ ι + συν8χ δ η η

Επει

,

Λύση

,

μ 2 χ = 1 - συν2χ συν 2 χ = 1 + συν2χ 2

2

, και εφ 2 χ = 1 - συν2χ εχουμε: 1 + συν2χ εφ 24χ - 2συν 24χ + 2ημ2 4χ = = 1 - συν8χ (1 + συν8χ) + (1 - συν8χ) = 1 + συν8χ = 1 - συν8χ 2συν8χ = 1 + συν8χ = 1 - συν8χ - 2συν8χ - 2συν 2 8χ 1 + συν8χ = 1 - 3συν8χ - (1 + συν16χ) = 1 + συν8χ = -3συν8χ - συν16χ = 1 + συν8χ = 3συν8χ + συν16χ 1 + συν8χ

ΕΥΚΛΕΙΔΗΣ Β' λη ' τ.l/28


Μαθηματικά Β ' Λυκεiου

8b Α ναλ ο γίε ς - Μ ε τρ ικ ές σχέσεις

του Θανάση Τσιούμα

Στην ύλη της Α ' Λυκείου αντιμετωπίζαμε τα γεωμετρικά σχήματα, από την άποψη της θέσης που είχαν σ ' αυτά, τα διάφορα γεωμετρικά στοιχεία. Τώρα με την εισαγωγή της έννοιας του «μέτρου» θα αντιμετωπίζουμε τα γεωμετρικά σχήματα, από την άποψη των μετρικών σχέσεων των γεωμετρικών τους στοιχείων. ΑΒ Β Γ Σχέσεις όπως για παράδειγμα η ΑΒ 2 ΒΓ ΒΔ ισοδυναμεί με την που είναι σχέση λόγων =

·

=

ΒΔ

ΑΒ

των μέτρων των αντίστοιχων τμημάτων. Ι . ΑΝΑΛΟΓΙΕ Σ •

2.

Το Θεώρημα ΘΑΛΗ το χρησιμοποιούμε για να αποδείξουμε αναλογίες, παραλληλία ευθειών, αλλά και ισότητες τμημάτων.

Δίνεται παραλληλόγραμμο ΑΒΓΔ και Α­ ΕΗΖ τυχαία τέμνουσα. Να αποδείξετε ότι: α) ΑΕ 2 = ΕΖ · ΕΗ

1 = 1 1 ΑΗ +

β) ΑΕ

ΑΖ •

Σ ημείωση

Από δειξη

Η σύγκριση των λόγων γίνεται συνήθως με τη με­ ταφορά τους σε κοινή ευθεία. Ασκήσεις

1.

Από τυχαίο σημείο Ρ της διαμέσου ΑΜ τρι­ γώνου ΑΒΓ, φέρουμε τις παράλληλες προς τις πλευρές ΑΒ, ΑΓ που τέμνουν την ΒΓ στα Κ, Λ αντίστοιχα. Να αποδείξετε ότι

ΒΚ = ΛΓ. Α

Λ ύση

Α

ΑΕ = ΕΗ ( 1 ) . Ζ ΑΕ Ε Η ( 1 ) προφανώς ισχύει αν μεταφέρουμε τους λόγους στην κοινή ευθεία ΔΒ, δηλαδή ΑΕ = ΔΕ = ΕΗ Ζ ΕΒ ΑΕ Ε ΑΕ + ΑΖ ΑΕ (2) β) Αρκεί να δείξουμε ότι: 1 = ΑΗ Όμως είναι: ΑΕ = ΕΒ (3) (ΔW/ΑΒ) α) Α ρκει, να δει'ξουμε οτι: ,

-

f___-___L ---,->-,--__:,.___ �

Β

Κ

Λ

Γ

και

Επειδή ΡΚ//ΑΒ από το Θ. Θαλή έχουμε = ΡΑ (1). ΚΒ -

--

ΜΓ ΑΜ

Από ( 1), (2) έχουμε:: ΚΒ ΛΓ = όμως ΜΒ ΜΓ =

ΜΒ ΜΓ (αφού Μ μέσο της ΒΓ) άρα ΚΒ ΛΓ. =

ΑΖ

-

-

ΒΔ

= ΔΕΑΗ(4) (ΑΔ//ΒΖ). -

ΔΒ

Προσθέτοντας τις (3), (4) κατά μέλη προκύπτει η (2)

ΜΒ ΑΜ

Επίσης ΡΝ/ΑΓ άρα ΛΓ = ΡΑ (2).

ΑΕ

-

Θ Ε Ω Ρ Ή Μ Α Τ Α Τ ΩΝ ΔΙΧΟΤΟ Μ ΩΝ

Η εσωτερική διχοτόμος ΑΔ τριγώνου ΑΒΓ χωρίζει την πλευρά ΒΓ εσωτερικά σε σταθερό λόγο δηλαδή ΔΒ ΑΒ = (1) -

ΔΓ

ΕΥΚΛΕΙΔΗΣ Β ' λη ' τ.l/29

-

ΑΓ


Α

6

� Β

Δ'

Δ

οπότε ΟΜ = 59 = 3. (3). ΜΑ 3 5

Γ

ενώ η εξωτερική διχοτόμος ΑΔ ' χωρίζει τη ΒΓ ε­ Από (2), (3) ισχύει η (1). 2. Έστω Ι το έγκεντρο τριγώνου ΑΒΓ και ΑΔ, ξωτερικά στον ίδιο λόγο δηλαδή Δ'Β = ΒΕ οι διχοτόμοι του. Να βρεθεί ο λόγος ΙΔ ΑΒ (2) ΙΑ Δ'Γ ΑΓ και να αποδείξετε ότι IA>IΔ. Π αρατη ρή σεις Λύ ση Α 1. Ισχύουν ΔΒ = �.ΔΓ = � (μπορεί να διαβ+γ β+γ τυπωθεί και μνημονικός κανόνας). Επίσης Δ'Β = �(β > γ),Δ'Γ = �(β > γ) . β-γ β-γ ΔΒ = Δ'Β άρα τα Δ, Δ ' 2. Από τις ( 1 ), (2) έχουμε ΔΓ Δ'Γ Γ θα είναι τα συζυγή αρμονικά των Β, Γ (Δ, Δ' ΒΙ διχοτόμος του τριγώνου ΑΒΔ θα έχουμε σταθερά) και επειδή ΔΆΔ = 90° προκύπτει ότι Επειδή ΙΔ ΔΒ , ΒΔ = -αγ οποτε , ΙΔ = α . = - ομως ο γεωμετρικός τόπος των σημείων Α για τα ο- β+γ ΙΑ ΑΒ ΙΑ β + γ ΑΒ ' κυ'κλος διαμε-' Είναι α < β+γ (τριγωνική ανισότητα) άρα , ειναι ' ποια - c (σταθ . ) ειναι ΑΓ _α_ < 1 . Επομένως ΙΔ < 1 ή ΙΔ < IA. τρου Δ ' Δ (Απολλώνιος κύκλος). β+γ ΙΑ Λ

--

=

Α σκή σεις

1.

ΟΜΟΙΟΤΗΤ Α ΤΡΙΓΩΝΩΝ

Αν θέλουμε να αποδείξουμε ισότητες της μορφής Στο παρακάτω σχήμα είναι ΟΑ 3, ΟΒ 2 α2 βγ ή αβ γδ και δεν εφαρμόζεται το Θ. Θαλή και ΟΜ = -6 . Αν ΟΔ διχοτόμος της xOy , ή το Θ. διχοτόμων, τότε αυτές ενδεχομένως να 5 προέρχονται από αναλογίες, που είναι συνέπεια να αποδείξετε ότι ΜΔΙ/Oy. ομοιότητας τριγώνων. Υ Επίσης την ομοιότητα των τριγώνων τη χρησιμο­ ποιούμε για τη μέτρηση απρόσιτων τμημάτων. =

=

=

Λ

ο

=

Α σκή σεις χ

1.

Λύ ση

, -ΟΜ = ΔΒ (1) (αντιΑρκει, να αποδει'ξουμε οτι ΜΑ ΔΑ στροφο Θ. Θαλή). ΔΒ = ΟΒ = -2 (2) (Θ. εσωτερικής διχοτό, Ειναι ΔΑ ΟΑ 3 μου). 'Εχουμε ΜΑ = ΟΑ - ΟΜ = 3 - � = 2_

Στο παρακάτω σχήμα είναι ΑΒ 2 = ΒΡ · ΒΓ

( 1).

Α

,

Β

Ρ

Γ

α) Ποια τρίγωνα είναι όμοια; β) Ποια ζεύγη ίσων γωνιών έχουμε;

5 5

ΕΥΚΛΕΙΔΗΣ Β' λη ' τ.l/30


Μαθηματικά Β' Λυκείου

Λύ ση

ΒΑ ΒΓ α) Η (1) γράφεται = - άρα τα τρίγωνα ΒΡ

ΒΑ

α) Να εκφραστεί το εμβαδό του ορθογωνί­ ου ΖΗΘΙ ως συνάρτηση του χ. β) Να βρεθεί η μέγιστη (max) τιμή του. Α

και ΒΑΓ είναι όμοια διότι έχουν τη γωνία ° 2 κριτήριο ομοιότη­ Β κοινή ( σύμφωνα με το τας). β) Τα ζεύγη των ομόλογων πλευρών των τριγώ­ νων είναι ΒΑ - ΒΓ, ΒΡ - ΒΑ, ΑΡ - ΑΓ επομέ­ νως έχουμε αντίστοιχα ΑΡΒ ΒΑΓ, ΒΑΡ = ΑΓΒ και Β : κοινή. ΒΡΑ

Λ

Λ

Λ

Λ

Γ

Λ

Λύ ση

α) Είναι EzHeι = ΖΗ ΗΘ = χ · y (1). Στο παρακάτω σχήμα να υπολογιστεί το Τα τρίγωνα ΑΙΘ και ΑΒΓ είναι όμοια και ο λό­ ύψος του κτηρίου ΑΒ = χ όταν ΚΛ = 3m γος ομοιότητάς τους, ισούται με το λόγο των (ΚΛ ράβδος) και ΓΔ = 1,80 m (ύψος αν­ ομόλογων υψών, άρα έχουμε: ΙΘ ΑΡ η, � _- 3 - y . θρώπου). 5 3 ΒΓ - ΑΔ Λύση Β ' 3 χ = 15 - 5 y η' y = 15 - 3χ και η (1) γι-' οποτε 5 νεται: Ε = χ - 3 - � χ ή Ε = - � χ 2 + 3χ . =

·

2.

_

(

)

β) Το εμβαδόν Ε(χ) = - � χ 2 + 3χ (τριώνυμο του 5 Α -β 5 3 ' 0 ' = = χ < α για επομενως χ ) -εχει 2, 2α = Έστω ότι η ΒΔ τέμνει την ΚΑ στο Ρ, τότε θέτουμε 5 -Δ 15 ΡΛ = y και ΡΚ ω. παρουσιάζει max το Emax = =4α 4 τ.μ. Από την ομοιότητα των τριγώνων ΔΡΛ και ΔΑΒ , ΡΛ ΔΛ , y 11. Μ Ε Τ ΡΙ ΚΕ Σ Σ ΧΕ Σ ΕΙ Σ εχουμε = - η - = -1 αρα y = -χ (1) . ΒΑ ΔΑ χ 11 11 Επίσης από την ομοιότητα των τριγώνων ΒΡΚ και Οι μετρικές σχέσεις στο ορθογώνιο τρίγωνο αλλά και το γενικευμένο Πυθαγόρειο Θεώρημα, καθορί­ , ΚΡ ΒΚ ΑΛ ΒΔΓ εχουμε - = - = (από Θ. Θαλή) ή ΑΔ ζουν και τα υπόλοιπα στοιχεία ενός τριγώνου. ΓΔ ΒΓ Το Πυθαγόρειο Θεώρημα προκύπτει από την ο­ � = 10 δ λ ω = � (2) . . η 1,80 1 1 11 μοιότητα τριγώνων και είναι σχέση μεταξύ των 18 15 εμβαδών των τετραγώνων, που ορίζονται από τις ' Είναι y + ω = 3 άρα y = 3 - - = - . Επομενως 11 11 πλευρές του. Έτσι μεγέθη όπως ΑΒ2 , ΑΒ ΑΓ .... 15 δηλαδή παριστάνουν γινόμενα μέτρων των αντίστοιχων από την (1) προκύπτει χ = 1 1 y = 1 1 · 11 τμημάτων και εκφράζουν εμβαδό. χ = 15 m. Με τα θεωρήματα των διαμέσων μπορούμε να προσδιορίζουμε κάποια δευτερεύοντα στοιχεία ε3. Στο παρακάτω σχήμα έχουμε ένα μεταβλη- νός τριγώνου. τό ορθογώνιο ΖΗΘΙ εγγεγραμμένο σε τρί- Με τις μετρικές σχέσεις στον κύκλο (Θεωρήματα τεμνόμενων χορδών και τέμνουσας και εφαπτομέ­ γωνο ΑΒΓ, όπου νης) μελετούμε το γινόμενο δύο τμημάτων, που ΒΓ = α = 5 και ΑΔ = υa = 3. =

,

ΕΥΚΛΕΙΔΗΣ Β' λη ' τ.l/31

·


Μαθηματικά Β' Λυκείου Γ

ορίζονται από μία τέμνουσα ενός κύκλου και από ένα σταθερό σημείο, που θα είναι ανεξάρτητο από τη θέση της τέμνουσας. Ασκήσεις

1.

Λ

β

Δ

Λ

Δίνεται τρίγωνο ΑΒΓ με Β- Γ = ι20ο • Έστω ΑΔ = δ η διχοτόμος της γωνίας Α και ΑΡ το ύψος του:

I<....L..-----'--' Β γ κ

Λ

Λύση

α) Να υπολογιστεί η γωνία ΔΑΡ . δ .J3 β) Να αποδείξετε ότι ΡΔ = .

Έστω ΑΔ = δ η διχοτόμος της γωνίας Α. Φέρουμε ΔΚ ..l ΑΒ . Αν ΔΚ = χ, τότε από την ο­ 2 μοιότητα των τριγώνων ΒΔΚ και ΒΓΑ έχουμε: Α ΒΚ = ΔΚ ή γ - χ = � (1) ΒΑ ΓΑ γ β διότι το τρίγωνο ΑΚΔ είναι ορθογώνιο ισοσκελές άρα ΑΚ = ΚΔ = χ οπότε ΒΚ = γ - χ. Από την (1) έχουμε Λύ σ η βγ - βχ = γχ <=:> χ(β + γ) = βγ <=:> ! = β + γ ή χ βγ α) Είναι ΔΑΡ = Α1 + ω όπου Α1 = Β - 90° (διότι -1 = -1 + -1 (2) Β = 90° + λ ως εξωτερική του τριγ. ΑΡΒ) και χ γ β ω = Α άρα ΔΑΡ = Β - 90° + Α = Όμως από το Πυθ . θεώρημα στο τρίγωνο ΑΔΚ. 2 2 Είναι χ 2 + χ 2 = δ 2 <=:> 2χ 2 = δ 2 <=:> .fix = δ ή = Β _ Α _ Γ _ Β + Α = Β - Γ = 120° = 600 . .J2 2 2 2 2 2 2 -1 = (3) δ χ Ρ β) Επειδή Μ = 60° στο ορθογώνιο τρίγωνο Από (2), (3) προκύπτει ότι .J2 = + ΡΑΔ θα έχουμε ότι η Δ1 = 30° επομένως δ β γ ΑΔ ΡΑ = - = δ/ 2 . 2 3. Δίνονται τα μήκη α = �5 + Jϊ8 , β = J3 και Εφαρμόζουμε το Πυθαγόρειο Θεώρημα στο ί­ γ = .fi . διο τρίγωνο οπότε προκύπτει ότι: α) Να αποδείξετε ότι τα α, β, γ είναι μήκη 2 πλευρών τριγώνου ΑΒΓ, να βρεθεί το ΡΔ2 = ΑΔ2 - ΡΑ 2 ή ΡΔ2 = δ 2 - % ή Λ

Λ

Λ

Λ

_!_ _!_

Λ

()

2.

είδος του και να κατασκευαστεί. β) Να υπολογιστεί η γωνία Α.

ΡΔ2 = 3δ 2 ή ΡΔ = δ -/3 . 2 4 ΛίJ ση Αν β, γ είναι οι κάθετες πλευρές ορθογωνί­ α) Παρατηρούμε ότι α > β > γ. ου τριγώνου ΑΒΓ και δ η διχοτόμος της Για να είναι τα α, β, γ μήκη πλευρών τριγώνου αρκεί β - γ < α < β + γ ή ορθής γωνίας Α να δεχτεί ότι: .fi ι ι J3 - .fi < �5 + Jl8 < J3 + .fi . =-+δ β γ Η ανισότητα J3 - .J2 < J5 + Jl8 προφανώς ισχύει, αρκεί (συνέχεια στη σελ. 47)

ΕΥΚΛΕΙΔΗΣ Β' λη ' τ.l/32



4511 ΔΙΕθΝΗΣ ΜΑ θΗΜΑ ΤΙΚΗ ΟΛ ΥΜΠΙΑΔΑ

Αθήνα, 4-18 Ιουλίου 2004

Διεθνής Μαθηματική Ολυμπιάδα (lnternational Mathematical Olympiad I ΙΜΟ) είναι ένας διε­ θνής μαθηματικός διαγωνισμός για νέους από όλον τον κόσμο. Ο πρώτος διαγωνισμός έγινε στην Ρουμανία το 1959 και από τότε γίνεται κάθε χρόνο σε διαφορετική χώρα. Στην αρχή στην ΙΜΟ συμμετείχαν λίγες χώρες, ως επί το πλείστον της Ανατολικής Ευρώπης. Αλλά τα τελευταία χρόνια συμμετέχουν χώρες από όλο τον κόσμο και συνεχώς οι συμμετοχές αυξάνονται πλησιάζοντας τις ενενήντά (90). Κάθε χώρα συμμετέχει με την εθνική της ολυμπιακή ομάδα, που αποτε­ λείται από έξι (6) διαγωνιζόμενους μαθητές, ένα αρχηγό, ένα υπαρχηγό ως συνοδό των μαθητών και πα­ ρατηρητές. Η Ελλάδα συμμετέχει στις Διεθνείς Μαθηματικές Ολυμπιάδες με ομάδες την ευθύνη της επιλογής των οποίων έχει η Ελληνική Μαθηματική Εταιρεία (ΕΜΕ). Για αυτόν το σκοπό η ΕΜΕ πραγματοποιεί κάθε χρόνο τρεις (3) πανελλήνιους μαθητικούς �ιαγωνισμούς μέσα από τους οποίους επιλέγεται η 6μελής ομάδα και οι αναπληρωματικοί, που θα συμμετάσχουν στην ΙΜΟ, η οποία πραγματοποιείται στα μέσα Ιουλίου, κάθε χρόνο. Μέλη της Ελληνικής Μαθηματικής Εταιρείας εργάζονται εθελοντικά παραδίδοντας χωρίς αμοιβή μαθήματα καθόλη τη διάρκεια του έτους σε όσους μαθητές φιλοδοξούν να συμμετάσχουν στην ομάδα. Η επιστημονική αρτιότητα της Ελληνικής Μαθηματικής Εταιρείας, η εμπειρία που διαθέτει από την συμμετοχή σε διεθνείς διαγωνισμούς, καθώς και τη διοργάνωση μαθηματικών διαγωνισμών σε διεθνές επίπεδο (Βάλκανικές Μαθηματικές Ολυμπιάδες), οι καλές σχέσεις που έχει αναπτύξει με τη διεθνή εκ­ παιδευτική και μαθηματική κοινότητα την ώθησαν να διεκδικήσει τη διοργάνωση Διεθνούς Μαθηματι­ κής Ολυμπιάδας. Από το 1997 η ΕΜΕ άρχισε την ενημέρωση της διεθνούς μαθηματικής κοινότητας για την πρόθεσή της να ανάλάβει την διοργάνωση Ολυμπιάδας. Το 1999 στο Βουκουρέστι, στο πλαίσιο της 40ης ΙΜΟ η Ελληνική αποστολή, αφού κατέθεσε πλήρη φάκελο στη Διεθνή Επιτροπή Διοργάνωσης Μαθηματικών Ολυμπιάδων (lnternational Mathematical Olympiad Advisory Board/ ΙΜΟ ΑΒ), της ανατέθηκε η διοργά­ νωση και διεξαγωγή της 45 ης ΙΜΟ τον Ιούλιο του 2004, στην Ελλάδα. Την επιτυχή κατάληξη της διεκδίκησης το 1999 ακολούθησε ένας κύκλος δραστηριοτήτων με σκοπό την προβολή και περαιτέρω προετοιμασία του όλου εγχειρήματος. Η ΕΜΕ προχώρησε στη διοργάνωση της 45 ης Διεθνούς Μαθηματικής Ολυμπιάδας (45 1h ΙΜΟ) υπό την αιγίδα του Υπουργείου Εθνικής Παιδείας και Θρησκευμάτων, του Υπουργείου Πολιτισμού και της Νομαρχίας Αθηνών. Την οικονομική διαχείριση της όλης διοργάνωσης είχε ο ειδικός λογαριασμός του Ελληνικού Ανοι­ κτού Πανεπιστημίου σε συνεργασία με το Εθνικό Ίδρυμα Νεότητας και ομάδα μελών της ΕΜΕ με υπεύ­ θυνο τον Αναπληρωτή Καθηγητή του Πανεπιστημίου Αθηνών κ. Ευάγγελο Ντζιαχρήστο.

Η

ΕΥΚΛΕΙΔΗΣ Β' λη ' τ.l/34


45η Διεθνής Μαθηματική Ολυμπιάδα

Η 45η Διεθνής Μαθηματική Ολυμπιάδα πραγματοποιήθηκε στους Δελφούς και την Αθήνα από 4 έως 18 Ιουλίου του 2004 και συμμετείχαν οι παρακάτω ογδονταοκτώ (88) χώρες.

Συμμετοχές με ομάδ α μαθητών (85 χώρες) ΑΖΕΡΜΠΑΪΖΑΝ ΑΛΒΑΝΙΑ

ΕΛΒΕΠΑ

ΜΟΛΔΑΒΙΑ

ΚΟΡΕΑ

ΣΑ ΟΥΔΙΚΗ ΑΡΑΒΙΑ ΣΕΡΒΙΑ ΚΑΙ

ΕΛΛΑΣ

ΚΟΥΒΑ

ΝΕΑ ΖΗΛΑΝΔΙΑ

ΑΡΓΕΝΠΝΗ

ΕΣΘΟΝΙΑ

ΚΟΥΒΕΪΤ

ΝΟΡΒΗΓΙΑ

ΑΡΜΕΝΙΑ

ΗΠΑ

ΚΡΟΑ ΤΙΑ

ΝΟΤΙΟΣ Α ΦΡΙΚΗ

ΣΛ ΟΒΑΚΙΑ

Α ΥΣrΡΑΛΙΑ

!ΑΠΩΝ/Α

ΚΥΠΡΟΣ

ΟΛΛΑΝΔΙΑ

ΣΛ ΟΒΕΝΙΑ

Α ΥΣrΡΙΑ

!ΝΔ/Α

ΚΥΡΓΚ/ΣrΑΝ

Ο ΥΓΓΑΡΙΑ

ΣΟ ΥΗΔΙΑ

ΒΕΛΓΙΟ

ΙΝΔ ΟΝΗΣΙΑ

ΛΕΥΚΟΡΩΣΙΑ

ΟfΖΜΠΕΚ/ΣrΑΝ

ΣΡΙ ΛΑΝΚΑ

ΒΕΝΕΖΟΥΕΛΑ

ΙΡΑΝ

ΛΕΤΟΝΙΑ

ΟΥΚΡΑΝΙΑ

ΤΑΪΒΑΝ

ΒΙΕΤΝΑΜ

ΙΡΛΑΝΔΙΑ

ΛΙΘΟΥΑΝΙΑ

ΟΥΡΟΥΓΟΥΆΗ

ΤΑΪΛΑΝΔΗ

ΒΟΣΝΙΑ ΕΡΖΕΓΟΒΙΝΗ

ΙΣΛΑΝΔΙΑ

ΛΟΥΞΕΜΒΟΥΡΓΟ

ΠΑΡΑΓΟΥΆΗ

ΤΟΥΡΚ/Α

ΒΟΥΛΓΑΡΙΑ

/ΣΠΑΝ/Α

ΜΑΚΑΟ

ΠΕΡΟΥ

ΒΡΑΖΙΛΙΑ

ΙΣΡΑΗΛ

ΜΑΛΑΙΣΙΑ

ΠΟΛΏΝΙΑ

ΓΑΛΛ/Α

ΠΑΛΙΑ

ΜΑΡΟΚΟ

ΠΟΡΤΟΓΑΛΙΑ

ΤΣΕΧΙΑ

ΓΕΡΜΆΝΙΑ

ΚΑΖΑΚΣrΑΝ

ΜΕΓΑΛΗ ΒΡΕΤΑΝΙΑ

ΠΟΥΕΡrΟ Ρ/ΚΟ

ΤΥΝΗΣΙΑ

ΜΑ ΥΡΟ-

ΒΟΥΝ/0 ΣΙΓΚΑΠΟΥΡΗ

ΤΟΥΡΚΜΕΝΙΣΤΆΝ

TPININTANT ΚΑΙ ΤΟΜΠΑΓΚΟ

ΠΡ. ΓΙΟΥΚΟΣΛΑΒΙΚΗ ΓΕΩΡΓΙΑ

ΚΑΝΑΔΑΣ

ΔΗΜΟΚΡΑ ΠΑ ΤΗΣ

ΜΕΞΙΚΟ

ΦΙΛΑΝΔΙΑ

ΜΑΚΕΔ ΟΝΙΑΣ ΔΑΝ/Α

Κ/ΝΑ

ΜΟΓΚΟΛΙΑ

ΡΟΥΜΆΝΙΑ

ΦΙΛΙΠΠΙΝΕΣ

ΕΚΟΥΑΝΔ ΟΡ

ΚΟΛ ΟΜΒΙΑ

ΜΟΖΑΜΒΙΚΗ

ΡΩΣΙΑ

ΧΟΝΚ ΚΟΝΓΚ

I ΜΠΑΓΚΛΑΝΤΕΣ

Συμμετοχές μόνο με παρατηρητές (3 χώρες)

I ΤΑΤΖΙΚΙΠΑΝ

I ΛΙΧΝΕΝΠΑΥΝ

Υπεύθυνος της ομάδας που είχε αναλάβει την προβολή της 45ης Διεθνούς Μαθηματικής Ολυ­ μπιάδας και τις δημόσιες σχέσεις ήταν ο Καθηγη­ τής του Εθνικού Μετσόβιου Πολυτεχνείου και μέ­ λος του Δ.Σ. της Ε.Μ.Ε. κ. Κωνσταντίνος Λασκα­ ρίδης Στις 4 Ιουλίου, ημέρα Κυριακή, ήρθαν αερο­ πορικώς στην Ελλάδα τα μέλη της Διεθνούς Επι­ τροπής Οργάνωσης Μαθηματικών Ολυμπιάδων (IMO ΑΒ), και πήγαν στους Δελφούς μαζί με τον Πρόεδρο της Οργανωτικής Επιτροπής του ΙΜΟ 2004 καθηγητή κ. Νικόλαο Αλεξανδρή, ο οποίος είναι και μέλος του ΙΜΟΑΒ.

Η Διεθνής Επιτροπή Οργάνωσης τωv Μαθηματικών Ολυμπιάδων συνεδριάζει στους Δελφούς

Τα μέλη της ΙΜΟΑΒ φιλοξενήθηκαν στον ξενώνα του Ευρωπαϊκού Πολιτιστικού Κέντρου Δελφών. Εκεί στις 5 Ιουλίου, ημέρα Δευτέρα, συνεδρίασαν και συζήτησαν θέματα σχετικά με τη διεξαγωγή της ΙΜΟ 2004, τις μελλοντικές διοργανώσεις, καθώς και θέματα που αφορούν στο θεσμό των Διεθνών Μα­ θηματικών Ολυμπιάδων. Στον ξενώνα του Ευρωπαϊκού Πολιτιστικού Κέντρου Δελφών κατέλυσαν επίσης αρχηγοί των ομά­ δων των 88 χωρών που συμμετείχαν σ ' αυτήν την Ολυμπιάδα. Οι αρχηγοί άρχισαν να φθάνουν σταδιακά μέσω του αεροδρομίου Ελ. Βενιζέλος της Αθήνας με ευθύνη του Α ' Αντιπροέδρου της Ελληνικής Μα­ θηματικής Εταιρείας, Επίκουρου Καθηγητή του Πανεπιστημίου Αθηνών κ. Γεώργιου Δημάκου και εθε­ λοντών μελών της Ελληνικής Μαθηματικής Εταιρείας από την Τρίτη 6 Ιουλίου. Λόγω του μεγάλου αΕΥΚΛΕΙΔΗΣ Β' λη ' τ.l/35


45η Διεθνής Μαθηματική Ολυμπιάδα

ριθμού των συμμετεχόντων, αρχηγοί ομάδων έμειναν σε δύο ακόμη κοντινά ξενοδοχεία της περιοχής των Δελφών. τις ημέρες που ακολούθησαν, στην αίθουσα συνεδριάσεων του Ευρωπαϊκού Πολιτιστικού Κέντρου Δελφών, οι αρχηγοί, που είναι διακεκριμένοι μαθηματικοί, καθηγητές πανεπιστημίων των χωρών τους, σε συνεχείς συνεδριάσεις της Διεθνούς Επιτροπής Κριτών (Intemational Jury Committee) με πρόεδρο τον Καθηγητή του Πανεπιστημίου Πατρών κ. Γεώργιο Δάσφ, επέλεξαν τα 6 προβλήματα στα οποία θα εξετάζονταν οι μαθητές. Τα προβλήματα αυτά επιλέχθηκαν από μια ομάδα 30 προβλημάτων (short list), στα οποία είχε καταλήξει η Επιτροπή Επιλογής Προβλημάτων, υπό την προεδρία του Καθηγητή του Πα­ νεπιστημίου των Ιωαννίνων κ. Θεόδωρου Μπόλη. Η επιτροπή αυτή σε επανειλημμένες συνεδριάσεις στα Ιωάννινα και την Αθήνα επεξεργάστηκε τα προβλήματα που είχε στείλει η καθεμιά από τις συμμετέχου­ σες χώρες (6 προβλήματα η κάθε χώρα), όπως ορίζεται από τους κανονισμούς της Ολυμπιάδας. Από άυ­ τά κατάρτισε τη λίστα των 30 προβλημάτων (short list) την οποία υπέβαλε στην ολομέλεια των αρχηγών των ομάδων (Jury Committee). Στη συνέχεια τα έξι προβλήματα μεταφράστηκαν από τους 85 αρχηγούς σε 56 διαφορετικές γλώσσες.

Ο πρόεδρος της Διεθνούς Επιτροπής Κριτών κ. Γεώργιος Λάσιος, Καθηγητής του Πανεπιστημίου της Πάτρας και ο Καθηγητής στο ίδιο Πανεπιστήμιο κ. Θεόδωρος Παπαθεο­ δώρου προεδρεύουν συνεδρίασης της Διεθνούς Επιτροπής Κριτών στους Δελφούς

Μέλη της Διεθνούς Επιτροπής Κριτών ψηφίζουν κατά τη διάρκεια συνεδρίασης της Διεθνούς Επιτροπής Κριτών στους Δελφούς

. Κατά την παραμονή τους στους Δελφούς, οι αρχηγοί και τα μέλη των επιτροπών επισκέφθηκαν τον Αρχαιολογικό χώρο των Δελφών και ξεναγή­ θηκαν στο αρχαιολογικό μουσείο της πόλης. Επί­ σης πραγματοποιήθηκαν δύο επισκέψεις στην Ιτέα - Γαλαξείδι, με ξενάγηση στο ναυτικό και εθνολο._ γικό μουσείο του Γαλαξειδιού και στο μοναστήρι του Οσίου Λουκά. Επίσης αξίζει να σημειωθεί ότι οι Δήμαρχοι των γύρω περιοχών έδειξαν με τον καλύτερο τρόπο την Ελληνική φιλοξενία προσφέ­ ροντας κάθε βράδυ σε παραδοσιακές ταβέρνες δεί­ πνο προς τιμήν των μελών της 45ης Διεθνούς Μα­ θηματικής Ολυμπιάδας Τα μέλη της Διεθνούς Επιτροπής Κριτών μπροστά Συγχρόνως την Αθήνα γίνονταν προετοιμασίες στο Ευρωπαϊκό Πολιτιστικό Κέντρο Δελφών για την υποδοχή των ομάδων των μαθητών και των συνοδών, από τις 88 χώρες, οι οποίες θα έφθαναν στις 9 και 10 Ιουλίου. Στις 8 Ιουλίου συγκεντρώθηκαν στην αίθουσα διαλέξεων της ΕΜΕ οι εθελοντές, που θα αναλάμβαναν την ευθύνη να συνοδεύουν και να διευκολύ­ νουν τις ομάδες κατά την παραμονή τους στην Ελλάδα, υπό την προεδρία του Α' Αντιπροέδρου της ΕΜΕ κ. Γεώργιου Δημάκου, ο οποίος ήταν συντονιστής των δραστηριοτήτων της 45ης ΙΜΟ στην Αθήνα. ΕΥΚΛΕΙΔΗΣ Β' λη ' τ.Ι/36

1


45" Διεθνής Μαθηματική Ολυμπιάδα

Εδώ πρέπει να τονίσουμε ότι το ιδιαίτερα δύσκολο αυτό εγχείρημα στηρίχθηκε εξολοκλήρου στην εθελοντική προσφορά και εργασία μελών της Ε.Μ.Ε., των επιτροπών της και των μελών του Δ.Σ. Στη γραμματεία κατατέθηκε πληθώρα αιτήσεων εθελοντικής προσφοράς φοιτητών και μεταπτυχια­ κών φοιτητών, καθώς και συναδέλφων μαθηματικών οι οποίοι προσφέρονταν να συνοδεύ9μν τις ομάδες σε κάθε τους βήμα, να τις βοηθούν και να συμπαρίστανται σ ' αυτές από την άφιξή τους στο αεροδρόμιο και καθ ' όλη τη διάρκεια της Ολυμπιάδας μέχρι την αναχώρησή τους. Οι εθελοντές ενημερώθηκαν για τα καθήκοντά τους και παρέλαβαν το υλικό που θα διένειμαν στους μαθητές (μπλουζάκια, καπέλα, χάρτες, γραφική ύλη). Η Παρασκευή και το Σάββατο 9 και 10 Ιουλίου κύλησαν με τις συνεχείς αφίξεις ομάδων (6 μαθητές και 1 υπαρχηγός - συνοδός),στο αεροδρόμιο Ελ. Βενιζέλος, την υποδοχή τους και τη μεταφορά τους σε κεντρικό ξενοδοχείο των Αθηνών (President). Την Κυριακή 1 1 Ιουλίου 2004, το πρωί έγινε περιήγηση των μαθητών στην πόλη της :Αθήνας, στις Ολυμπιακές εγκαταστάσεις, καθώς και επίσκεψη στο χώρο που θα διεξαγόταν ο διαγωνισμός. Εκεί στο μεγάλο αμφιθέατρο του Μαθηματικού Τμήματος του Πανεπιστήμίου Αθηνών μίλησε προς όλες. τις απο­ στολές ο Αναπληρωτής Πρόεδρος του τμήματος Καθηγητής Γρηγόρης Καλογερόπουλος. Το απόγευμα της Κυριακής, στις 16.00 στο Μέγαρο φίλων της Μουσικής Αθηνών, έγινε η Τελετή Έναρξης της 45ης Διεθνούς Μαθηματικής Ολυμπιάδας. Το Μέγαρο κατακλύσθηκε από νέα παιδιά, 15-20 ετών, που φωτογραφίζονταν χαμογελαστά, κρατώ­ ντας τις σημαίες των χωρών τους αψηφώντας τη ζέ­ στη της ημέρας. Στο θεωρείο των επισήμων του Μεγάρου φίλων της Μουσικής Αθηνών βρίσκονταν οι αρχηγοί των ομάδων και μέλη επιτροπών που είχαν έλθει από τους Δελφούς, απομονωμένοι από τους μαθητές, ό­ πως ορίζουν οι κανονισμοί, για να παρακολουθήσουν την εκδήλωση. Η τελετή ξεκίνησε με ενθουσιασμό, με την πα­ ρουσίαση των ομάδων των μαθητών, αλφαβητικά Πε ιμένοντας 'για την τελετή έναρξης στο Μέ'γαρο ρ κατά χώρα, από τη σκηνή του Μεγάρου Μουσικής. των Φίλων της Μουσικής Αθηνών Συγχρόνως με την εμφάνιση των μαθητών κάθε χώρας στην σκηνή προβάλλονταν σε ειδική οθόνη στοιχεία για την χώρα και ακουγόταν χα­ ρακτηριστικό απόσπασμα εθνικής μουσικής. Μετά την ολοκλήρωση της παρουσίασης των 86 εθνικών ομάδων μαθητών, η τελετή συνεχί. στηκε με ομιλίες. Ομιλητές ήταν ο Πρόεδρος της Οργανωτικής Επιτpοπής της 45ης Διεθνούς Μα­ θηματικής Ολυμπιάδας και Πρόεδρος της Ελλη­ νικής Μαθηματικής Εταιρείας Καθηγητής κ. Νι­ κόλαος Αλεξανδρής, ο Πρόεδρος της Διεθνούς Επιτροπής Διοργάνωσης Μαθηματικών Ολυ­ μπιάδων, Καθηγητής κ. Joszef Pelikan, που μί­ λησε και σε άπταιστα Ελληνικά, η Αναπληρωτής Υπουργός Πολιτισμού κ. Φάνη Πάλλη Πετραλιά, ο Νομάρχης Αθηνών κ. Ιωάννης Σγουρός και ο Η Αναπληρωτής Υπουργός Πολιτισμού Υ φυπουργός Εθνικής Παιδείας και Θρησκευμά­ κ. Φάνη Πάλλη Πετραλιά των κ. Γεώργιος Καλός, ο οποίος κήρυξε την έ­ στην ομιλία της κατά την τελετή έναρξης ναρξη της διοργάνωσης. ΕΥΚΛΕΙΔΗΣ Β' λη ' τ.U37


45η Διεθνής Μαθηματική Ολυμπιάδα

Όλοι τόνισαν τη σπουδαιότητα της διοργά­ νωσης, στάθηκαν στο υψηλό επίπεδο που βρί­ σκεται η μαθη ματική επιστήμη στην Ελλάδα, γε­ γονός που ήταν και ο λόγος που της ανατέθηκε η Ολυμπιάδα, και επισή μαναν την ευτυχή συγκυ­ ρία της πραγματοποίησής της, την ίδια χρονιά με την τέλεση των Ολυμπιακών Αγώνων.

Ακολούθησε ομιλία του Καθηγητή του Πανεπι­ στημίου του Καίμπριτζ και βραβευμένου με το βρα­ βείο Naylor και το Αριστείο της Ακαδημίας Αθηνών, κ. Θανάση Φωκά. Ο Πρόεδρος της Διεθνούς Επιτροπής Διοργάνωσης Μα­ θηματικών Ολυμπιάδων, Καθηγητής κ. Joszef Pelikan στην Τελετή Έναρξης στο Μέγαρο Μουσικής Αθηνών

Υφυπουργός Εθνικής Παιδείας και Θρησκευμάτων κ. Γεώρ­ γιος Καλός, ο οποίος κήρυξε την έναρξη της 45ης Διεθνούς Μαθηματικής Ολυμπιάδας

Ο Νομάρχης Αθηνών κ. Ιωάννης Σγουρός στην Τελετή Έναρξης στο Μέγαρο Μουσικής Αθηνών

Το καλλιτεχνικό πρόγραμμα που ακολούθησε, περιελάμβανε την παρουσίαση των αρχαίων ελληνι­ κών οργάνων «'Ύδραυλις » και «Χαλκεόφωνο» και την παρουσίαση ελληνικών παραδοσιακών χορών και τραγουδιών από την Μακεδονία, Μικρά Ασία και Θ ρ άκη, από το χορευτικό συγκρότημα του Δήμου Κερατσινίου. Μετά το τέλος της τελετή ς οι μαθητές με τους υπαρχηγούς των ομάδων και τους εθελοντές συνο­ δού ς τους επέστρεψαν στο ξενοδοχείο τους και οι αρχηγοί των ομάδων με τα μέλη της Διεθνούς Επι­ τροπής Ο ργάνωση ς Μαθηματικών Ολυμπιάδ ων και άλλα μέλη επιτροπών στους Δελφούς. Ο Καθηγητής του Πανεπιστημίου του Κέιμπριτζ κ. Αθανάσιος Φωκάς απονέμει μετάλλιο. ΕΥΚΛΕΙΔΗΣ Β ' λη ' τ.l/38


45η Διεθνής Μαθηματική Ολυμπιάδα Q Τη Δευτέρα και Τρίτη στις 1 2 και 13 Ιουλίου 2004, σε ειδικά διαμορφωμένη αίθουσα, στο κτίριο του Μαθηματικού Τμήματος του Πανεπιστημίου Αθηνών στην Πανεπιστημιούπολη Ζωγράφου έγινε ο δια­ γωνισμός της 45ης Διεθνούς Μαθηματικής Ολυμπιάδας με συντονιστή τον Α' Αντιπροέδρο της ΕΜΕ κ. Γεώργιο Δημάκο. Οι μαθητές διαγωνίσθηκαν σε 6 θέματα, τρία κάθε μέρα, τα οποία έπρεπε να λύσουν μέσα σε 4 Υ2 ώρες κάθε φορά. Τα θέματα στάλθηκαν με φαξ από τους Δελφούς στην Αθήνα τη Δευτέρα και την Τρίτη το πρωί και αναπαράχθηκαν στο Πανεπιστήμιο Αθηνών από ειδική επιτροπή.

Μαθητές κατά τη διάρκεια του διαγωνισμού

Κάθε διαγωνιζόμενος έλαβε τα θέματα στη μητρική του γλώσσα και στην αγγλική γλώσσα. Οι δια­ γωνιζόμενοι μπορούσαν να υποβάλουν γραπτές ερωτήσεις, που μέσω φαξ διαβιβάζονταν στην επιτροπή κριτών (lnternational Jury Committee) του διαγωνισμού που συνεδρίαζε στους Δελφούς και η οποία σε ολομέλεια έκρινε και έδινε την απάντηση στον διαγωνιζόμενο, στη γλώσσα του, μέσω φαξ. Την πρώτη ημέρα του διαγωνισμού τέθηκαν περίπου 20 ερωτήματα και τη δεύτερη περίπου 60. Η διαδικασία τελούσε υπό την εποπτεία του προέδρου του τμήματος Μαθηματικών του Πανεπιστη ­ μίου Αθηνών, Καθηγητή κ. Σταύρου Παπασταυρίδη. Με τη λήξη του διαγωνισμού, τα γραπτά κάθε δια­ γωνιζόμενου μπήκαν σε σφραγισμένους φακέλους. Στη συνέχεια φωτοτυπήθηκαν και ταξινομήθηκαν από την ομάδα διεξαγωγής του διαγωνισμού, για να παραδοθούν αντίγραφα των γραπτών στην Επιτροπή Βαθμολογητών, πρόεδρος της οποίας ήταν ο Καθηγητής του Πανεπιστημίου Αθηνών κ. Στυλιανός Αν­ δρεαδάκης, και τα πρωτότυπα γραπτά στους αρχηγούς των ομάδων για τη βαθμολόγησή τους. Οι αρχηγοί αναχώρησαν από τους Δελφούς την Τρίτη 1 3 Ιουλίου το μεσημέρι και κατέλυσαν στο ξε­ νοδοχείο Ledra Mariott στην Αθήνα. Την επόμενη μέρα Τετάρτη 14 Ιουλίου ξεκίνησε η βαθμολόγηση και η εξαγωγή των αποτελεσμάτων σε ειδικά διαμορφωμένες αίθουσες.

Βαθμολόγηση γραπτών σε ειδικά διαμορφωμένες αίθουσες στο ξενοδοχείο Ledra Maήott ΕΥΚΛΕΙΔΗΣ Β ' λη ' τ.V39


45η Διεθνής Μαθηματική Ολυμπιάδα

Στη βαθμολόγηση συμμετείχαν αμισθί έμπειροι βαθμολογητές, μέλη της ελληνικής πανεπιστημιακής κοινότητας, μέλη της επιτροπής διαγωνισμών της ΕΜΕ και πρώην Ολυμπιονίκες φοιτητές και πτυχιούχοι ελληνικών και ξένων πανεπιστημίων. Η βαθμολόγηση διήρκεσε 2 ημέρες. Η καταχώρηση της βαθμολο­ γίας των γραπτών σε ηλεκτρονική βάση δεδομένων και η έκδοση των αποτελεσμάτων έγινε από ομάδα μελών της ΕΜΕ, με υπεύθυνο τον Δημήτρη Ζέρβα πρώην μέλος του Δ.Σ. της ΕΜΕ Στο διάστημα αυτό οι μαθητές, απαλλαγμένοι πια από το άγχος των εξετάσεων επισκέφθηκαν μαζί με τους συνοδούς τους και τους έλληνες εθελοντές οδηγούς τους την Ακρόπολη και το Ναό του Ποσει­ δώνα στο Σούνιο. Το απόγευμα της Τετάρτης 14 Ιουλίου η Πρόεδρος της Βουλής των Ελλήνων κ. Άννα Ψαρούδα Μπενάκη δεξιώθηκε τις εθνικές αποστολές και τα μέλη της οργανωτικής επιτροπής της ΙΜΟ 2004 στον προαύλιο χώρο της Βουλής, τους απεύθυνε θερμό καλωσόρισμα και συνομίλησε για αρκετή ώρα μαζί

Ομάδες συμμετεχόντων προσέρχονται

Η ομάδα του Μεξικού στο αίθριο της Βουλής των Ελλήνων

Η Πρόεδρος τη ς Βουλής των Ελλήνων κ. Άννα Ψαρούδα Μπενάκη συνομιλεί με μαθητές

Η Πρόεδρος της Βουλής των Ελλήνων κ. Άννα Ψαρούδα Μπενάκη Ο Γενικός Γραμματέας της Βουλής κ. Γεώρ-yιος Καραμπατζός, ο Πρόεδρος της 4�ς ΙΜΟ κ. Νικόλαος Αλεξανδρής, ο Πρόεδρος της ΙΜΟ/ΑΒ Jozsef Pelikan και ο Γραμματέας της ΙΜΟ/ΑΒ John Webb, στο αίθριο της Βουλής των Ελλήνων

Την Παρασκευή 1 6 Ιουλίου οργανώθηκε για όλους τους συμμετέχοντες στην ολυμπιάδα, εκδρομή και ξενάγηση στις Μυκήνες, όπου επισκέφθηκαν το Θησαυρό του Ατρέα, την Ακρόπολη και το Μουσείο των Μυκηνών και το Ναύπλιο, όπου τους δόθηκε η ευκαιρία να περιπλανηθούν στα γραφικά δρομάκια της πόλης και στο αρχαίο θέατρο της Επιδαύρου, όπου εντυπωσιάσθηκαν από τη φυσική ομορφιά του τοπίου και την άψογη ακουστική του θεάτρου. ΕΥΚΛΕΙΔΗΣ Β' λη ' τ.V40


45η Διεθνής Μαθηματική Ολυμπιάδα

Εκδ ρ ομή και ξενάγηση στην Ακρ όπολη των Μυκηνών

Μαθητές στην Ακρόπολη

Το Σάββατο 1 7 Ιουλίου 2004 έγινε η Τελετή Λήξης της 45ης Διεθνού ς Μαθηματική ς Ολυμπιάδας, στο Μέγαρο Φίλων της Μουσικής των Αθηνών

Η αίθουσα του Μεγάρ ου των φίλων Μουσικής Αθηνών κατά την τελετή λήξης. Στην πρώτη σειρ ά διακρ ίνονται η Υπου ρ ­ γός Παιδείας κ. Μα ρ ιέττα Γιαννάκου, ο Νομάρχης Αθηνών κ. Ιω άννη ς Σγου ρ ός, ο κ. Κωνσταντίνος Τσαμαδιάς, Γενικός Γ ραμματέας Επιμό ρ φωσης Ενηλίκων, και ο Πρ όεδ ρ ος της 45ης ΙΜΟ και Π ρ όεδ ρ ος της ΕΜΕ κ. Νικόλαος Αλεξανδ ρής

Στην τελετή παραβρέθηκαν ως εκπρόσωπος του Πρωθυπουργού

κ.

Κωνσταντίνου

Καραμανλή ,

η

Υπουργός Εθνικής Παιδείας και Θρησκευμάτων κ. Μαριέττα Γιαννάκου, ο Νομάρχης Αθηνών κ. Ιωάν­ νης Σγουρός, ο Γενικός Γραμματέας Εκπαίδευσης Ενηλίκων του Υπουργείου Παιδείας κ. Κωνσταντί­ νος Τσαμαδιάς, ο Ειδικός Γραμματέας του Υπουρ­ γείου Παιδείας κ. Αθανάσιος Κυριαζής και ο Πρόε­ δρος του Εθνικού Ιδρύματος Νεότητας κ. Αντώνιος Αντωνάκος. ·

Χαιρετισμούς απεύθυναν η Υπουργό ς Παιδείας, ο Νομάρχης Αθηνών, ο Πρόεδρο ς της Διεθνούς Επιτροπή ς Διοργάνωσης των Μαθηματικών Ολυμπιάδων Καθηγη­ τής κ Joszef Pelikan, ο οποίος συνεχάρη την Ελληνική Μαθηματική Εταιρεία για την άψογη διοργάνωση της Ολυμπιάδας, και ο Πρόεδρος της Οργανωτικής Επιτρο­ πή ς της 45 ης Διεθνούς Μαθηματική ς Ολυμπιάδας και Πρόεδρος της Ελληνική ς Μαθηματικής Εταιρείας Καθη­ γητής Νικόλαος Αλεξανδρής. EYΚ.\E I.lliΣ

Η Υπου ργός Παιδείας κ. ' Ιαριέττα Γ ιαννάκο υ στη'· ομύiα τη,; κατά την τελετή λήξης

ο · ί.ιι' τ.υ4 ι


45" Διεθνής Μαθηματική Ολυμπιάδα

Ακολούθησε η απονομή των μεταλλίων στους μαθητές που διακρίθηκαν. Εδόθησαν συνολικά 243 μετάλλια, 45 χρυσά, 78 αργυρά και 1 20 χάλκινα Η απονομή έγινε από την Υπουργό Παιδείας, το Νομάρ­ χη Αθηνών, Γενικούς Γραμματείς των Υπουργείων, τα μέλη του ΙΜΟ ΑΒ, επιφανή μέλη της ελληνικής ακαδημαϊκής κοινότητας προέδρους των επιτροπών της 45ης Δ.Μ.Ο και μ�λη του Δ.Σ. της ΕΜΕ. Συγκεκριμένα τα χρυσά μετάλλια απένειμε η Υπουργός Παιδείας κ. Μαριέττα Γιαννάκου και ο Νο­ μάρχης Αθηνών κ. Ιωάννης Σγουρός.

Η Υπουργός Παιδείας κ. Μαριέττα Γιαννάκου απονέμει μετάλλια

Ο Νομάρχης Αθηνών κ. Ιωάννης Σγουρός απονέμει μετάλλια

Αργυρά μετάλλια απένειμαν τα μέλη του ΙΜΟ ΑΒ Jozsef Pelikan, (Πρόεδρος της ΙΜΟ Adνisory Board), John Webb (Γραμματέας της ΙΜΟ Advisory Board), Titu Andreescu, Fedrico Ardila, Nazar Agakhanov, Hiroshi Nogushi, Νικόλαος Αλεξανδρής (Πρόεδρος του Δ.Σ. της ΕΜΕ και Πρόεδρος της Οργανωτική ς Επιτροπής της 45ης ΙΜΟ), Radmila Bulajich Manfrino, Gregor Dolinar, και ο Αθανάσιος . Φωκάς, (Καθηγητής του Πανεmστημίου του Cambήdge). Χάλκινα μετάλλια απένειμαν οι Καθηγητές Ανδρεαδάκης Στυλιανός, Γαλανής Ευστράτιος (πρώην Πρόεδρος της ΕΜΕ), Δάσιος Γεώργιος, Εξαρχάκος Θεόδωρος ( πρώην Πρόεδρος της ΕΜΕ), Κουνιάς Ευστράτιος, (πρώην Πρόεδρος της ΕΜΕ), Κυριαζής Αθανάσιος (Ειδικός Γραμματέας Ανωτάτης Εκπαί­ δευσης), Λασκαρίδης Κωνσταντίνος, Μπόλης Θεόδωρος (πρώην Πρόεδρος της ΕΜΕ) Παπασταυρίδης Σταύρος, Τσαμαδιάς Κωνσταντίνος, (Γενικός Γραμματέας Εκπαίδευσης Ενηλίκων), τα μέλη του Δ.Σ. της Ε.Μ.Ε. Αναπολιτάνος Διονύσιος, (Αντιπρόεδρος του Δ.Σ. της ΕΜΕ), Αχτσαλωτίδης Χριστόφορος, ΕΥΚΛΕΙΔΗΣ Β' λη ' τ.l/42


45" Διεθνής Μαθηματική Ολυμπιάδα

Βουργάνας Παναγιώτης, Δημάκος Γεώργιος (Αντιπρόεδρος του Δ.Σ. της ΕΜΕ), Ζέρβας Δημήτρης, Καρ ­ κούλιας Γεώργιος, (Γενικός Γραμματέας του Δ.Σ. της ΕΜΕ), Μπαραλής Γεώργιος, Σάλαρης Κωνσταντί­ νος, Φελλούρης Ανάργυρος, Χριστόπουλος Παναγιώτης

Ο π ρόεδρος της Διεθνούς Επιτροπής Κριτών Καθη-yητής του Πανεπιστημίου της Πάτρας κ. Γεώργιος Δάσιος απονέμει μετάλλιο στό μαθητή Μ. Παπαμιχάλη

Την απονομή πλαισίωσαν με την παρουσία τους κοπέλες από το συγκρότημα Ελληνικών χορών της Δώρας Στράτου, ντυμένες με πανέμορφες ελληνικές παραδοσιακές φορεσιές. Στην 45η Διεθνή Μαθηματική Ολυμπιάδα (IMO 2004) συμμετείχαν 85 χώρες με 486 μαθητές. Στη σειρά της βαθμολογίας την πρώτη θέση κατέλαβε η ομάδα της Κίνας, τη δεύτερη η ομάδα των Η.Π.Α. και την τρίτη η ομάδα τη ς Ρωσίας. Η Ελληνική ομάδα κατέλαβε την 26η θέση. Αξίζει να σημειώσουμε ότι στις 1 5 πρώτες χώρες της Ευρωπαϊκής Ένωσης πήρε την 3 η θέση μετά τη Γερμανία και την Αγγλία, ενώ στις 24 χώρες της πρόσφατα διευρυμένης Ευρωπαϊκής Ένωσης που συμμετείχαν κατέλαβε την 5η θέση μετά την Ουγγαρία, Πολωνία, Γερμανία και Αγγλία.

Ο Κωνσταντίνος Τσαμαδιάς απονέμει μετάλλιο

Μέλη του Δ.Σ. της ΕΜΕ απονέμουν μετάλλια

Η εξαμελής Ελληνική ομάδα πήρε πέντε μετάλλια και μια εύφημο μνεία. Συγκεκριμένα Αργυρό με­ τάλλιο πήραν ο Στέργιος Αντωνακούδης και ο Μηνάς Παγωνάκης, Χάλκινο οι Στέφανος Αρετάκης, Μάριος Παπαμιχάλης και Φάνης Παπαμιχάλης και Εύφημο Μνεία ο Αχιλλέας Πορφυριάδης. Αρχη­ γός της ελληνικής ομάδας ήταν ο επίκουρος καθηγητής του ΕΜΠ Ανάργυρος Φελλούρης, με υπαρχηγό τη μαθηματικό Αγγελική Βλάχου. Η τελική προετοιμασία της Ελληνικής ομάδας έγινε τον τελευταίο μήνα πριν τη Διεθνή Μαθηματική Ολυμπιάδα στην Αθήνα από την Επιτροπή Διαγωνισμών και εργάστηκαν aφιλοκερδώς με ιδιαίτερο ζήλο ΕΥΚΛΕΙΔΗΣ Β ' λη ' τ.l/43


45" Διεθνής Μαθηματική Ολυμπιάδα

οι μαθηματικοί Φελλούρης Ανάργυρος, επίκουρος Καθηγητής του ΕΜΠ (πρόεδρος της εmτροπής), Βλά­ μος Παναγιώτης, Βλάχου Αγγελική, Δούναβης Αντώνιος, ΚοντογιάνVης Δημήτριος, Λουρίδας Σωτήριος, Πουλόπουλος Παναγιώτης και Τυρλής Ιωάννης. Η Οργανωτική Επιτροπή σύμφωνα με το πνεύμα του Ολυμmσμού, απένειμε σε όλους τους συμμετέ­ χοντες μαθητές κότινο (στεφάνι αγριελιάς) ως σύμβολο ειρήνης και συναδέλφωσης των λαών μέσα σ' ένα κλίμα συγκίνησης και ενθουσιασμού

Μαθητές παραλαμβάνουν τον κότινο

Ο Πρόεδρος της Οργανωτικής Επιτροπής της 45ης Διεθνούς Μαθηματικής Ολυμπιάδας Καθηγητής του Πανεπιστημίου Πειραιά κ. Νικόλαος Αλεξανδρής παρέδωσε τη σημαία των Διεθνών Μαθηματικών Ολυμπιάδων στην Καθηγήτρια Πανεπιστημίου κ. Radmila Bulajich Manfήno, αρχηγό της αντιπροσωπεί­ ας του Μεξικό, επόμενης διοργανώτριας χώρας (IMO 2005).

Η ομάδα του Μεξικού παραλαμβάνει τη σημαία της ΙΜΟ από τον Πρόεδρο της ΕΜΕ, κ. Νικόλαο Αλεξανδρή Καθηγητή του Πανεπιστημίου Πειραιά

Όλοι οι μαθητές αποχώρησαν από την τελετή έχοντας στο κεφάλι τους το στεφάνι της ελιάς και οι ξένες αποστολές στο σύνολό τους εξέφραζαν τον ενθουσιασμό τους για την υψηλού επιπέδου διοργάνω­ ση και την ελληνική φιλοξενία και είναι σίγουρο ότι θα γίνουν οι καλύτεροι πρεσβευτές της Ελλάδας στις χώρες τους.

ΕΥΚΛΕΙΔΗΣ Β' λη ' τ.l/44


45η Διεθνής Μαθηματική Ολυμπιάδα

Η Υπου ργός Παιδείας κ Μαριέττα Γιαννάκου, ο Ν ομάρχης Αθηνών κ. Ι. ΣΎουρός με τον Πρόεδρο της ΕΜΕ καθη-yητή κ. Ν. Αλεξανδρή και τον Α' Αντιπ ρόεδρο της ΕΜΕ καθη-yητή κ. Γ. Δημάκο

Η 45η Διεθνή ς Μαθηματική Ολυμπιάδα ολοκληρώθηκε με δεξίωση στο Γκολφ της Γλυφάδας όπου συμμετείχαν 1200 άτομα, οι μαθητές και οι συνοδοί τους, μέλη των επιτροπών που συμμετείχαν στη διοργάνωση τη ς Ολυμπιάδας , συνάδελφοι μαθηματικοί, Βουλευτές, γενικοί γραμματείς Υπουργείων, Νομαρχιακο ί Σύμβουλοι, μέλη της ακαδημαϊκή ς κοινότητας, καθηγητές του Πανεπιστημίου Αθηνών, του Πανεπιστημίου Πειραιά και του Εθνικού Μετσόβιου Πολυτεχνείου Αθηνών.

Δεξίωση στο Γκολφ της Γλυφάδας

Τις δύο επόμενες ημέρες, Κυριακή 1 8 Ιουλίου και Δευτέρα 1 9 Ιουλίου αναχώρησαν τμηματικά οι αποστολές, με τις καλύτερες εντυπώσεις.

ΧορηΎοί της 45ης •

• •

Διεθνούς Μαθηματικής Ολυμπιάδας,

Εκτός από

Το Υπουργείο Εθνικής Παιδείας και Θρησκευμάτων Το Υπουργείο Πολιτισμού Την Νομαρχία Αθηνών

που ανέλαβαν τη χρηματοδότηση τη ς Ολυμπιάδας ,συνέβαλαν ουσιαστικά στην επιτυχία της διοργάνω ­ σης με τις προσφορές τους και οι παρ ακάτω: ΕΥΚΛΕΙΔΗΣ Β ' λη ' τ.l/45


45η Διεθνής Μαθηματική Ολυμπιάδα • •

• •

• • •

Α Έφορεία Αρχαιοτήτων Ακρόπολης Αλουμίνιο της Ελλάδος Α γροτικός Συνεταιρισμός Σητείας ALMME Ένωση Συνεταιρισμών Βέροιας ASPIS S.A., Άργος ΑΤΕΡΑΡ S.A., Ιεράπετρα Τράπεζα της Ελλάδος Βαρβαγιάννης Οινοποιία Πλωμάρι Λέσβου Εμπορικό Επιμελητήριο Κορινθίας Ένωση Μαστιχοπαραγωγών Χίου Coca Cola Ελλάς Α . Ε.

• •

• •

Νομαρχία Κορινθίας Υπηρεσία συντήρησης της Ακρόπολης, Τμήμα εκπαιδευΠεριφέρεια Ανατολικής Μα­ τικών προγραμμάτων κεδονίας και Θράκης ΕΝΟΑΜ Ένωση Οινοποιών Σπυρόπουλος Οινοποιία, Αρ­ Βέροιας καδία ΕΡΟΜ Οινοποιία Μυτιλήνης Ποτοποιία Θράκης Ι. Μπουτάρη Οινοποιία Ένωση Venus, Βέροια Αγία Μονή Τοπλού, Σητεία Βλάμος Φροντιστήρια, Μο­ σχάτο Μαυράκης Οινοποιία, Άργος ΜΕΒΓΑΛ Α. Ε. Καθώ ς και οι Χο ρηγοί Επικοι­ Δήμος Άμφισσας νωνίας Δήμος Αράχωβας Δήμος Δελφών Εφημερίδα «ΤΑ ΝΕΑ» Δήμος Λιβαδειάς ALPHA Τηλεόραση ΝΟΗΣΗ Φροντιστήρια, ΜαALPHA Ράδιο 98, 9 ρούσι Νομαρχία Βοιωτίας •

Ο σχεδιασμός, η διοργάνωση και η επιτυχημένη πραγματοποίηση της 45ης Διεθνούς Μαθηματική ς Ολυμπιάδ ας, είναι αποτέλεσμα συλλογική ς, ένθερμης προσπάθειας και συνεργασίας του Προεδρείου, των μελών του Διοικητικού Συμβουλίου και προεδρείων των παραρτημάτων της Ελληνική ς Μαθηματι­ κής Εταιρείας. Κατά την προετοιμασία και τη διεξαγωγή της 45 ης ΙΜΟ ήταν σημαντική η εθελοντική συ­ νεισφορά μελών της ΕΜΕ, τόσο από την έδρα της όσο και από τα παραρτήματα καθώ ς και η συμμετοχή φοιτητών που εθελοντικά πλαισίωσαν το κλίμα της θερμής ελληνικής φιλοξενίας μας, ώστε όλοι οι συμ­ μετέχοντες να αποκομίσουν εξαιρετικές εντυπώσεις.

ΛΝΛΚΟΙΝΩΣΗ

Οι διαγωνισμοί της Ελληνικής Μαθηματικής Εταιρείας θα διεξαχθούν: «0 Θαλής>> 30 Οκτωβρίου 2004 « 0 Ευκλείδης» 18 Δ εκεμβρίου 2004 «0 Αρχιμήδης» 12 Φεβρουαρίο υ 2005

ΕΥΚΛΕΙΔΗΣ Β ' λη ' τ.l/46


Μαθηματικά Β' Λυκείου

(συνέχεια από τη σελ. 32) �5 + Jϊ8 < J3 + J2 ή 5 + Jϊ8 < 3 + 2 + 2J6 ή Jϊ8 < J24 που ισχύει. α 2 = 5 + Jϊ8 , αρα α 2 > β 2 + γ 2 που Έχουμε β2 + γ2 = 5 σημαίνει ότι τα άρρητα μήκη β = J3 και γ = J2 όπως είναι γνωστό κατασκευάζονται, θα πρέπει να κατασκευάσουμε το α = �5 + Jϊ8 . Είναι α = �3 + 2 + 3 J2 = JJ3 2 + J2 2 + 3 J2 . Θέτουμε: χ 2 = J3 2 + J2 2 (το χ θα είναι η υπο­

στε να ισχύει ΜΑ 2 - ΜΒ 2 = 3R 2 (1). α) Να βρεθεί το μήκος της προβολής ΝΗ της διαμέσου ΜΝ του τριγώνου ΜΚΛ. β) Να αποδείξετε ότι η ΝΗ είναι εφαπτομέ­ νη του κύκλου (Λ, ρ).

}

τείνουσα ορθογωνίου τριγώνου με κάθετες J3 και J2 ) και y 2 = 3 J2 (το y θα είναι η μέση ανάΝ:ηος των τμημάτων με μήκη 3 και J2 ). Επομένως α 2 = χ 2 + y 2 που είναι η υποτείνου­ σα ορθογωνίου τριγώνου με κάθετες πλευρές που έχουν μήκη χ, y. Το τρίγωνο ΑΒΓ τώρα κατασκευάζεται. β) Από το Θεώρημα των συνημίτονων έχουμε ή α 2 = β 2 + γ 2 - 2βγσυνΑ α2 = 3 + 2 - 2 J6συνΑ = 5 - 2 J6συνΑ . Όμως α2 = 5 + Jϊ8 άρα είναι J3 οπότε -2 J6συνΑ = Jϊ8 ή συνΑ = 2 4.

Δίνονται οι κύκλοι (Κ, R) και (Λ, ρ) με R > 2ρ όπου ΚΛ = 2R+ρ. "

α) Τα τρίγωνα ΜΑΚ, ΜΒΛ είναι ορθογώνια οπό­ τε είναι: ΜΑ 2 = ΜΚ 2 - R 2 και ΜΒ2 = ΜΛ2 - ρ 2 (Πυθαγ. θεώρ.) άρα ΜΑ 2 - ΜΒ 2 = ΜΚ 2 - ΜΛ2 - R 2 + ρ 2 ή ΜΚ 2 - ΜΛ2 = 4R 2 - ρ 2

ι ''

I

\

Από (2) , (3) προκύπτει ότι: 2(2R + ρ) · ΝΗ = 4R 2 - ρ 2 ή 2(2R + ρ) · ΝΗ = (2R - ρ)(2R + ρ) οπότε 2R ρ ΝΗ = - δηλαδή NH = R - . 2 β) Για να δείξουμε ότι ΝΗ εφαπτομένη του κύ­ κλου (Λ, ρ) θα πρέπει ΛΗ = ρ διότι ΜΗ .l ΚΛ . Είναι 2R + ρ ΛΗ=ΝΛ-ΝΗ = R- ΝΗ = 2 δηλαδή ΛΗ = R + Ε_ - R + Ε_ = ρ . 2 2

/{

κ;

5.

' ' ' I

(2).

Από το 2° θεώρημα της διαμέσου στο τρίγωνο ΜΚΛ έχουμε ΜΚ 2 - ΜΛ2 = 2ΚΛ · ΝΗ ή (3). ΜΚ 2 - ΜΛ2 = 2(2R + ρ) · ΝΗ

Μ

I

I I I I I ι

Λύση

( !{)

Σε τρίγωνο ΑΒΓ (β > γ) φέρουμε τη διάμε­ σο ΑΜ και έστω Θ το βαρύκεντρο του. Στα Α, Θ φέρουμε κάθετες ευθείες στην ΑΜ, που τέμνουν την ευθεία ΒΓ στα Κ και Ρ α­ ντίστοιχα. Να υπολογιστούν τα τμήματα ΚΜ και ΡΜ.

' '

I

Λύ ση

'

I

'

I

' '

I I

'

Α

Ν

κ

Από σημείο Μ του επιπέδου φέρνουμε τα εφαπτόμενα τμήματα ΜΑ και ΜΒ έτσι ώ-

Ρ Β Δ

Μ

Γ

Φέρουμε το ύψος ΑΔ του τριγώνου ΑΒΓ οπότε

ΕΥΚΛΕΙΔΗΣ Β' λη ' τ.l/47


Μαθηματικά Β' Λυκείου

από το ορθογώνιο τρίγωνο ΑΜΚ έχουμε

2 ΑΜ 2 = ΜΔ · ΜΚ ή ΜΚ = � ΜΔ 2β 2 + 2γ 2 α 2 Είναι μ � = (2) και 4 β 2 - γ 2 = 2 α . ΔΜ (2° θεώ ρημα διαμέσου) β2 - γ άρα ΔΜ (3). =

(1)

2

ε

β) Είναι ΡΑ 2 = 0Ρ2 - R 2 (3) όμως από το ορθο­

Από ( 1 ), (2), (3) προκύπτει ότι 2 2 + 2γ 2 - α 2 ΜΚ = � · β (4). 2 β 2 - γ2 •

γώνιο τρίγωνο ΟΡΗ έχουμε

Τα ορθογώνια τρίγωνα ΜΡΘ και ΜΑΚ είναι όμοια (αφού ΘΡ//ΑΚ) άρα

ΜΡ ΜΘ 1 , ς -- = -- = - επομενω ΜΚ ΜΑ 3

Επομένως ΡΑ = R J7 . 7.

ΜΡ = .!_ ΜΚ και από την (4) έχουμε 3 2β 2 + 2γ 2 - α 2 ΜΡ = -α · 6 β 2 - γ2

Δύο κύκλοι (Κ, 8) και (Λ, 6) τέμνονται στα Α, Β έτσι ώστε η γωνία ΚΑΛ = 90° . Αν η διάκεντρος ΚΑ τέμνει τον ι κύκλο C1 στο Ρ και τον 2° κύκλο C2 στο Μ να υπολογι­ στεί το τμήμα ΜΡ. Λ

0

-=------:---=--..,...--

6.

ΟΡ 2 = ΟΗ 2 + ΡΗ 2 οπότε η (3) γίνεται ΡΑ 2 = ΟΗ 2 + ΡΗ 2 - R 2 ή ΡΑ 2 = ( R J5 ) 2 + ( R .J3 ) 2 - R 2 ή ΡΑ2 = 7R2 •

Δίνεται κύκλος (0, R) και ευθεία ε που α­ πέχει από το κέντρο Ο απόσταση ΟΗ = R.JS . Έστω Ρ μεταβλητό σημείο της ευθείας ε και ΡΑ, ΡΒ τα εφαπτόμενα τμή­ ματα του κύκλου. Αν η χορδή ΑΒ τέμνει την ευθεία ΟΗ στο Ε, να αποδειχτεί ότι: α) ΟΕ = -ι ΟΗ 5 β) ΡΑ = RJ7 όταν ΡΗ = RJ3 .

Λύ ση

Λύ ση

α)

Αν η ΟΡ τέμνει την ΑΒ στο Κ τότε ΟΡ .l ΑΒ . Επομένως το τετράπλευρο ΡΚΕΗ θα είναι εγγράψιμο άρα ΟΕ · ΟΗ = ΟΚ · ΟΡ ( I ) . Επειδή το τρίγωνο ΑΟΡ είναι ορθογώνιο στο Α και ΑΚ το ύψος του άρα

ΟΚ · ΟΡ = ΟΑ2 ή OK · OP = R 2 (2).

οπότε είναι

Από ( 1 ), (2) έχουμε ΟΕ · ΟΗ = R 2 ή

OE · R J5 = R 2 ή ΟΕ =

R J5 5

Από το Πυθαγόρειο θεώρημα στο τρίγωνο ΑΚΛ έχουμε ΚΛ2 = 8 2 + 6 2 ή ΚΛ2 = 100 ή ΚΑ = 10. Η δύναμη του Κ ως προς τον κύκλο C 2 είναι Δ�, = ΚΜ · ΚΝ = ΚΑ 2 ή ΚΜ(ΚΜ + ΜΝ) = 8 2 ή ή ΚΜ(ΚΜ + 1 2) = 64 θέτουμε ΚΜ = χ

ή ΟΕ =

ΟΗ 5

.

χ 2 + 12χ - 64 = 0 <::> χ = 4 ή χ = -16 (απορ.). Άρα ΚΜ = 4, επομένως ΜΡ = ΚΡ - ΚΜ = 8 - 4 = 4 .

ΕΥΚΛΕΙΔΗΣ Β' λη ' τ.l/48


Μαθηματικά Β ' Λυκείου

8.

Δίνεται τρίγωνο ΑΒΓ εγγεγραμμένο σε κύ­ κλο (0, R). Φέρουμε τη διάμεσο του ΑΜ και τη διάμετρο ΑΔ. Έστω Ρ η προβολή του Μ στην ΑΔ να δεχτεί ότι α) β 2 + γ 2 = 4R · AP , β) β 2 + γ 2 = 2ΑΜ · ΑΚ όπου Κ το σημείο τομής της ΑΜ με τον κύκλο.

που ισχύει διότι το τετράπλευρο ΜΡΔΚ είναι εγγράψιμο σε κύκλο, αφού Ρ = 90° και Λ Κ = 90° ως εγγεγραμμένη σε ημικύκλιο. Προτεινόμενες για λύση 1.

Να αποδείξετε ότι τα βαρύκεντρα των τεσσά­ ρων τριγώνων που ένα τετράπλευρο χωρίζεται από τις διαγώνιές του, είναι κορυφές πα­ ραλ/μου.

2.

Σε τρίγωνο ΑΒΓ είναι Α = ι 20° . Αν ΑΔ είναι μια διχοτόμος του να αποδείξετε ότι:

Λύ ση

Λ

ι

α) Εφαρμόζουμε στο τρίγωνο ΑΒΓ το ι ο θεώρημα

3.

α)

Δίνεται τρίγωνο ΑΒΓ και τα τμήματα ΑΔ, ΒΕ, ΓΖ που συντρέχουν στο Κ. Να απο­ δείξετε ότι

( 1 ).

Έστω ότι η γωνία ΑΟΜ είναι αμβλεία τότε στο τρίγωνο ΑΟΜ έχουμε ΑΜ 2 = 0Α 2 + 0Μ 2 + 20Α · ΟΡ (2) (θεώρημα αμβλείας γωνίας) οπότε η ( ι ) γί­ νεται β 2 + γ 2 = 20Α 2 + 20Μ 2 + 40Α · ΟΡ + 2ΒΜ 2 = 2R 2 + 2(0Μ 2 + ΒΜ 2 ) + 4R · ΟΡ = 2R 2 + 2R 2 + 4R · OP = 4R 2 + 4R · OP = 4R(R + OP) = 4R · AP ( ΟΜ 2 + ΒΜ 2 = R 2 αφού το τρίγωνο ΟΜΒ είναι ορθογώνιο στο Μ) Λ

Αν η γωνία ΑΟΜ είναι οξεία το Ρ θα ανήκει στο τμήμα ΑΟ και θα ισχύει ΑΜ 2 = ΟΑ,. 2 + ΟΜ 2 - 20Α · ΟΡ οπότε θα έχουμε β 2 + γ 2 = 4R(R - ΟΡ) = 4R · ΑΡ δηλαδή καταλήγουμε επίση ς στην ίδια σχέση.

β)

ι

ΑΔ

κ

της διαμέσου και έχουμε β 2 + γ 2 = 2ΑΜ 2 + 2ΒΜ 2

ι

-+- =-. ΑΒ ΑΓ ΑΔ ΑΔ (Υποδ. αρκεί + =1) ΑΒ ΑΓ

Σύμφωνα με το α) πρέπει να αποδείξουμε ότι 2ΑΜ · ΑΚ = 4R · ΑΡ � ΑΜ · ΑΚ = 2R · ΑΡ � � ΑΜ · ΑΚ = ΑΔ · ΑΡ

ΔΒ . ΕΓ . ΖΑ = 1 . ΔΓ ΕΑ ΖΒ Να διατυπωθεί και να αποδειχτεί το αντί­ στροφο. (Θεώρημα Ceva).

β)

Να αποδείξετε ότι ο διάμεσοι και τα ύψη τριγώνου συντρέχουν. (Υποδ. α) από τις κορυφές Β, Γ φέρουμε πα­ ράλληλες προς την ΑΔ, β) με βάση το αντί­ στροφο του α) ). 4.

Σε τραπέζιο ΑΒΓΔ (ΑΒ > ΓΔ και ΑΒ//ΔΓ) παίρνουμε τα σημεία Ρ, Σ και ΑΔ, ΒΓ αντί­ στοιχα τέτοια ώστε να ισχύει ΑΡ ΒΣ 3 -=-=ΡΔ ΣΓ 4 Η ΡΣ τέμνει τις ΑΓ, ΒΔ στα Κ, Λ αντίστοιχα, να αποδείξετε ότι: 4ΑΒ + 3ΓΔ

α)

ΡΣ =

β)

ΡΚ = ΛΣ και ΚΑ =

ΕΥΚΛΕΙΔΗΣ Β ' λη ' τ.l/49

7

4ΑΒ - 3ΓΔ

7

.


Μαθηματικά Β' Λυκείου

ΡΑ ΡΔ 4 , (Υποδ. α) ειναι ΡΣ = ΡΛ + ΛΣ,- = - = ­ ΑΒ ΑΔ 7 ΛΣ 3 ). και - = ΔΓ 7 5.

Δίνεται ισοσκελές τρίγωνο ΑΒΓ (ΑΒ και το ύψος του ΓΕ. Να αποδείξετε ότι α 2 = 2γ · ΒΕ .

=

ΑΓ)

(Υποδ. Προεκτείνουμε ..... ) 6.

7.

Από σημείο Ρ εκτός κύκλου (0, R) φέρουμε δύο κάθετες μεταξύ τους τέμνουσες του κύ­ κλου στα σημεία Α, Β, Γ και Δ. Να αποδείξε­ τε ότι ΟΑ 2 + 0Β 2 + 0Γ 2 + 0Δ2 = 4R 2 (Υποδ. Από το Α φέρουμε χορδή ΑΕ/ΙΓΔ οπότε ΑΓ = ΔΕ). Αν σε τρίγωνο ΑΒΓ ισχ6ει β 4 = 2(α4 + γ4 ) να αποδείξετε ότι η γωνία Β � 90° .

8.

Να υπολογιστούν οι πλευρές τριγώνου ΑΒΓ αν οι διάμεσοι έχουν μήκη �. �. μγ. (απ. α 2 = .± (2μ � + 2μ � - μ ; ) . 9

9.

Δίνεται τρίγωνο ΑΒΓ (β > γ) και τα σημεία Δ, Ε της ΒΓ τέτοια ώστε ΒΔ = ΔΕ = ΕΓ . Να α­ ποδείξετε ότι: α) ΑΒ 2 + 3ΑΕ 2 = ΑΓ 2 + 3ΑΔ2

β)

Το τρίγωνο ΑΒΓ είναι ορθογώνιο αν 4α 2 ΑΓ 2 + 3ΑΔ2 = . 3 (Υποδ. ι ο Θεώρημα διαμέσου). 10. Σε κάθε τρίγωνο ΑΒΓ να αποδείξετε ότι:

α2 α) μ ; + βγ > - > μ; - βγ 4 β) 3(α 2 + β 2 + γ 2 ) � 4 ( υ ; + υ � + υ� ) .

11. Δίνεται τ ρίγωνο ΑΒΓ εγγεγραμμένο σε κύκλο

(0, R). Έστω ΑΜ η διάμεσος του που τέμνει τον κύκλο στο Ρ και Θ το βαρύκεντρο του τριγώνου. Να αποδείξετε ότι: α2 α) ΜΑ · ΜΡ = 4 ι e β) �(O, R ) = - 9 (α 2 + β 2 + γ 2 ) . 12. Στη διάμετρο ΑΒ κύκλου (0, R) παίρνουμε

σημείο Γ. Έστω Σ σημείο του κύκλου. Φέ­ ρουμε χορδή ΣΜ .l ΣΓ . Να αποδείξετε ότι: α) Η ευθεία ΜΣ ' όπου Σ ' το aντιδιαμετρικό του Σ, διέρχεται από σταθερό σημείο Ρ. β) Το γινόμενο ΡΜ · ΓΣ είναι σταθερό. (απ. α) Ρ είναι το συμμετρικό του Γ ως προς το Ο, β) ΡΜ · ΓΣ = Δ�ο. R > ).

r Γεωμετρ ικ ά συμπερ άσματα σε τρ ίγω νο ό ταν δύο γω ν ίε ς του δ ι α φ έρ ου ν κ ατ ά 90 °

Γιάννη ς Στρατήγη ς Παρουσιάζουμε με πέντε ασκήσεις, σχέσεις τμημάτων και γωνιών σε τρίγωνο ΑΒΓ στο οποίο είναι = 90° . Για εμπέδωσή τους δίνουμε μια υπολογιστική άσκηση. Στην απόδειξή τους χρησιμοποιούμε όλη την ύλη της επιπεδομετρίας. Μπορούμε με κατάλληλους συνδυασμούς να βρούμε και άλλους τρόπους απόδειξης.

Β-r

ΕΥΚΛΕΙΔΗΣ Β ' λη ' τ.l/50


Μαθηματικά Β' Λυκείου

στο ορθογώνιο τρίγωνο ΗΒΔ έχουμε Λ Λ Λ Η1 = 90° - ΔΒΗ = 90° - ΕΒΓ , γιατί ΔΒΗ . ΕΒΓ

Λυμένες Ασκήσεις 1)

Σε τρίγωνο ΑΒΓ με Β- Γ = 90° , να δείξετε ότι ΑΑΒ = 45° , όπου ΑΔ διχοτόμος. Λύση

είναι κατακορυφήν και εφόσον στο ορθογώνιο Λ Λ τρίγωνο ΕΒΓ είναι ΕΒΓ = 90° - Γ προκύπτε ι Λ

Λ

Λ

Η1 = 90° - (90° - Γ) = Γ

Α

Λ

Λ

(2).

Από τις σχέσεις ( 1 ), (2) ισχύει Α1 = Η1 , που σημαίνει ότι το τρίγωνο ΑΒΗ είναι: ii) Λ

Λ

Γ

Λ

Στο τρίγωνο ΑΒΓ είναι Α = 1 80° - Β - Γ , αλλά Λ Λ από την υπόθεση έχουμε Β = 90° + Γ , οπότε Λ

Λ

Λ

Α = 1 80° - (90° + Γ) - Γ = 90° - 2 Γ ,

ίίί) Διαπιστώσαμε ότι το σημείο Γ ανήκει στην με­

Λ

Λ Α άρα - = 45 ° - Γ ( 1 ). 2

Στο τρίγωνο μένως

ΑΓ Δ η

σοκάθετη του τμήματος ΑΗ, άρα ΓΑ = ΓΗ, δη­ λαδή το τρίγωνο ΗΓΑ είναι ισοσκελές.

Λ

ΑΔΒ είναι η εξωτερική επο-

3)

Λ

Λ Λ Λ Α Λ (1) ΑΔΒ = - + Γ =(45 ° - Γ) + Γ , 2 δηλαδή ΜΒ = 45 ° .

2) Αν σε τρίγωνο ΑΒΓ είναι Β- Γ = 90° και Η το ορθόκεντρό του, να αποδείξετε ότι: i) ΔΑ = ΔΗ, αν ΑΔ το ύψος του ii) ΑΗ 2 = 4 · ΔΒ · ΔΓ , iii) Το τρίγωνο ΗΓΑ είναι ισοσκελές. Λύση

Λ

γιατί από την υπόθεση

Β- 90° = Γ

.

( 1 ),

Είναι ΔΑΒ = 90° - ΑΒΔ = 90° - (1 80° - ΑΒ Γ) = = ΑΒΓ - 90° = Γ , που δηλώνει ότι η ευθεία ΑΔ είναι εφαπτομένη του περιγεγραμμένου κύκλου του τριγώνου ΑΒΓ, οπότε για το εφαπτόμενο τμήμα ΔΑ και την τέμνουσα ευθεία ΔΓ έχουμε ΔΑ 2 = ΔΒ · ΔΓ

Από το ορθογώνιο τρίγωνο ΑΒΔ είναι Α1 = 90° - ΑΒΔ . Οι γωνίες όμως ΑΒΔ και ΑΒΓ είναι παραπληρωματικές οπότε Αι = 90° - (1 80° - ΑΒΓ) = ΑΒΓ- 90° = Γ

Θεωρούμε τρίγωνο ΑΒΓ με Β- Γ = 90° . Δείξατε ότι: i) ΑΔ2 = ΔΒ · ΔΓ , όπου ΑΔ το ύψος του (Πανελλήνιες Β ' ΛΥΚΕΙΟΥ 1981). ίί) ΑΒ 2 + ΑΓ 2 = 4R 2 , όπου R η ακτίν� του περιγεγραμμένου του κύκλου. Λύ ση

i)

i)

Ισοσκελές με ΒΔ ύψος του, άρα και διάμεσός του, οπότε ΔΑ = ΔΗ. Από την ομοιότητα των ορθογωνίων τριγώνων ΑΒΔ και ΓΑΔ προκύπτει: ΑΗ 2 = (2ΑΔ ) 2 = 4 · ΔΒ · ΔΓ (γιατί ; )

ii)

Φέρουμε ΑΕ .l ΑΔ και αφού ΑΔ .l ΔΓ θα εί­ ναι ΑΕ//ΒΓ, οπότε

Εξάλλου

ΕΥΚΛΕΙΔΗΣ Β' λη ' τ.l/51


γ2 + β2 -γ2 2 = 2 ( β2 2α-γ2 )2 ή β2 +γ2 = 2 (β2 -γ2 ) , 2 2 β2 4α22 = β2 2 2 δηλαδή α ( + γ ) ( -γ ) .

Μαθηματικά Β' Λυκείου

ΑΓΒ = ΕΑΓ ή ΑΒ = ΕΓ ή ( 1 ). ΑΒ = ΕΓ Εξάλλου ΒΓΕ = ΑΒΓ , επειδή είναι εγγεγραμμένες που βαίνουν σε ίσα τόξα, επομένως ΑΓΕ = BrE-BrΑ = ΑΒΓ-BrΑ = 90° . Με εφαρμογή του πυθαγορείου θεωρήματος στο2ορθογώνιο τρίγωνο ΑΓΕ προκύπτει ότι ΑΓ + ΓΕ2 = ΑΕ2 ή ΑΓ2 + ΑΒ2 = ΑΕ2 = (2R)2 ή ΑΒ2 +ΑΓ2 = 4R 2 . Αν σε τρίγωνο ΑΒΓ ισχύει Β- r = 90° , τότε 2 2 2 2 2 Λ

Λ

..-...

Λ

Λ

4)

...-..

είναι (β - γ ) = α (β + γ ) . Λύση Α

�Γ Έστω ΑΔ, ΑΜ το ύψος και η διάμεσος του τριγώ­ Δ

Β

Λ

Λ

= Β-r 90° , δείξατε ότι: Σε τρίγωνο ΑΒΓ /β2 +γ2 i) ΜΔ 22 β2 = ' γ =-r='=== ii) ΒΔ = ΓΔ ' �β2 +γ2 �β2 +γ2 iii) ΒΕ 2 +ΓΕ 2 = ( β -γ/ , αν ΑΔ, ΑΜ, ΑΕ είναι ύψος, διάμεσος και διχοτόμος, αντίστοιχα. ν

5)

>

Λ

(ι)

= δα = δα' = � α=

Λ

Λ

α •

Λ

>

Αν δα, δα' είναι η εσωτερική και η εξωτερι­ κή διχοτόμος ενός τριγώνου ΑΒΓ με Β - Γ 90° , που αντιστοιχούν στην Α , τότε: i) J2 βγ , β2 + γ 2 ii) δ J2υ Λ

Μ

νου ΑΒΓ, αντίστοιχα. Είναι Β = 90° + Γ 90° , δηλαδή η Β είναι αμβλεία, οπότε Β Γ , οπότε β γ και από το θεώρημα των διαμέσων στο τρίγωνο ΑΒΓ έχουμε: β2 -γ2 = 2α·ΜΔ = 2α(ΜΒ+ΒΔ) = 2α (Ξ +ΒΔ ) ή β2 -γ2 - � = ΒΔ ( 1 ). 2α 2 Στην άσκηση (3i) δείξαμε ΑΔ2 = ΔΒ · ΔΓ , αλλά από το πυθαγόρειο θεώρημα στο ορθογώνιο τρί­ 2 2 = 2 γωνο ΑΒΔ είναι ΑΔ γ -ΒΔ , επομένως γ2 -ΒΔ2 = ΔΒ·ΔΓ ή γ2 -ΒΔ2 =ΔΒ· (ΒΔ+α) και λόγω της σχέσης2 γράφουμε γ2 - ( β22α-γ2 - �2 ) = ( β2 2α-γ2 �2 ) ·.( β2 2α_ γ2 �2 +α) 2 -γ2 = ή γ2 - ( β2 2α-γ2 )2 - ί4αJ + .i ft2 _ βjJ = ( βΊ�γ' )' - i ή Λ

*Πρ ο τ ειν όμενη Ά σκηση

Λύ ση

>

i)

Αν η διχοτόμος ΑΔ τέμνει τον περιγεγραμμένο κύκλο του τριγώνου ΑΒΓ στο Ε θα είναι ΑΒΔ ::::: ΑΕΓ (επειδή ΑΒΔ = ΑΕΓ και Α συνεπώς -γ = - ή ΒΑΔ = ΕΑΓ = -), ΑΕ β 2 γ = ΑΔ ή βγ = ΑΔ2 +ΑΔ·ΔΕ ( 1 ). ΑΔ+ΔΕ β Για τις τεμνόμενες χορδές ΑΕ, ΒΓ του κύκλου στο Δ είναι ΑΔ ·ΔΕ = ΒΔ·ΓΔ = _β+!!Υ_·� γ β +γ Δ

Δ

Λ

Λ

ΕΥΚΛΕΙΔΗΣ Β ' λη ' τ.V52

Λ

Λ

Λ

ΑΔ


Μαθηματικά Β' Λυκείου

δηλαδή ΑΔ ·ΔΕ = α2 (β+βγγ)2 (για τα τμήματα ΔΒ, ΔΓ που χωριζει η διχοτόμος της πλευράς ΒΓ) και αν λάβουμε υπόψη μας την σχέση που αποδείξαμε στην άσκηση 4 έχουμε ΑΔ ΔΕ = (ββ22 -γ+γ22) 2 Από τη σχέση ( 1 ) έχουμε ΑΔ2 βγ-ΑΔ ·ΔΕ , επομένως ·

....:.____.;. ..., --,--­

=

Ισχύ ει υ ρηση Παρατή α

=

βγ �β 2 γ2 +

Άσκηση

Σε τρίγωνο ΑΒΓ με Β- r = 90° είναι ΑΒ = lm και ΑΓ = 7m. Να υπολογίσετε τα μήκη της πλευράς ΒΓ, της εσωτερικής διχοτόμου ΑΔ, της εξωτερικής διχοτόμου ΑΔ , του ύψους ΑΕ και της ακτίνας του περιγεγραμμένου του κύκλου, καθώς και τα τμήματα ΕΒ, ΕΓ. '

Α πάντηση

α = 245J2 m,δα = δα' = -75 m, υα = 7J22 m, J2 5 J2 Ε 49 J2 m' Β = m m ΕΓ R= 2

Από την άσκηση 1 είναι ΜΒ = 45° και ΔΜ ' = 90° (γωνία διχοτόμων δύο εφεξής και παραπληρωματικών 'γωνιών), άρα Μ'Δ = 45° και το τρίγωνο ΑΔΔ είναι ισοσκελές με Το τρίγωνο ΑΒΓ με Β- r = 90° ονομάζεται ψευ­ ΑΔ ΑΔ ' δηλαδή . δορθογώνιο β γ J2 δ = δ = �β2 +γ2 Με το πυθαγόρειο θεώρημα στο ορθογώνιο και ' ισοσκελές τρίγωνο ΑΕ Δ έχουμε 2 '2 = ΑΔ2 --

--

ιο

'

=

ιο

Ση μείωση

=

α

ii)

α

.

ΑΕ

Όλη η ύλη τη ς

Άλγεβρας της Α1 Λυκείου

σε μαθήματα

ανά σχολική διδακτική ώρα . Κάθε μάθ η μα περιλαμβάνει :

1.

2.

,.... nu ι-ια .,... na

3.

4.

μ:aOrιtrt καθηγηι:r'} υποψήφιο tου ΛΣΕΙl :..- rl« ro φοιrηιή tOU t;oμ{a ΔΙδCΙΚCΙΙ<Ι]S ιωv ΜaJημaτικών Jr"

ω

ιuv

5.

rov

6.

Σχέδιο Μαθήματος . Φύλλο Εργασίας . Ασκήσεις εμπέδωση ς και ελέγχου γνώσεων . Εργασία γιο το σπίτι . Παρουσίαση του μαθήματος - σχόλιο . Ασκήσεις .

Στο βιβλίο περιέχονται ακόμα διαγωνίσματα και αναλυτικές λύσεις των φύλλων εργασίας και

ΓΑΡΑΤΖΙΩΤΗΣ - ΜΑΣ'rΑΚΑΣ

των ασκήσεων.

Ά λγεβρα Α · Λυ κείου

ΕΥΚΛΕΙΔΗΣ Β' λη ' τ.l/53


Δ ι α ν υ σμ ατι κές πpάξεις ­ Συ ντετα γμένε ς στο επίπεδο ­ Εσω τερι κ ό γινόμε νο διανυσμάτων Δημήτρης Μυλωνάς . . . Όμως προσέξτε : δεν μεrάνιωσα ποτέ μου οτο παραμικρό για τα όσα έμαθα σε εκείνα τα δύο χρόνια της ειδίκευ­ σής μου οτα Μαθηματικά, κι ας μην αποκόμισα από αυτά καμία άμεση χρησιμότητα : όπως είναι γνωοτό, τα θέμα­ τα της καθημερινής ζωής μπορούν να αντιμεrωπισθούν και χωρίς να γνωρίζεις το Αξιωματικό Σύοτημα των Τσερ­ μέλο - Φραίνκελ, τις μεrρικές ιδιότητες των χώρων Μπάναχ και τις λεπτομέρειες του Προβλήματος της Ταξινομή­ σεως των Μη οβελιανών Πεπερασμένων Απλών Ομάδων. Οι aπολαύσεις που στερείται ο μη μαθηματικός είναι μόνο αισθητικής, όχι πρακτικής, τάξεως αλλά αυτές είναι μοναδικές, ένα πάντρεμα του Ωραίου με το Λογικό, που δεν απαντιέται σε καμία άλλη ανθρώπινη δραστηριότητα. Είμαι λοιπόν ευγνώμων οτον θείο Πέτρο και οτο αμφιλε­ γόμενο υπόδειγμά του οτην εφηβεία μου, που με οδήγησε να γευθώ την έννοια της μαθηματικής ομορφιάς. Έοτω και για λίγο, έοτω και βρέχοντας μοναχά τις άκρες των ποδιών μου οτον απέραντο ωκεανό της. Το σημάδι της πα­ ραμένει ανεξίτηλο οτη ζωή μου, γεύση ενός κόσμου ιδεατού, ιδανικού μέσα οτο απόλυτο, άυλο κάλλος του. . ... «0 θείος Πέτρος και η Εικασία του Γκόλντμπαχ» Απόστολος Κ. Δοξιάδης Θεωρώντας λοιπόν περιπό οποιοδήποτε οχόλιο πάνω οτο παραπάνω απόσπασμα και κρατώντας την αίσθηση που αυτό αφήνει, παραθέτω μία σειρά από έξι λυμένα παραδείγματα πάνω οτις διανυσματικές πράξεις, οτις συντεrαγμένες οτο επίπεδο και οτο εσωτερικό γινόμενο. Στα παραδείγματα αυτά υπάρχουν γενικεύσεις κλασι­ κών θεμάτων, προβλήματα αλλά και θέματα που δείχνουν όλη τη διαδικασία προσέγγισης της λύσης μίας σει­ ράς ασκήσεων, με μεθοδολογικό τρόπο. τέλος αφήνονται για λύση οτον αναγνώστη 13 θέματα που άπτονται όλων των παραπάνω και που αποτελούν την ολοκλήρωση μίας πορείας οτο διανυσματικό λογισμό που έχει ως αφετηρ ία της το οχολικό βιβλίο της Θεrικής - Τεχνολογικής Κατεύθυνσης της Β ' Λυκείου.

ΛΥΜΕΝΑ ΘΕΜ Α Τ Α - ΓΕΝΙΚΕΥΣΕΙΣ ­ Π ΡΟΤΕΙΝΟΜΕΝΕΣ Α ΣΚΗΣΕ ΙΣ

1.

Σε παραλληλόγραμμο ΑΒΓΔ θεωρούμε τα ..... ..... σημαία Μ και Ρ ώστε ΑΜ 2 ΑΔ και 3 ..... 2 ..... ΑΡ - ΑΓ . Να δείξετε ότι τα Β , Ρ, Μ είναι 5 συνευθειακά. = -

=

Λύ ση yς τρόπος

Έστω

-+

(διανυσματικός) -+

ΑΒ = ΔΓ = a

και

-+

-+

-

ΑΔ = ΒΓ = β

.

Τότε ..... .....

2 ..... ..... 2 ..... 5 5 .... .... . . 3_ 23 2 = - - ΑΒ+ - ΑΔ = - - α + - β = 5 5. 5 5 3 - 2= - ( - α + - β) ( 1 ) και 3 5

ΒΡ

=

.....

.....

.....

ΑΡ - ΑΒ = - ΑΓ - ΑΒ = - (ΑΒ+ ΑΔ) - ΑΒ =

ΕΥΚΛΕΙΔΗΣ Β ' λη ' τ.V54


Μαθηματικά Β' Λυκείου

--+ --+ 2 --+ --+ --+ 2ΒΜ = ΑΜ- ΑΒ = -ΑΔ- ΑΒ = -<i + -β και λόγω 3 3 --+ --+ --+ 3 --+ της ( 1 ) ΒΜ = - ΒΡ οπότε ΒΜ // ΒΡ και επει5 δή τα τμήματα ΒΜ και ΒΡ έχουν κοινό το σημείο Ρ, τα Β, Ρ, Μ είναι συνευθειακά. 2"ς τρόπος

2.

Θεωρούμε ii , β δύο μη συγγραμμικά δια­ νύσματα του επιπέδου και τα σημεία Α, Β --+ τέτοια ώστε ΟΑ = 2ii και 08 = 3β (Ο η αρχή των αξόνων). Αν το Μ είναι σημείο --+ του τμήματος ΑΒ με ΑΜ = ρ ΑΒ , Ο < ρ < 1. Τότε : --+ α) Να εκφράσετε τα διανύσματα ΑΒ και ΟΜ συναρτήσει των ii , β , ρ. β) Αν τα ευθύγραμμα τμήματα. ΟΜ και ΒΝ (Ν μέσο του ΟΑ), τέμνονται στο Λ και --+ --+ ΒΛ = χ ΒΝ , xeR, να εκφράσετε το ΟΑ συναρτήσει των ii , β , χ. --+ --+ γ) Αν δίνεται ότι ΟΑ = ψ ΟΜ , ψεR να --+ εκφράσετε το ΟΑ συναρτήσει των ii , β ' ψ, ρ. δ) Να υπολογίσετε τα χ, ψ συναρτήσει του ρ και να εκφράσετε το ΟΑ συναρτήσει των ii , β , ρ. --+

-

--+

(αναλυτικός) Δ (γ-β.δ)

--+

-

--+

--+ --+ Έστω Α(Ο,Ο), Β(β,Ο), Γ(γ,δ) και επειδή ΔΓ = ΑΒ ισχύει Δ(γ-β, δ). Άρα ΑΜ = 3.Μ = 3. (γ - β), 3. δ δηλαδή 3 3 3

(

)

(� (γ - β), � δ ) και Ρ (� γ, � δ) Ισχύει ΒΡ = (� γ - β, � δ ) και ΒΜ = (� (γ - β) - β, � δ ) = (� γ - � β, deι (ΒΡ, ΒΜ ) = ;3. γ -5β 3.l δ = Μ

(

3γ - 3β

) (

3

--+

Λύ ση Υ

Β

δ

Α

)

= 3. δ 3. γ - β _ 3. δ 3. γ - � β = 5 3 3 3 5 = 4 γδ - 32 βδ - 4 γδ + 32 βδ = Ο , 15 15 --+ --+ οπότε ΒΜ // ΒΡ και επειδή τα τμήματα ΒΜ και ΒΡ έχουν κοινό το σημείο Ρ, τα Β, Ρ, Μ είναι συνευθειακά.

χ

χ

-+

-+

-+

-+

-+

y'

-

-+

α) ΑΒ = ΟΒ - ΟΑ <::::> ΑΒ = -2<i + 3β

Σ χόλιο :

Στο παραπάνω θέμα φαίνονται δύο βασικοί τρόποι, ο διανυσματικός και ο αναλυτικός, για να αποδει­ κνύουμε ότι τρία σημεία του επιπέδου είναι συνευ­ θειακά. Το σκεπτικό έχει τις ρίζες του στην ευκλεί­ δεια γεωμετρία όπου είναι γνωστό ότι «παράλληλες ευθείες» με ένα κοινό σημείο ταυτίζονται.

'

β)

-+

-+

-+

-+

(1)

ΟΜ = ΟΑ+ ΑΜ <::> ΟΜ = ΟΑ+ ρ ΑΒ και λόγω της ( 1 ) --+ 0Μ = 2<i + ρ(-2<i + 3β) <::::> --+ ΟΜ = -2(ρ - l)<i + 3ρβ (2) ΒΛ = χ ΒΝ <::> ΟΛ- ΟΒ = χ(ΟΝ- ΟΒ) <::> -+

-+

-+

-+

-+

-+

-+

-+

-+

-+

-+

-

-

ΟΛ = ΟΒ+ χ ΟΝ- χ ΟΒ <::::> ΟΛ = 3β + χ<i - 3χβ --+ <::::> ΟΑ = x<i + (3 - 3χ)β (3)

ΕΥΚΛΕΙΔΗΣ Β' λη ' τ.l/55


Μαθηματικά Β' Λυκείου

γ)

ΟΛ = ψΟΜ και λόγω της (2) ΟΑ = Ψ [ -2(ρ -l)ίi + 3ρβ ] � ΟΛ =-2(ρ- l)ψίi+3ρψβ (4) Εξισώνοντας τα δεύτερα μέλη των (3) και (4) έχουμε: χίi +(3-3χ)β = -2(ρ-1)ψίi +3ρψβ � [χ + 2(ρ -1)ψ] α = (3ρψ -(3-3χ) ] β . Διακρίνουμε τις περιπτώσεις: Αν (2ρ -1)χ + 2(ρ -1)ψ Ο, τότε α- = 3ρψ-(3-3χ) χ + 2(ρ-l)ψ · v , δηλαδη' α, β συγγραμμικά, πράγμα άτοπο, άρα χ + 2(ρ- 1)ψ = ο (5) Ο , τότε ΙΙ.Αν- 3ρψ-(3-3χ) χ + 2(ρ -1)Ψ · α- , δηλαδη' α,- β- συγβ = 3ρψ-(3-3χ) γραμμικά, πράγμα άτοπο, άρα 3ρψ-(3 -3χ) =ο (6) Οι (5) και (6) δίνουν σύστημα : + 2(ρ -1)ψ = ο � { χ + 2(ρ -1)ψ = ο . {χ3ρψ-3+3χ =0 χ + ρψ = 1 D = 1 11 2(ρρ-1) 1 =ρ-2(ρ-1) = = ρ -2ρ + 2 = -ρ + 2 αφού Ο<ρ< 1 = 101 2(ρ-1) ρ 1 = -2(ρ-1) =-2ρ+2 = 1 11 Ο1 Ι = 1 . Άρα το σύστημα έχει μοναδική λύση την (χ,ψ) = (-t' o) = 1 ) και = ( -2-ρρ++22 ' -ρ1+ 2 ) = ( 2ρ-2 ' ρ -2 ρ -2 λόγω της (3) -3ρ β . + ΟΑ = 2ρ-2 ίi ρ-2 ρ-2 -->

δ)

joA.j = α , jo"Bj = β και η γωνία των διανυ-

-->

-->

-->

-

Ι.

-->

σμάτων ΟΑ και ΟΒ είναι φ. Αν Μ είναι σημείο της ευθείας ΑΒ, με ΑΜ = κ· ΑΒ , τότε : Να υπολογίσετε το μέτρο του διανύσματος ΟΜ συναρτήσει των α, β, φ και κ. β) Να βρείτε το συνημίτονο της γωνίας ω των διανυσμάτων ΟΜ και ΑΒ . γ) Ν α βρείτε το συνω όταν : i) το Μ είναι μέσο του ΑΒ, ίί) το Μ ταυτίζεται με το Α ίίί)το Μ ταυτίζεται με το Β ίν) φ = 90° ν) ΟΑ = ΟΒ δ) Να βρείτε τη γωνία των διανυσμάτων ΟΜ και ΑΒ , όταν φ = 45°, α = 2 και β =

α)

-->

_R_

f.

-->

-->

ι.

ε)

-->

Να δείξετε ότι η προβολή του ΟΜ στο ΑΒ στην περίπτωση που α = β είναι προβ _, ΟΜ = 2κ 1 · ΑΒ . 2 -->

ΑΒ

__,

D

ψ

D

3.

D

ψ

--

__,

Λύ ση ο

f.O

χ

-->

-->

:t:.

D

-->

-->

α)

Α

ΟΜ = ΟΑ +ΑΜ = ΟΑ+κ·ΑΒ = ΟΑ +κ· (ΟΒ-ΟΑ) = ΟΑ +Κ· ΟΒ - κ· ΟΑ άρα ΟΜ = (1-κ) · ΟΑ2 +κ· ΟΒ (1) Οπότε : I ΟΜ 1 = ΟΜ 2 = [(1-κ) ΟΑ+ κ · ΟΒ]2 = (1-κ)2 · ΟΑ2 + 2κ(l-κ) · ΟΑ· ΟΒ+ κ2 · ΟΒ2 = 4

-+

-+

ΕΥΚΛΕΙΔΗΣ Β' λη ' τ.l/56

-+

-->

-->

Δίνεται τρίγωνο ΟΑΒ στο οποίο είναι

-+

-->

-+

-+

-+

-+

-+

-->

-->

-->

-+

-+

-->

-+


Μαθηματικά Β' Λυκείου

(1-κ)2 · I OAI2 + 2κ(1-κ)· (1-κ)2 · α2 + 2κ(1-κ)αβσυνφ + κ2 β2 . Και επομένως: ΟΜ = �·(1---κ-)..,-2_α2_+__2κ_(-1- κ-)α_β_σ_υν_φ_+_κ=-2β-=-2 Ισχύει2 ότι : ΑΒ = ΟΒ -ΟΑ (2) άρα 2= ( 2 . .... ) = = ΑΒ ΟΒ-ΟΑ ΟΒ-ΟΑ 1 I -> 1 I -> ΟΒ22-20Β·ΟΑ+ΟΑ2 ο\ 8\ - 2 1 oBI · I oΛI · συνφ + \ οΛ1 2 = = α2 -2αβσυνφ + β2 . Άρα IAΉI �α2 β2 -2αβσυνφ Επίσης λόγω των (1) και (2) ΟΜ · ΑΒ = [( 1-κ)ΟΑ+κ·ΟΒ }( ΟΒ -ΟΑ) =(1-κ) ΟΑ ΟΒ-(1-κ)ΟΑ2 +κ0Β2 -κ0ΑΟΒ= (1-2κ)ΟΑ ΟΒ-(1-κ)ΟΑ2 + κ ·ΟΒ2 = (1-2κ)αβσυνφ-(1-κ)α2 +κ β2 . Άρα, ΟΜ· ΑΒ (1-2κ)αβσυνφ - (1-κ) α2 +κβ2 (3). Οπότε συνω ΟΜ·ΑΒ ·

β)

.....

.....

.....

.....

-->

--+

--+

=

--+

=

.....

-+

-+

=

-+

-+

--+

.....

-+

-+

-+

=

.....

I oM! + �i (

(4) i) Αν το Μ είναι μέσο του ΑΒ τότε κ = .!.. και ο τύπος (4) γίνεται : συνω = 1 1 1 (1-2· -)αβσυ 2 νφ--α2 2 +-2 β2 1 φ+-1 β2 ·να1 2 β2 -2αβσυνφ -α41 2 +-αβσυν 2 4

0\MJ)

γ)

--+

+

--+

.....

--+

(l-2κ)ι(Όwρ-(1-κ) fi +J4f �(1-κ/ fi +2κ(l-κ)ι(Όwρ+κ1f · �fi +JY -�

2

+

_!_2 (β2 -α2 ) � συνω = 2 + β2 2 β 2 � }�( α ) - ( 2α συνφ ) 2 .� συνω = �α4 + β4 + 2αβ22β_2α-4α 2β2συν2φ συνω = �α4 +β4 +2αβz2β-α2 ·2( 1-2συν2φ ) � 2 -α2 συνω = �α4 + β4 β-2α 2β2 · συν2φ ii) Αν το Μ ταυτίζεται με το Α τότε κ = Ο και ο τύπος (4) γίνεταιβ: 2 συνω = Ν · �αα 2συν+ βφ-α 2 -2αβσυνφ � συνω = α�αα(2 +βσυνφ-α) β2 -2αβσυνφ � συνω = �α2 +βσυνφ-α β2 -2αβσυνφ iii) Αν το Μ ταυτίζεται με το Β τότε κ = 1 και ο τύπος (4) γίνεταιβ : 2 συνω = � · �α-α2 συνφ+β +β 2 -2αβσυνφ ) � συνω = .β�αβ2 (β-ασυνφ + β2 -2αβσυνφ συνω = �α2 +ββ-ασυνφ 2 -2αβσυνφ iv)Aν φ=90° τότε ο τύπος (4) γίνεται : συνω = � � 2 συνω = �(1-κ)κ2βα22-(1-κ)α +κ2β2 · �α2 + β2 v) Αν ΟΑ = ΟΒ = α τότε ο τύπος (4) γίνεται: συνω = (1 - 2κ)α2συνφ - (1 - κ)α2 + κα2 � �(1-κ)2α2 +2κ(l-κ)α2συνφ+κ2α2 �α2 +α2 -2α2συνφ συνω =

ΕΥΚΛΕΙΔΗΣ Β ' λη ' τ.V57

--;==:====:='=====:=:====

--;==:=='===:=='====

-;==:==='=:====='==

(1- 2κ)συν90'- (1- κ:)α2 + κβ2 (1- κ:)V + 2κ:(l- κ)αβσυv9(J + �β2 α2- 2αβσυν90" + β2 ο

= r===========--r===

ο


Μαθηματικά Β ' Λυκείου

α2 [(1 - 2κ)συνφ - 1 + κ + κ] �1 - 2κ + κ2 + 2κ(1 - κ)συνφ + κ2 �2 - 2συνφ

α2 συνω = +==(2κ=�-1)�r====== � (1-2κ)συνφ = -�r2=κ�2== = = = = == = = = - 2κ + 1 + 2κ(1 - κ)συνφ . �2 - 2συνφ συνω = (2κ -1)(1-συνφ) -�r2=κ�2== - 2== κ +=1=+=2=κ=(1=-=κ=)συ==ν=φ�.-J2r2�.��=1-==συ=ν=φ � Από το ερώτημα έχουμε: 22 -12 συνω = �24 +14 -2·22 ·12 · συν90° � συνω = ν1b7 συνω �Ο, 728 , άρα ω� 43,3° Ισχύει ότι : προβ _, ΟΜ = λ· ΑΒ , λ R. Άρα ΟΜ· ΑΒ = ΑΒ·προβ .... ΟΜ � ΟΜ· ΑΒ = λ· ΑΒ2 και λόγω της (3) (1-2κ) α2 συνφ-(1-κ) α2 +Κ α2 = λ( α2 -2α2συνφ+ α2 ) � α2 [(1-2κ) συνφ-(1-κ) +Κ] = λα2 (1-2συνφ+1) � (1-2κ)συνφ +2κ-1 = λ(2-2συνφ) � = (2κ-1)(1-συνφ) � 2λ(1-συνφ) 2κ-1 . � λ = -2 2κ-1 ΑΒ.. Οπότε: προβ .... ΟΜ = --· 2 •

=

Λύ ση Γ

γί)

δ)

ε)

-->

ΑΒ

---7

-->

-->

-->

-->

ΑΒ

ε

=

ΑΒ

-->

Ε

Β

ΓΕ άρα ΓΑ+ΓΒ = 0� . Μ ·ΓΕ =Ο� ΑΒ+ΑΓ 2 2 (ΑΒ+ ΑΓ)(ΓΑ+ ΓΒ) = Ο� .... .... ] [-ΒΑ+(ΒΓ-ΒΑ)] · [(ΒΑ-ΒΓ)-ΒΓ) = 0� = Ο� (ΒΓ-2ΒΑ) · (-2ΒΓ+ΒΑ) .... .... .... .... .... 2 = 0 � -2ΒΓ.... 2 +ΒΓ·ΒΑ+4ΒΓ·ΒΑ-2ΒΑ .... .... =2ΒΓ.... 2 +2ΒΑ.... 2 � 5ΒΓ·ΒΑ

α) ΑΔ ..l

-+

-+

-+

-->

-+

-->

-+

---7

---7

-+

-+

-->

-->

--)>

-+

-->

Σ χόλιο :

Με τη βοήθεια του παραπάνω θέματος διαπιστώ­ νουμε τη διαδικασία σύμφωνα με την οποία αν γνωρίζουμε δύο οποιαδήποτε μη συγγραμμικά δια­ νύσματα του επιπέδου καθώς και τη μεταξύ τους γωνία, μπορούμε να υπολογίσουμε το μέτρο ο­ ποιουδήποτε γραμμικού τους συνδυασμού, όπως επίσης και τη γωνία δύο οποιωνδήποτε διανυσμά­ των που αποτελούν γραμμικούς συνδυασμούς των δύο αρχικών. 4.

=

β) Στην περίπτωση που συνΒ -4 ισχύει 5 , συνΑ .Jlii ι . οτι

Σε τρίγωνο ΑΒΓ οι διάμεσοι ΑΔ και ΓΕ τέ­ μνονται κάθετα. Να δείξετε ότι : 4 α) συνΒ � 5

Αρκεί να δείξουμε ότι συνΒ � �5 και λόγω της 2 ! Br! 2 + 2 1 ΒΛ1 2 4 (1) r Λ 5 5 1B i · IB I 2 1 Brl2 + 2 1 ΒΛ1 2 � 4 1 Bri · I BΛI � B1 rl2 + I BΛI2 -2 1 Bri · I BΛi � o � (1 Bri - I BΛIJ2 �ο . Η τελευταία είναι προφανής ανισότητα οπότε

ΕΥΚΛΕΙΔΗΣ Β ' λη ' τ.l/58

>-�


Μαθηματικά Β ' Λυκείου

Β�-.45 2 συνΒ = � � (1 Bri - I BAI ) =ο� I Brj = I BA I �

ισχύει και η συν β)

ν

()() '

{

6.

··•

β

α

ν = 3β .

Υ

με

2

β νι = 3 3 l i3l

60 '

{

1

ν

.

!

Διακρίνουμε τις περιπτώσεις : I) Αν τα και βρίσκονται προς το ίδιο ημιε­ πίπεδο στο οποίο χωρίζει ο φορέας του το επίπεδο, ισχύει ότι τα και v είναι ομόρροπα = και επειδή ι άρα

β

α

(1) α , ν και έχουμε: ν . α = λ·<i2 +μ· βα � ν2 =λ . αν+ μ . β-ν Ι νΙ ·Ι αΙ · συν(ν, α) = == λ · l <i l 2 + μ · l ί3I · Ι α Ι · συν(β, α) Ι νΙ 2 = λ · Ι αΙ · Ι νΙ · συν( α, ν)+ + μ · l ί3 I ·Ι ν Ι · συν(β, ν) 0 = λ ·12 + μ ·1·1· συν60° � 323 ·1·= λσυν60 ·1· 3συν60° + μ ·1 · 3 · συν120° 3· .!.2 =λ+μ· .!.2 � 9 � 3λ·Ξ+3μ ( � Ξ) 2λ+μ= 3 και λύνοντας το σύστημα � {3λ-3μ = 18 προκύπτει ότι λ = 3 και μ = - 3. Δηλαδή ν = 3<i -3β

Ισχύουν

=

-3β

.

Πολλαπλασιάζουμε την

Λύ ση

-

3α·

.

Αν τα μοναδιαία διανύσματα α και β σχη­ ματίζουν γωνία φ με Ο < φ < 90° και τα δια­ νύσματα ρ = α + 2β και q = sα - 4β είναι κάθετα, τότε να αναλύσετε το διανυσμα v με lvl = 3 και ( v,α) = 60° σε δύο συνιστώσες παράλληλες στα α και β .

-

α

ν = λα + μβ ( 1)

180° � 2Α = 180° - Β και συν(180°-Β) � -1 = -συνΒ � 2συν2Α = 1 - �5 � συν 2Α = _..!_10 _ και επειδή Α<90° (αφού Α=Γ) άρα συνΑ = νΙΟ �.

j αj = l i3 l = 1 (1) και p·q = ο�( α+ 2β)( 5α-4β) =ο� 5α2 -4αβ + ι οαβ - 8β 2 = ο� 5 j <ij 2 + 6αβ -8 l i3 1 2 = ο και λόγω της (1) 5+6αβ-8= ο�αβ - � Ι α Ι · I ί3 1 συνφ = � και λόγω της (1) συνφ = � άρα φ = 60°.Δη λαδή (β, α) = (ν, α) = 60°.

ν

τα και βρίσκονται σε διαφορετικά η­ μιεπίπεδα στα οποία χωρίζει ο φορέας του το επίπεδο, ισχύει ότι

Γ=Α. Είναι Α+Β+Γ = επομένως συν2Α = 2συν 2 Α

5.

β

Π) Αν

Σε καρτεσιανό σύστημα αξόνων χ 'χ και ψ'ψ, τα σημεία Α(Ο,α J3 ), Β(-α,Ο) και Γ(α,Ο), α > Ο, παριστάνουν τρεις πόλεις. Τα ευθύγραμμα τμήματα ΑΒ και ΑΓ παριστά­ νουν δύο εθνικούς δρόμους που συνδέουν την πόλη Α με τις πόλεις Β και Γ αντίστοι­ χα. Πάνω στον εθνικό δρόμο ΑΒ βρίσκεται το αεροδρόμιο Δ που εξυπηρετεί και τις τρεις αυτές πόλεις σε σημείο ώστε ΑΔ = i ΔΒ ενώ πάνω στον εθνικό δρόμο ΑΓ βρίσκεται ο κεντρικός σιδηροδρομικός σταθμός Ε που εξυπηρετεί και τις τρεις αυ-

ΕΥΚΛΕΙΔΗΣ Β ' λη ' τ.l/59


[ χΔΔ-α= ν'32 =-2 ψΔΔ �[�2 χΔΔ = αν'32 � Ψ � Ψ [ΨχΔ,==-::�- {� j3J

Μαθηματικά Β' Λυκείου

τές πόλεις ώστε ΑΕ = 2ΕΓ . α) Να βρείτε τις συντεταγμένες του αεροδρο­ μίου Δ και του σιδηροδρομικού σταθμού Ε ως συνάρτηση του α. β) Ένας ευθύγραμμος δρόμος ΒΕ συνδέει την πόλη Β με το σιδηροδρομικό σταθμό Ε και ένας ευθύγραμμος δρόμος ΓΔ <rυνδέει την πόλη Γ με το αεροδρόμιο Δ. Στο σημείο που τέμνονται οι δύο δρόμοι χτίζεται ένα μεγάλο συνεδριακό κέντρο. Να βρείτε τις συντεταγμένες του συνε­ δριακού κέντρου ως συνάρτηση του α. γ) Να αποδείξετε ότι ο ευθύγραμμος δρό­ μος που συνδέει την πόλη Α με το συνε­ δριακό κέντρο, τέμνει κάθετα το δρόμο ΒΕ. δ ) Αν η τριγωνική περιοχή ΑΒΕ έχει εμβαδόν ι so.J3 τετραγωνικά χιλιόμετρα, να. βρείτε το α και στη συνέχεια την απόσταση του συνεδριακού κέντρου από το σιδηροδρομι­ κό σταθμό και το αεροδρόμιο. Λί> ση Υ

Α(Ο.α-!3)

χ

α)

'

Δ( χΔ , ψΔ

y'

Έστω ) οι συντεταγμένες του αερο­ δρομίου και Ε( ) οι συντεταγμένες του σιδηροδρομικού σιδηροδρομικού σταθμού. Οπότε: 1

χ ε, ψε

ΑΔ =-2 ΔΒ� ( χ Δ - Χ Α ,ψΔ -ψΑ ) =-2 (Χ Β - ΧΔ ,ψΒ -ψΔ ) � ( χ Δ -Ο,ψΔ -αν'3 ) = � ( -α- ΧΔ ,0-ψΔ ) � -->

-->

1

_ .!_ α _ .!.. χ

%

Άρα

3

= _ .!_ α

α,

Εργαζόμενοι όμοια έχουμε :

ΑΕ = 2 ΕΓ� ( χ ε - χ Α , Ψε -ψΑ ) = 2 ( χ r - Χ ε , Ψr -ψε ) � ( χε -Ο, ψε -α.J3 ) = 2 ( α- Χ ε,Ο-ψε) � � ( 3χ3ψεε == 2αα.J3 � . ( 2 αν'3 Άρ α Ε - 3 α , -3J --+

-->

β)

Σ

Έστω το σημείο τομής των δρόμων ΒΕ και όπου θα χτιστεί το συνεδριακό κέντρο. Τα σημεία ,Ε είναι συνευθειακά αν και μόνο αν τα διανύσματα και είναι παράλληλα. Επομένως :

ΓΔ

Β, Σ

ΒΣ ΒΕ det ( ΒΣ,ΒΕ ) = ο� ι ΧεχΣ -- χχ ΒΒ ΨΨΣε --ψΨΒΒ Ι = Ο ΨΣ -0 = χ Σ +α 2 αν'3 0� -5 α -α + α ---0 3 3 3 αν'3 χΣ +α )- -5 αψΣ = 0� -( 3 3 αν'3 ( χ Σ +α )-5αψΣ = 0� (1) ΧΣ .J3-5ψΣ = -αν'3 Τα σημεία Γ, Σ ,Δ είναι συνευθειακά αν και μόνο αν τα διανύσματα ΓΣ και ΓΔ είναι παράλληλα. Επομένως : deι ( ΓΣ,ΓΔ ) = ο� Ι χΧΔΣ -- xXrr ΨΨΔΣ -- ΨrΨr l = o

ΕΥΚΛΕΙΔΗΣ Β' λη ' τ.l/60

-->

-->

-->

-->


Μαθηματικά Β ' Λυκείου

ΨΣ = 2ι:ι.J3 0� 3

Το εμβαδόν της τριγωνικής περιοχής ΑΒΕ ως συνάρτηση του α είναι : 2α..f7 = . Ε = _!_2 I BEI · I AΣI = _!_2 . 2αJ2Ϊ 7 3 2α2 J3 14α2 J3 = -= = 2α2 .fi2:3 21 2 21 3 , 2α J3 = 150ν3 � και επομενως 3 2 � α = 225 άραα= 15 kιη. Η απόσταση του συνεδριακού κέντρου από το αεροδρόμιο είναι: 2α 2 ' •j3 J' � + + � : � ) [ � ( Τ I ΔΣΙ 2 = αJ82:7 = = ( 16α21 )2 + [ 8α21J3 J2 =�448α 212 21 = 8α21..f7 = 8·1521..f7 = 407..f7 Η απόσταση του συνεδριακού κέντρου από το σιδηροδρομικό σταθμό είναι : 2α ' + � � :� ·� J' � ) [ ( � Τ IΕΣΙ ΑΒΕ

(2) Οι (1) και (2) δίνουν δύο εξισώσεων με δύο αγνώστους. Αφαφώντας την (1) από τη (2) 2αJ3' οτι' 7 ΨΣ =2α "-'r;;3 δη η ΨΣ = προκυπτει 7 και αντικαθιστώντας (2) έχουμε ότι : J3 χΣ J3 + 4α7 = αJ3 �χ Σ J3 = 3αJ3 7 � � Χ Σ = τ3α · Άρα οι συντεταγμένες του συνεδριακού κέ­ ντρου είναι Σ[ 3; , 2"7.,/3 ] . ΑΣ = [ 37α 2α7J3 -αJ3] = [ 3α7 ' 5αJ3 7 J

r;;

σύστημα

λαδ '

στη

γ)

- Ο,

km .

_

= (� �:)' + ·: ]' = ���' � 2�� � = 2 . ιs.J7 = ι ο..JΊ 21

Οπότε ο ευθύγραμμος δρόμος ΑΣ τέμνει κάθε­ τα τον ευθύγραμμο δρόμο ΒΕ. = 9α492 + 75α49 2 = �84α492 = 2αJ2Ϊ 7 25α2 +3α2 = -9 9

7

Κm

ΠΡΟΤΕΙΝΟΜΕΝΑ ΘΕΜΑΤΑ 1.

ΑΒΓ. Δίνεται.... τρίγωνο Αν Ρ σημείο τέτοιο, .... .... ώστε = -3ι ΑΒ+-32 , να δείξετε ότι το Ρ είναι σημείο της ΒΓ. 2 .... Β....Ρ = .... Α....Β = ... = -ΒΓ) 3 .... Δίνονται τα διανύσματα ΟΑ = (κ+ l)a + 3β , ΟΒ=2κa+(3κ-1)β , ΟΓ = -a-5β , όπου β δύο μη συγγραμμικά διανύσματα . Να βρείτε την τιμή του κ ώστε τα Α, Β, Γ να είΑΡ

ΑΓ

(Υπόδειξη :

2.

ΑΡ-

-

α,

ΕΥΚΛΕΙΔΗΣ Β' λη ' τ.l/61

-

-+

-


Μαθηματικά Β' Λυκείου

ναι συνευθειακά. (ΔΙΑΓΩΝΙΣΜΟΣ ΕΚΠΑΙ­ ΔΕΥτΙΚΩΝ ΑΣΕΠ-2002) (Υπόδειξη: ΓΑ=..... ΟΑ-ΟΓ = ... = (κ+ 2)α + 8β .... . .... . ... = (κ-1)α +(3κ-4)β . και ΓΒ=ΟΒ-ΟΓ= .... . .... . Αρκεί ΓΑ= λ · ΓΒ, λεR οπότε κ=Ο ή κ=2) Δίνεται τρίγωνο ΑΒΓ και τα..... σημεία Δ,1 Ε και..... .... . .... . 1 Ζ τέτοια ώστε: = -3 ΑΒ , ΓΕ = -2 ΒΓ και - 53 ΑΓ..... . ..... α) Αν ΑΒ = α και ΑΓ = β- να εκφράσετε τα ΔΕ και ΔΕ συναρτήσει των α και β . β) Να δείξετε ότι τα σημεία Δ, Ε, Ζ είναι συνευθειακά. (Απάντηση: α) ΔΕ =-�α+lβ και ....Δ . 6=- 1 2α+ 3-β , Ζ 3 S ..... ..... β) Να δείξετε ότι ΔΕ = -5 · ΔΖ ) 2 Εστω τρίγωνο ΑΒΓ, η ....διάμεσος ΑΔ και ση.... . . 1 μείο Ε, ώστε ΑΕ =-2 . Στην ευθεία ΒΕ ..... = λΒ....Ε. , λεR. θεωρούμε σημείο Μ,....ώστε ΒΜ . α) Να γράψετε το ΑΜ ως γραμμικό συνδυασμό των ΑΒ και ΑΓ..... . β) Αν Ρ το σημείο τομής των ευθειών ΒΕ και ΑΓ,..... να..... βρείτε τον αριθμό κ ώστε ΑΡ=κΑΓ. ..... -3λ+4 ..... λ ..... (Απάντηση: α) ΑΜ = 4 , 4 ΑΒ+-ΑΓ 3 β) Κ= -) 16 Δίνονται τα σημεία Α(2,5), Β(0,4), Γ(7,2). α Να βρείτε τις συντεταγμένες του σημείου Μ0 ώστε να ισχύει ΑΜ..... 0 = 3 ΒΓ..... . β) Να β ρείτε τις συντεταγμένες του Μκ , κ εΝ ώστε ΑΜκ =(κ+3) ΒΓ . .......

3.

-+

-

6.

ΑΔ

ΑΖ

4.

---+

= -

.....

7.

)

ΑΔ

)

.....

5.

)

.....

.....

Να δείξετε ότι τα σημεία Μ0, Μ2, Μ5 είναι συνευθειακά. (Απάντηση : α) Μ 0 (23,-1), ..... β) Μκ(7κ+ 23,-2κ-1) γ) Μ 0 Μ 2 = (14,-4) 3 0....Μ. 2 ) και Μ2....Μ. 5 = (21, -6) =-Μ 2 Οι κορυφές Α και Γ ενός τετραγώνου ΑΒΓΔ έχουν συντεταγμένες (8,-6) και (2,2) αντί­ στοιχα. Να βρείτε τις συντεταγμένες των κο­ ρυφών Β και Δ. (Απάντηση : Β(9,1) και Δ(l, - 1)) Για τα διανύσματα α, β δίνεται ότι : ι αι = 1, J β l = 2 και (α, β) = � . Εστω τα διανύσματα ίi = 2α + 3β, ν = α-2β . Να υπολογίσετε: α το εσωτερικό γινόμενο α· β . β) Τα μέτρα iui και ι ν ι των διανυσμάτων ίi και ν . γ) το εσωτερικό γινόμενο ίi · ν . δ) Το συνημίτονο της γωνίας των διανυσμά­ των ίi και ν . (ΠΡΟΑΓΩΓΙΚΕΣ ΕΞΕΤΑΣΕΙΣ Β ' ΛΥΚΕΙΟΥ 2001) (Απάντηση : α α . β=1, β) ι υ ι = 2Ji3 ' ι ν ι = J13 γ) ίi . ν= -21, δ) συν(ίi, ν) = _ Ε_ ) 26 Μια ομάδα που αποτελείται από 50 αθλητές θα συμμετέχει στην τελετή έναρξης μιας πανευρωπαϊκής αθλητικής εκδήλωσης που θα γίνει στο Ολυμπιακό Στάδιο της Αθήνας. Η ομάδα είναι χωρισμένη σε δύο γκρουπ των 25 αθλητών. Για να γνωρίζουν οι αθλητές τις θέ­ σεις τους, έγινε μια τοποθέτησή τους σε χάρ­ τη, σε ορθοκανονικό σύστημα αξόνων, στο οποίο το διάνυσμα θέσης των 25 αθλητών του πρώτου γκρουπ είναι γ)

8.

ΕΥΚΛΕΙΔΗΣ Β' λη ' τ.l/62


Μαθηματικά Β' Λυκείου

0Aλ =2i+λ(3 i+ j) όπου λ=1,2, ... ,25 ενώ το διάνυσμα θέσης των υπολοίπων 25 αθλη­ τών του δεύτερου γκρουπ είναι OBv = 5 j +ν(- i + 2 j) όπου ν=1 ,2, ... ,25, και Ο η αρχή των αξόνων. α) Να β ρείτε τις συντεταγμένες του σημείου λ. ως συνάρτηση του λ και του σημείου ως συνάρτηση του ν. β) Να εξετάσετε αν το σημείο (-4,13) μπορεί να είναι η θέση κάποιου αθλητή του πρώ­ του γκρουπ ή του δεύτερου γκρουπ. γ) Να β ρείτε την απόσταση του 3 ou αθλητή από το πρώτο γκρουπ από τους υπολοί­ πους του δεύτερου γκρουπ ως συνάρτηση του ν. δ) Να β ρείτε ποιος αθλητής του δεύτερου γκρουπ απέχει από τον 3° αθλητή του πρώτο γκρουπ απόσταση ίση με (Απάντηση : α) λ. (2+3λ,λ) και (-ν,2ν+5) β) του αθλητή από το δεύτερο γκρουπ γ) A3B v = �(-ν-11) 2 +(2ν+2) 2 = = .J5ν2 + 30ν + 125 , δ) ο 5ος αθλητής ) Αν δ = 5 - β , όπου lϊil 2, ι β ι =5 και (a, β) = 5: , να υπολογίσετε το j δ j καθώς και τα συνημίτονα των γωνιών (δ, } και (δ, β) . (Απαντηση: συν ( δ-,α- ) �51+J3 , + 2J3 , 3 συν ( δ,- β- ) ιο.J5+=+J3=2J3 �

......

-+

-+

10.

11.

Α

Βν

4ου

9.

ίi

4

=

ίi

=

=

---,:. 4

::::= )

ΑΒΓ

--+

--+

Βν

Α

(4, 2)

ίi

αποτελούν

-+

20.

Δίνονται τα διανύσματα = (2, -3) , β = και γ= (-2, -1) . Να εξετάσετε αν πλευρές τριγώνου. (Απάντηση : λ = λ = β 1 i) Σε τρίγωνο ισχύει !Arl = 2 ι ΑΒι . Αν ΑΔ διάμεσος7 του τριγώνου, Ε σημείο ώστε ΑΕ=-10 · ΑΓ και Δ ΔΕ , να δείξετε ότι (Υπόδειξη : ΑΔ · ΔΕ = Ο � -(2 ΑΒ+ΑΓ)(ΑΕ- ΑΔ) = 0 � . . . � συνΑ = -2 δηλαδή Α Να αποδείξετε ότι, εάν ένα έχει δύο διάμεσους ίσες τότε είναι ισοσκελές. --+

Α

.l

--+

Α=60° . --+

1

--+

--+

--+

--+

--+

Ι

= 60°)

12.

τρίγωνο

-+

(Υπόδειξη :

-+

ΒΑ+ΒΓ = ΓΒ+ΓΑ � 2 2 ΑΜ · = Ο , όπου διάμεσος είναι κάθετη στην πλευρά ΒΓ, άρα ΑΒ = ΑΓ). Έστω , β , χ επιπέδου τέτοια ώστε τα β , χ τα και να ισχύουν ·χ 1=0, · β =2 α) Να εξετάσετε αν τα και β είναι συγγραμμικά β) Να εκφράσετε το διάνυσμα χ συναρτή­ σει των και β . β είναι συγγραμμικά τότε (Υπόδειξη : α) , χ κάθετα οπότε · χ 1 δηλαδή 1 =0 άτοπο, β) χ = -2 +.!.β) 2 -+

-+

<::::>

--+

--+

ΒΓ

• • •

ΑΜ διάμεσος. Δηλαδή η

ΑΜ

13.

ίi

μη μηδενικά διανύσματα ενός να είναι κάθε­

ίi

ίi

+

και

l ίi 1= 1 .

ίi

ίi

Αν ίi ,

ίi

ΕΥΚΛΕΙΔΗΣ Β' λη ' τ.l/63

ίi

+

=0

ίi


Μαθηματ ι κά

για τη Γ'

τάξη

του

Λ υκείου

\) Θέματα Μιγαδικών αριθμών του Βασίλη

Καρκάνη

Στόχος του άρθρου αυτού είναι να βοηθηθεί ο μαθητής της θετικής και Τεχνολογικής κατεύθυνσης της Γ' Λυκείου στο Κεφάλαιο των Μιγαδικών Αριθμών. Έγινε προσπάθεια, ώστε στο πρώτο μέρος με τις ερω­ τήσεις θεωρίας που παρατίθενται, ο μαθητής να έχει την απαραίτητη εικόνα της ύλης που εξετάζεται στις πανελλήνιες εξετάσεις. Στη συνέχεια στο δεύτερο μέρος οι λυμένες ασκήσεις και η μεθοδολογία τους, δίνει στο μαθητή τους βασικούς άξονες δουλειάς του κεφαλαίου αυτού. Για πληρέστερη γνώση της ύλης του κεφαλαίου που εξετάζεται, θυμίζω τα άρθρα των τευχών 41, 45, 49 του Ευκλείδη Β ' των Σπύρου Γιαννακοπούλου - Γιώργου Κατσούλη, Χρήσου Λαζαρίδη και Γιώργου Τριά­ ντου αντίστοιχα. •

Η έννοια του μιγαδικού αριθμού

Πράξεις μιγαδικών αριθμών.

Συζυγής μιγαδικού αριθμού.

Μέτρο μιγαδικού αριθμού.

Α. Ε ρωτήσεις Θεωρίας 1. 2.

τι λέμε εικόνα του z στο μιγαδικό επίπεδο και πως συμβολίζεται; Ποια σχέση έχει η εικόνα ενός μιγαδικού και του συζυγή του στο μιγαδικό επίπεδο; στ)Αν Μ(α, β) είναι η εικόνα του z στο μιγα­ δικό επίπεδο τότε η διανυσματική ακτίνα ΟΜ παριστάνει τον μιγαδικό αριθμό z; ζ) Να γραφεί το άθροισμα z + και η δια­ φορά z - συναρτήσει των α, β. η) Να γραφεί το Re(z) και το lm(z) συναρτήσει των και Να δοθεί ο ορισμός της ισότητας δύο μιγαδι­ κών αριθμών z1 και z2• Αν z1 = α+ βi και z2 =γ+δi με α, β,γ,δ εJR να ορίσετε τους αριθμούς: β) Ζ 1 - Ζ 2 α) Ζ 1 + z 2 ε)

Να ορίσετε το σύνολο των μιγαδικών αριθ­ μών C. Για τον μιγαδικό αριθμό z = α+ βi με α, β εJR . α) Να ορίσετε το πραγματικό και το φαντα­ στικό μέρος του. Πώς συμβολίζονται; β) Πότε ο z είναι πραγματικός αριθμός και πότε φανταστικός; Ποια μορφή έχουν οι φανταστικοί αριθμοί; γ) Πότε θα είναι z δ) Να ορίσετε τον συζυγή του z. Πώς συμβο­ λίζεται;

z

z

z

3. 4.

= Ο;

ΕΥΚΛΕΙΔΗΣ Β' λη ' τ.l/64

z.


Μαθηματικά Γ Λυκείου

5.

Να ερμηνεύσετε γεωμετρικά τις πράξεις (α) και (β). Να γραφούν οι ιδιότητες των συζυγών μιγαδι­ κών αριθμών και να δειχθεί ότι: α) Ζ ι + z2 = zΊ + z2 β) ( z v ) = (zγ .

6.

9.

10.

11.

Ποια ιδιότητα χαρακτηρίζει το στοιχείο ί του C; β) Να γραφούν οι τιμές της δύναμης ίν όπου ν Ν . Στη συνέχεια να υπολογιστεί η δύ­ ναμη i2004• γ) Αν z :;e Ο και ν Ν να συμπληρωθούν οι ισότητες: 1

μεία

α)

.... � ιzι + z2 ι � . ...

Ε

Ε

'

Ζ ''' 2 111) z = ..... ο V ) Ζ -ν = = .....

Ι)

12.

z = ..... iv) Ζ ν = .... ii)

. . . .

7.

Δίνεται η εξίσωση αz2 + βz + γ = Ο (1) με α, β, γ R και α :;e Ο . α) Τι γνωρίζετε για τη λύση της (1); β) Αν z 1 , z2 είναι οι ρίζες της ( 1 ) να συμπλη­ ρώσετε τις ισότητες: Ε

i)

z 1 + z2 = ....

ii)

z1 • z2 = ....

Παρατή ρ η ση Σ το σύ νο λο C τ ων μιγαδικών αριθμών δ εν ορί­ ζεται η έννοια τη ς διάταξη ς. Σ τη ν π ερίπτ ω ση π ου μας δ ώ σ ου ν ότ ι ισχύ ει : α + βί > γ + δί με α , β , γ , δ Ε R υ π οχρεωτικά θα είναι : β

=

δ

=

Ο και α > γ.

Όμοια στη ν π ερίπτ ω ση α + βί

8.

<

Β. Λυμένες Ασκήσεις - Μεθοδολογία. χ

Ε

13.

Να γράψετε τι παριστάνουν τα σύνολα λύσε­ ων των παρακάτω εξισώσεων: α) l z - z0 1 = ρ με ρ > Ο και z0 γνωστό μιγαδικό αριθμό. β) l z - z1 1 = lz - z2 1 με zι, Zz γνωστούς μιγαδικούς αριθμούς και z1 :;e z2 . Να γράψετε τι παριστάνουν τα σύνολα λύσε­ ων των παρακάτω ανισώσεων: α) lz - zo l > ρ ,ιz - zo l � ρ ,ιz - zo l < < ρ , Ι z - z0 1 � ρ με ρ > Ο και z0 γνωστό μιγαδικό αριθμό. β) lz - z1 1 > lz - z2 1 , Ιz - z1 1 � l z - z2 1 , lz - z 1 1 < 1z - z2 1 ,Ιz - z1 1 � 1z - z2 1 με zι, Zz γνωστούς μιγαδικούς αριθμούς και z1 :;e z2 . γ) ρ ι < Ι z - z σ l < ρ 2 , ρ ι � Ι z - zo l � ρ2 με ρ 1 , ρ2 > Ο και zo γνωστό μιγαδικό αριθμό.

γ + δ ί κ.λ. π .

Δίνεται ο μιγαδικός αριθμός z = ψί και ψ R και Μ(χ,ψ) η εικόνα του. α) τι ονομάζουμε μέτρο του z και με τι ισούται; β) Πότε θα ισχύει l z l = Ο ; γ) Ποια σχέση συνδέει τα μέτρα των μιγαδι­ κών αριθμών z, - z, z, - z ; χ,

Πέντε μιγαδικοί αριθμοί έχουν ίσα μέτρα. τι συμπεραίνεται για τις εικόνες τους; Να γράψετε τις ιδιότητες του μέτρου των μι­ γαδικών αριθμών. Δίνονται οι μιγαδικοί αριθμοί z 1 , Zz και τα ση­ Μ( z1 ), Μ(z2 ) που είναι οι εικόνες τους α) Να δείξετε ότι: l z1 · Z 2 I = I zι l · l z 2 1 · β) Να συμπληρωθεί η ισότητα: (Μ 1 Μ 2 ) = .... γ) Να συμπληρωθεί η ανισότητα

+

Ά σ κη ση 1η 3+i 2-i Δίνονται οι μιγαδικοί αριθμοί z 1 = -+ -i

2+i

και z 2 = (x + ψi ) · i με χ,ψ E R . α) Να γραφούν οι z1 και z2 στη μορφή α+βί. β) Να βρεθούν οι χ, ψ όταν z1 = !z 2 •

5 γ) Να βρεθούν οι χ, ψ όταν 5z1 = z2 •

ΕΥΚΛΕΙΔΗΣ Β ' λη ' τ.l/65


Μαθηματικά Γ Λυκείου

δ) Να βρεθούν οι χ, ψ όταν ο μιγαδικός αριθ­ Επίσης δίνεται ο μιγαδικός w = z με z ;t: -4 . z +4 μός u = Sz1 + z2 είναι: α) Να γραφεί ο w στη μορφή α+βί. i) πραγματικός αριθμός. ii) φανταστικός αριθμός. β) Να βρεθεί ο γε�μετρικός τόπος των σημεί­ ων Μ όταν: Λύ ση i) ο w είναι φανταστικός. α) Είναι: ii) ο w είναι πραγματικός. 3+i 2-i = (3+i) · i + (2-i)2 = = --

z

--

2+ί ί2 (2+ί)(2-ί) 8 19 . = ... = - - -1 5 5 Επίσης: z = (χ + ψί)ί = xi + ψί2 = -ψ+ xi . 2 1 <=> --�ι 8 19 . =-(-ψ+ 1 χι). <::> β) Επίσης z =-z 5 2 5 5 5 <:::> � - 1: i = - � + � i <:::> - � = � χ 19 <:::> - = -5 5 = -8 <:::> ψχ =-19 } οι ζητούμενοι 19 ι· J = -ψ- χι. <=> γ) Ακομη 5 z = z2 <:::> 5 ( 58 - S <::> 8-19ί = -ψ-χί <::> 8-19= -ψ } <:::> ψχ == 19-8} οι I

+ --

Ϊ

1

,

ζητούμενοι.

8 19 · J + (-ψ+ χι) = ---ι ( 2 5 5 = 8-19i-ψ+ xi = (8 -ψ)+ (χ -19)i οπότε:

i)

"

u = 5 zI + z = 5

Αν

u

Ε

R

πρέπει:

= 0 <:::> χ -19 =0 <:::> χ = 19 . Δηλαδή αν χ 19 και ψ R ο u είναι lm(u)

=

ii)

z -4 <:::> χ + ψί -4 <:::> (χ, ψ) (-4,0) (1). ;t:

Ε

πραγματικός. Αν u Ε R πρέπει: Re( u) =

z χ + ψί = z+4 χ + ψi+4 (χ +ψi) · [(χ + 4)-ψi] [(χ +4) + ψί] . [(χ + 4) -ψί] χ2 +4χ + ψ2 + 4ψ ι. (χ +4)2 + ψ2 (χ +4)2 +ψ2

w = -- =

-----,----'"7"

η ζητούμενη μορφή του w με

ψ 8 =

χ

Ε

R

ο

u

β)

+4χ + ψ2 α = (χχ2 +4)2 +ψ2 i)

Αν

και β =

4ψ . (χ + 4)2 + ψ2

W E I <::> Re(w ) = O <::> α = O <::>

<:::> χ2 +4χ + ψ2 =0 <:::> χ2 +4χ + 4+ψ2 = 4 <:::> (χ + 2)2 + ψ2 = 4 οπότε ο ζητούμενος

γεωμετρικός τόπος των σημείων Μ είναι τα σημεία του κύκλου με κέντρο το και Εξαιρείται το σημείο ακτίνα ρ λόγω της Αν w Ε R Im( w ) = Ο β = Ο =Ο = Ο οπότε ο ζητούμενος γε-

2. (1). ii) <:::> <:::> 4ψ <:::> ψ =

<:::>

Κ(-2,0) (-4,0) <:::>

ωμετρικός τόπος των σημείων Μ είναι τα σημεία της ευθείας Ο δηλαδή του άξονα Εξαιρείται το σημείο λόγω της

Ο <:::> 8 -ψ = Ο <:::> ψ = 8 .

Δηλαδή για και νταστικός αριθμός;

;t:

;t:

Έτσι

_

= -Χ

δ) Είναι:

α) Είναι

}

(α)

I

Λύ ση

είναι φα­

χ'χ.

ψ

=

(-4,0)

(1).

Μ Ε ΘΟΔΟ Σ l α) Όταν ζη τείται ν α γρ ά ψουμε ένα μιγαδικό α­

Ά σ κη σ η 2η

ρι θμό ς στ η μορφή α+βί τ ότε κ άν ουμε τις

Δίνεται ο μιγαδικός αριθμός z = χ + ψί με χ, ψ Ε R και Μ(χ,ψ) η εικόνα του z.

π ρ άξεις όπ ως έχουν οριστεί στη θ εωρία.

ΕΥΚΛΕΙΔΗΣ Β ' λη ' τ.l/66


,

Μαθηματικά Γ ' Λυκείου

-1 = -1 = -1 = - 1 Α = .1 - 2 = ι · ( -ι ) -ι· 2 ι·2 - 1 υ 2 -1 = --1 · i = -i = ι. Α= 1-2 =. (-1) (-i) i i 2 -1 -1 = -1 = 1 . 1-2 =υ=3 Α= (-i) · (-i) i 2 -1 i . (i - 1) 2006 = i . (i - 1) 2004 . (i - 1) 2 = β) ' (i + 1) 2004 (i + 1) 2004 γ) Όταν ζη τείται να δείξουμε ότι z1, z2 είναι συ­ .ι - 1 2004 · . ζυγείς ή ζη τούνται οι συνθήκες ώστε να είναι = ι. · · (ι 2 - 2ι + 1 ) = . ι +-1 συζυγείς τότε διαπιστώνουμε ότι α γ και β . 2004 = -δ ή απαιτούμε α = γ και β = -δ απ ' όπου = ι. (ι - 1) 2 . ( -i - 2"ι + j,) = + 1)(i - 1) (i προκύπτουν οι ζη τούμενες συνθήκες. 2004 ( 2004 2 2 1 2i+ 1 i -1i+ . . 2 i ( -2i ) � -2i ) δ) Όταν ζητείται να δείξουμε ότι ο z = α + βί εί­ ι -1 -1-1 ναι πραγματικός αριθμός ή φανταστικός ή -2ι. 2004 = 2 · ι· 2004 = 2 · 1 = 2 = 2 · . ζητούνται οι συνθήκες ώστε ο z να είναι _2 πραγματικός ή φανταστικός τότε διαπιστώ­ νουμε ότι β = Ο ή α = Ο αντίστοιχα ή απαι­ ΜΕΘΟΔΟΣ 2 τούμε β = Ο ή α = Ο απ ' όπου προκύπτουν οι Όταν ζητείται να υπολογίσουμε παραστάσεις με ζητούμενες συνθήκες (ή οι ζητούμενοι γεω­ ακέραιες δυνάμεις του i τότε για τον θετικό α­ κέραιο ν η δύναμη ίν = μετρικοί τόποι). β) Όταν ζητείται να δείξουμε ότι δύο μιγαδικοί αριθμοί z1, z2 είναι ίσοι ή ζητούνται συνθή­ κες ώστε να είναι ίσοι τότε γράφουμε τους z1, Ζ2 στη μορ φή z 1 = α + βί, z2 = γ + δί και διαπιστώνουμε ότι: α = γ και β = δ ή απαιτούμε α = γ και β = δ απ ' όπου προκύπτουν οι συν­ θήκες ώστε να είναι ίσοι.

=

-"

για

=

είναι:

για είναι: Επισης:

( (

(

Παρατήρηση Ο z ε R <::> z = z

)

)

( )

=

)

ι,

ο z ε ι <=> z = -ι

ί4π+υ

= ίυ = i, -1, -i,

)

(

για για για για

=

υ=Ο υ=l υ=2 υ=3

α) Να υπολογίσετε την τιμή της παράστασης όπου π, υ ακέραιοι 2004 6 Α = i ν++ 2i για τις διάφορες τιμές του Άσκηση 4η I Να λυθούν στο σύνολο C οι εξισώσεις: θετικού ακεραίου ν. α) (3 + 2i) · z - 4i = 4 + i · z . 6 β) Να δείξετε ότι: i (i - 1)200 = 2 . β) z 2 - 2z + 3 = 0 (i + 1) γ) 2z + z = 4 - 3ί Λύση δ) l z l 2 + z - z = 34 + 6i 0 1 α) i 2004 = i4'5 = l,i 6 = i4 i 2 = 1 · (-1) = -1 Λύση ν=4π +u ·4 π+ ι· ν+3 = ι• ν · ι· 3 ι υ · (-ι" ) = ι• υ · ( -ι ) ' α) (3 + 2i) · z - 4i = 4 + iz <=> Ο:!>υ<4 -1 = -1 = -i = -ι. <=> (3 + 2i) · z - i z = 4 + 4i <=> (3 + i) · z = 4 + 4i Α= 1-2 =υ=Ο ' ·2 . . 1 · (-ι) -ι ι ι <::> z = 4 + 4i <=> z = (4 + 4i)(3 - i) <=> ... 3+i (3 + i)(3 - i) υ=1 •

3

2004

Είναι:

για για

=

ειναι: είναι:

"

-"

οποτε:

Είναι:

--

ΕΥΚΛΕΙΔΗΣ Β ' λη ' τ.l/67


Μαθηματικά Γ' Λυκείου

' . � z= S8 + S4.ι η λυση α) Δίνεται ο z = x + ψi όπου x = 3 + 2i και β) Για την εξίσωση: z2 -2 · z + 3 =Ο έχουμε ψ = 6 + Να βρεθεί το μέτρο του Δ = β 2 -4αγ = (-2)2 -4 · ι·3= 4-ι2 = -8 β) Αν ισχύει: οπότε 2± 2i..fi - ι _+ ι· ..fi χ + ψι. = ((3 + J5i)s) · ( νι - ι·)(ι +ν ιι ) _ zι.z -_ -β ±i2αH _- 2±i.J8 2 2 J5 + 3i δηλαδή Ζ1 = ι- ..fii ή Ζ2 = ι+ ..fii Οι λύσεις. με χ,ψ Ε να βρεθεί η τιμή τη ς παράστα­ γ) Έχουμε την εξίσωση: 2z + z =4-3i (ι). σης Α= �ΧΖ + ΨΖ Θέτουμε z= x + ψi με χ,ψΕR τότε z = χ -ψί και η ( 1 ) γράφεται: 2(χ -ψί) + χ +ψί = 4-3i � 3χ -ψί =4-3i α) Είναι: χ = 3 + 2i και ψ = 6 + 9i οπότε: z =χ+ ψί = ( 3 + 2i) + ( 6 + 9i ) i = 3χ =4} χ = 34 � -ψ=-3 3 + 2i + 6i -9 = -6 + 8i ψ=3 2 l z l=�( οπότε: -6) +82 =�36+64 = νΓιΟΟ = 1 4 ' ' Ά ρα z= + 3ι η λυση. β) Για το μιγαδικό αριθμό z = χ + ψί είναι J δ) Έχουμε την εξίσωση: l z l 2 + z-z = 34 + 6i ( 1). l zl = �χ2 + ψ2 οπότε Α= l zl Αν θέσουμε z = χ + ψί με χ, ψ Ε R τότε 'Εχουμε χ + ψι· = (3+J5i)s ( ν - ι· ) ( ι + ν ι ) l zl 2 =χ 2 +ψ2 και z-z = 2ψί οπότε η ( 1 ) γράφε­ ( J5 +3i ) } =34 � ται: χ 2 +ψ2 +2ψi=34+6i� χ2 +ψ22ψ=6 χ 2 + 9=34} � χ2 =25} � χ =±5 � ψ=3 ψ=3 ψ=3 Άρα z1 = -5 + 3i ή z2 = 5 + 3ί οι λύσεις της Άσκηση Sη

z.

9ί .

<Ο

5

r;; 7

r;; 7.

JR

Λύ ση

0

5

ι;:; Ί ι

ι;:; 7. ι

( ι );

ΜΕθΟΔΟΣ 3

Όταν έχουμε να λύσουμε εξισώσεις στο C τότε:

α) Για τη μορφή α · z = β με α , β ε R και α :;t: Ο παίρνουμε z = 1!_ ως λύση. α β) Για τη μορφή αz 2 + βz + γ = Ο με α :;t: Ο και α,β,γ

ε

R εργαζόμαστε με τη διακρίνουσα

κατά τα γνωστά από τη θεωρία. γ) Όταν στην εξίσωση υπάρχουν δυνάμεις του z (π.χ. z 2 ή z3 κ.λ.π.) το μέτρο του z ή ο συζυγής του z τότε θέτουμε z = χ + ψί , με χ, ψ ε JR κάνουμε τις πράξεις και από την τελική μορ­ φή της εξίσωσης υπολογίζουμε τα χ, ψ άρα και τον z.

ΜΕθΟΔΟΣ 4 Όταν ζητείται το μέτρο ενός μιγαδικού ή μιας παράστασης μιγαδικών τότε: α) φέρνουμε το μιγαδικό στη μορφή α + βί οπό­ τε το μέτρο του ισούται με �α 2 + β 2 β) εφαρμόζουμε στην παράσταση τις ιδιότητες του μέτρου μιγαδικών. Άσκηση 6η

Αν για του μιγαδικούς αριθμούς l z ι Z z l l z 1 Z z l να δείξετε ότι:

+ = -

ΕΥΚΛΕΙΔΗΣ Β' λη ' τ.l/68

z ι,

Zz ισχύει


Μαθηματικά Γ Λυκείου

Λύ ση

Για του μιγαδικούς zι , z2 ισχύει: l z , + z 2 1 = lz , - Ζ2 1 οπότε lz, + z2 12 = lzι - z2 12 � {z 1 + z 2 ) · { z1 + z2 ) = {z 1 - z 2 ){ z1 - z2 ) � � � + Ζι · Ζ2 + Ζι · Ζ2 + � = = � - Ζι · Ζ2 - Ζι · Ζ2 + � � � 2z ι . z2 + 2zl . z 2 = ο � { 1) l z ι l 2 + l z 2 1 2 = lz ι - z z l 2 <=> <::::> Ζ 1 Ζ + z 2 · Ζ2 = ( Ζ 1 - Ζ2 ){ z1 - Ζ2 ) <=> � + � = = � - Ζι · Ζ2 - Ζι · Ζ2 + � <=> <=> Ζι . z2 + Ζι . Zz = ο (1)

Έστω:

που ισχύει λόγω της άρα ισχύει και η αρχική Άσκηση 7η

Αν για το μιγαδικό z ισχύει: l z - 21 = 3 τότε να δείξετε ότι: l 3z - 91 = l z - 11 1 . Λύση

Ισχύει: l z - 21 = 3 <=> l z - 212 = 32 <=>

Παρατήρηση / z / 2 = 1 <=> z · z = 1 l zl = 1 ο ' z = =1 ' -z = -1 z z ιr ( z )I lf {z)l 2 = α 2 <=> f {z) · f {z) = α2 - α2 2 α ) = -= f {z) = f( z f {z) f {z)

Αν ισχύει τότε ποτε: η Γενικότερα αν =α τότε ή

οπότε:

Άσκηση 8η

Αν η εικόνα του μιγαδικού z1 ανήκει στον κύκλο χ 2 + ψ 2 = 4 και ο z 2 = 4 + 3ί να βρεθεί η ελάχιστη και η μέγιστη τιμή του l zι + z 2 1 · Λύ ση

εικόνα του z ανήκει στον κύκλο 2 + ψ2 = 4 που έχει κέντρο το Ο) και ακτίνα ρ 2 οπότε: lz ι l = 2 Επίσης για τον z2 = 4 + 3i είναι: lz 2 i = .J42 + 32 = ../25 = 5 Όμως: llzι l - lzz ll l zι + Zz l lzι l + l zz l ή l2 - 5l s l z 1 + z 2 l s 2 + 5 ή l-3l s l z 1 + z 2 l s 7 οπότε η ζητούμενη ελάχιστη τιμή του lz1 + z2 1 είναι το 3 και η μέγιστη το 7. Η

1

χ

=

0(0,

S

S

{z - 2 ){ z - 2 ) = 9 <=> z · z - 2z - 2z + 4 - 9 = ο <=> (1) <=> z . z - 2 z - 2z - 5 = ο Άσκηση 9η l3 z - 91 = l z - 1 1 1 <=> l 3z - 91 2 = l z - l ll 2 <=> Να βρεθεί η ελάχιστη και μέγιστη τιμή του l z l <=> { 3 z - 9){ 3z - 9) = ( z - 1 1 ){ z - 1 1 ) <=> όταν l z + 3i l = 5 . <=> 9z · z - 27 z - 27z + 81 = Λύση = z . z - 1 1 z - 1 1z + 121 <=> <=> 8z · z - 16 z - 16z - 40 = Ο <=> l z + 3il = 5 (1) <=> z · z - 2 z - 2z - 5 = Ο (1) z = z + 3i - 3i (2) Ιz l = I( z + 3i) - 3il = 1( z + 3i) + (-3i )I ΜΕΘΟΔΟΣ 5 ll z + 3il - l-3ill s I { z + 3i) + { -3i )I s Όταν ζητείται να δειχθεί μια ισότητα μέτρων

Έστω

άρα ισχύει και η αρχική .

που ισχύει λόγω της

με την ιδιότητα

/ f (z)/ 2 f( z ) f( z ) =

=

οπότε

Όμως είναι:

τότε υψώνουμε στο τετράγωνο και εφαρμόζου­

l z/2 z

Έχουμε: Επίσης:

·

i ή γενικότερα

{Ι)

s lz + 3il + l-3il<=> (2)

·

ΕΥΚΛΕΙΔΗΣ Β' λη ' τ.l/69


Μαθηματικά Γ Λυκείου

� 1 5-31 � l zl � 5 +3 � 2� l zl � 8 οπότε η ζητού­ μενη ελάχιστη τιμή του l zl είναι το 2 και η μέγιστη το 8. Άσκηση lΟη

Επειδή οι εικόνες του z ανήκουν στην ευθεία ε προφανώς το l zl δεν παίρνει μέγιστη τιμή . δ) Είναι λ =- Α = _± . Αν ΟΜ ε τότε Β 3 , 1 3 οποτε η ε ισωση της ΟΜ ειναι: λ = --=λ. 4 ΟΜ:ψ=-43 Χ . Από το σύστημα της ε και της ΟΜ έχουμε: 4χ+3ψ+10=0 χ =- �5 8 6. αρα z=----ι 3 5 5 ψ=-4 Χ ψ=--56 ο ζητούμενος μιγαδικός

γ)

ε

οΜ

Δίνεται ο μιγαδικός αριθμός

α) Να βρεθεί ο γεωμετρικός τόπος των εικόνων του z. β) Να βρεθεί η ελάχιστη τιμή του lzl γ) Το l zl παίρνει μέγιστη τιμή; δ) Να βρεθεί ο μιγαδικός z με το ελάχιστο μέτρο.

l_

_

,

}

ξ'

}

'

Άσκηση l lη

'Εχουμε τον μιγαδικό z = (�- 4) + ( 2- � } με Για τον μιγαδικό αριθμό z ισχύει: l z - 3il = 2 . α) Να βρεθεί ο γεωμετρικός τόπος των εικόκεffi. νων του z. α) Έστω z =χ+ ψί με χ, ψ JR και Μ(χ, ψ) οι ειβ) Να βρεθεί η ελάχιστη και η μέγιστη τιμή κόνες του z τότε θα είναι: του lz l χ = � -4 4χ+16=κ} � 6-3ψ = κ α) Έχουμε l z -4-3il =1 � J z- ( 4+3i )J = 2 οπότε ψ = 2- � ο ζητούμενος γεωμετρικός τόπος των εικόνων οπότε: 4χ+}6=6-3ψ ή 4χ+3ψ+10=0 z είναι τα σημεία του κύκλου με κέντρο το Δηλαδή ο ζητούμενος γεωμετρικό τόπος των του 3) και ακτίνα ρ = 2. σημείων Μ είναι τα σημεία της ευθείας β) Κ(4, Λόγω του (α) είναι ε: 4χ + 3ψ+ 10 =Ο (OK)= J42 +32 = .J25 =5 β) Λόγω του (α) η ελάχιστη τιμή του l z l ισούται με την απόσταση το Ο από την ε. Λύ ση

4-

ε

}

Λύ ση

� -

Υ

Υ

ε

ο

/ /

χ

χ

'

χ

y'

y'

Οπότε η ζητούμενη ελάχιστη τιμή του l zl είναι: ( ΟΚ ) -ρ=5-2=3 και η μέγιστη: ( οκ ) + ρ= 5 + 2 = 7 Ε ΥΚΛΕΙΔΗΣ Β ' λη ' τ.l/70


Μαθηματικά Γ Λυκείου

ΜΕΘΟΔΟΣ 6 Σε ασκήσεις που ζητείται η ελάχιστη και η μέ­ γιστη τιμή μέτρου μιγαδικο ύ χρησιμοποιούμε την τριγωνική ανισότητα

l l z ι l - l z z ll l z ι Zz l l z ι l + l z z l �

±

I

χ 2 = συν 2 ω (+) 2 2 42 � � + .!... = συν 2 ω + ημ 2 ω ή 2 2 .!.2 .. = ημ2 ω 4 3 32 χ 2 + ψ2 = 1 . 42 y

Δηλαδή ο ζητούμενος γεωμετρικός τόπος των εικόνων Μ(χ, ψ) του z είναι έλλειψη. Άσκηση 12η β) Είναι: l zl 2 = χ 2 + ψ2 και z 2 + 'Z2 = ( χ + ψi) 2 + ( χ - ψί)2 = Δίνεται ο μιγαδικός αριθμός z = χ + ψί με = χ 2 -ψ2 + ?Ψι + χ 2 - ψ2 - ?Ψι = 2χ 2 - 2ψ2 χ, ψ Ε � και Μ(χ, ψ) οι εικόνες του z. Να βρεθεί ο γεωμετρικός τόπος των εικόνων του z όταν: οπότε η ισότητα που δόθηκε γράφεται: χ 2 + ψ2 + 2χ 2 - 2ψ 2 - 3 = ο <::> α) z = 4συνω + 3ημω · i με ω Ε [ 0,2π ) 3χ 2 - ψ 2 = 3 <::> χ 2 _ .!_2 = 1 3 Δηλαδή ο ζητούμενος γεωμετρικός τόπος των Λύ ση εικόνων Μ(χ, ψ) του z είναι υπερβολή. α) Για ω Ε [ 0,2π ) είναι: ή τη γεωμετρική ερμηνεία του μέτρου μιγαδικού.

}

}

i

χ = 4συνω <::> = συνω οποτε: , ψ = 3ημω -ψ = ημω 3

3i

Μαθημ ατι κ ά Γε νι κ ής Παι δ εία ς Α ΝΑΛ ΥΣΗ

Χρυσταλλένης Κυριακοπούλου - Κυβερνήτου 1.

Να βρείτε το σύνολο ορισμού των συναρ­ τήσεων: α) f(x) = -x- + .Jx 2 - 1 χ-3 β) g(x) = .

{χχ2--31 οο <=> {(χχ ,-31)(χ + 1) ο <::> '" 3 ('> (χ 1 ή χ -1) { Άρα, το σύνολο ορισμού της f είναι: :;t:

Α = (-οο,-1] υ [1,3) υ (3, +οο)

Λύ ση

α)

:;t:

συνάρτηση f ορίζεται για τους αριθμούς β) Η συνάρτηση g ορίζεται για τους αριθμούς χ Ε � , για τους οποίους ισχύουν: χ Ε � , για τους οποίους ισχύουν:

Η

ΕΥΚΛΕΙΔΗΣ Β' λη ' τ.lΠl


{

Μαθηματικά Γ Λυκείου

{

χ + 1 :;e O χ+1*0 � � 2-χ (2 χ)(χ + 1) ;::: ο ;::: ο χ+1 χ :;e -1 � � (χ - 2)(χ + 1) � ο

2.

--

{

Άρα, το σύνολο ορισμού της g είναι: Β = (-1, 2] (μέσο) . Έο:τω η συνάρτηση f(x) = e 2 x - 2ex + 3 . α) Ο αριθμός 1 είναι τιμή της f; β) Ο αριθμός 2 είναι τιμή της f; Λύση

1) Η

ζάχαρη που πούλησε τις 9 πρώτες ημέρες εί­ ναι: f (9) 60 9 - 1-1- = 60 8 + _!_ = =

f(x) = 1 � e 2 x - 2ex + 3 = 1 � � (ex ) 2 - 2ex + 2 = 0 (1).

εξίσωση ( 1 ) είναι δευτέρου βαθμού ως προς ex και η δακρίνουσά της είναι Δ =4-8=-4 <0. Άρα, η εξίσωση ( 1 ) είναι αδύνατη και συνεπώς ο αριθμός 1 δεν είναι τιμή της f. β) Ο αριθμός 2 είναι τιμή της f αν, και μόνο αν, υπάρχει χ ε � με: Η

f(x) = 2 <=:> e 2 x - 2ex + 3 = 2 � e 2 x - 2ex + 1 = 0 � (ex - 1) 2 = ο � ex = 1 � ex e0 � χ = ο . Άρα f (Ο) = 2 και συνεπώς ο αριθμός 2 είναι =

(

9+1

) ( 10 )

κιλά. 2) Η ζάχαρη που πούλησε την 4η και την 5η ημέρα μαζί είναι: f(5) - f(3) = 60 4 + 60 2+ =1 15 κιλά. = 480 + 6 = 486

( �)- ( l)

3) Η ζάχαρη που πούλησε την 3η ημέρα είναι:

( l) - 6o (1 + �) = 55 κιλά.

f(3) - f(2) = 60 2 + 4.

Το σύνολο ορισμού της f είναι το � . α) Ο αριθμός ι είναι τιμή της f, αν και μόνο αν, υπάρχει χ ε � , με:

3.

Λύ ση

Μία περιβαντολογική μελέτη μιας πόλης, με p χιλιάδες κατοίκους, έδειξε ότι το μέσο ημερήσιο επίπεδο του μονοξειδίου του άν­ θρακα στον αέρα είναι f(p) = O,Sp + 1 μέρη ανά εκατομμύριο. Εκτιμάται ότι t χρόνια από τώρα ο πληθυσμός της πόλης αυτής θα είναι p(t) = 10 + O,lt 2 χιλιάδες άτομα. α) Να εκφράσετε το επίπεδο του μονοξειδί­ ου του άνθρακα στον αέρα συναρτήσει του χρόνου. β) Να βρείτε πότε το επίπεδο του μονοξει­ δίου του άνθρακα θα φθάσει τα 6,8 μέρη ανά εκατομμύριο. Λύση

α) Το επίπεδο του μονοξειδίου του άνθρακα στον τιμή της f. αέρα συναρτήσει του χρόνου είναι: f ( p (t) ) = f( IO + 0, 1t 2 ) = 0, 5(ιΟ + 0, 1t 2 ) + ι = Το βάρος της ζάχαρης σε κιλά που πουλάει = 6 + Ο, 05t 2 (μέρη ανά εκατομμύριο). ένα νέο σούπερ μάρκετ συναρτήσει των η­ μερών λειτουργίας του t είναι: β) Το ζητούμενο θα συμβεί όταν:

(

f(t) = 6o t - ι + -1t+l

)

Να βρείτε πόσα κιλά ζάχαρη πούλησε: 1) Τις 9 πρώτες ημέρες. 2) Την 4η και την Sη ημέρα μαζί. 3) Την 3η ημέρα.

f ( p (t) ) = 6, 8 � 6+0,05t 2 =6, 8 � 0, 05t 2 = 0,8 � 5t 2 = 80 � t 2 = 16 � t = 4 (γιατί t � Ο ).

Άρα, το ζητούμενο θα συμβεί μετά από 4 χρόνια.

ΕΥΚΛΕΙΔΗΣ Β ' λη ' τ.1Π2


Μαθηματικά Γ Λυκείου

5

+

. 'Εστω η συνά ρτηση: f( χ ) - χ2 -Γ3χ 2 • νχ - 1 α) Να βρείτε το σύνολο ορισμού της f. β) Να βρείτε το όριο L = limf(x) . χ

-->1

συνάρτηση f ορίζεται για τους αριθμούς χ ε � , για τους οποίους ισχύουν: χ�ο χ�ο χ�ο

Η

{fx-1:;t0 <=> {fx:;t1 <=> {χ � 1

Άρα, το σύνολο ορισμού της f είναι: Α= υ . β) Έχουμε: . · χ - 3χ + 2 = L = Ιιm f( χ ) = ιιm

[0, 1) (1, +οο) 2 Χ-->1 Χ-->1 Fx -1 . (x-l)(x- 2) ( Fx +1 ) = = lιm ( Fx -1 ) ( Fx + 1 ) .χ-->1 ( x-1)(x -2) ( Fx +1 ) = = lιm χ -1 = �� [ < χ - 2) ( Fx + 1 )] = -2

Χ-->2 (

Χ-->1

Λύση

Έχουμε: L = Ι.ιm

1 1

1 χ-->2 + χ-->2 L 2 - ιm-Χ-->2 3χ--26 -- ιΧ-->ιm· 2 3(χ--22) -- ι·Χ-->ιm2 3 -- 3 . ( ν'χ + 3 - 2){ ν'χ +3 + 2) = L3 = lιm <χ-υ ( ν'χ+3 +2) = lim 1 = 1 χ-->Ι ν'χ+3 +2 4 - ι·

...... ι

χ - 2 = 1ιm . -- = (χ - 2)(χ 2) χ+2 4 Χ

...... ι

Σε κάθε μία από τις παρακάτω συναρτή­ σεις, της πρώτης στήλης να aντιστοιχίσετε την παραγωγό της στη δεύτερη στήλη: (α) συν2χ (1) e (β) e (2) Ο 2 (γ) ημ χ (4) -4ημχσυνχ (δ) eημχ (5) συν 2 χ (6) eημχ συνχ . •

Να βρείτε τα παρακάτω όρια: χ-2 6 3χ - -L 1 = ι.ιm -L = 1.ιm -2 χ-->2 χ2 - 4 Χ-2 Χ-2 2 - J;. L = ι.ιm Γχ+3 - 2 L = ι·ιm χ Γ 3 4 Χ-1 '\j Χ - 1

Χ-->1

...... ι

7.

χ-> !

6.

1.

χ-> Ι

ΛίJση

α)

+ 3 - 4 = ιm---:--:χ -1==-----,......ι (χ-1) ( ν'χ+3 +2) ......1 (χ-1) ( ν'χ+3 +2) . ( χ 2 - Fx) · ( Fx +1 ) { χ 2 + Fx) L4 = lιm ( Fx -1 )( Fx + 1)( χ 2 + Fx ) . 1 (χ 4 - υ ( Fχ +1 ) lιm. χ(χ 3 - υ ( Fχ + 1 ) = = lιm ...... (x-1) ( x 2 + Fx ) ( x -1) ( x 2 + Fx) . χ(χ - 1)(χ 2 + x + l) ( Fx +1 ) = lιm (x-l) ( x 2 + Fx) = .χ-->1 χ(χ 2 + χ + l) ( Γχ +1 ) == -1·3·2 =3 = lιm 2χ + Fx 2 χ

. = Ιιm

)

Λύση

·

(α) � 4, γιατί: (συν2χ)' = -(ημ2χ) (2χ)' = = -2ημ2χ = -4ημχσυνχ (β) � (2), γιατί: (e )' = Ο . (γ) � γιατί: (ημ 2 χ)' = 2ημχ(ημχ)' = 2ημχσυνχ = ημ2χ . (δ) � (6), γιατί: ( eημχ )' = eημχ (ημχ)' = eημχ συνχ .

(3),

8. Να βρείτε τις παραγώγους των παρακάτω

συναρτήσεων :

Χ

ημχ 1 συνχ 4) f(x) = (χ 3 - 2χ 2 1)4 2) f(x) =

2

3) f(x) = ex + -Rn--=2-x 5) f(x) = -1'1-

Λύ ση

Έχουμε: 1) = (χ 2 ) ' · Rnx χ (Rnx)' =

f '(x)

ΕΥΚΛΕΙΔΗΣ Β' λη ' τ.1Π3

+2

·

+

+


Μαθηματικά Γ Λυκείου

Έτσι, έχουμε: = 2xRnx + χ 2 · ..!.. = 2xRnx + χ χ (1) � ι! · Ο + (λ2 + 1) · 2 + λ(-5) + λ = ο � συνχ)' + -ημχ(1 συνχ) + (ημχ)'(1 = 2) f'(x) = 2λ2 + 2 - 5λ + λ = 0 � 2λ2 - 4λ + 2 = 0 � (1 + συνχ) 2 λ2 - 2λ+1=0 � (λ - 1)2 = 0 � λ - 1 = 0 � λ = 1 = συνχ(l + συνχ) - ημχ(-ημχ) = (1 + συνχ) 2 10. Έστω η συνάρτηση f(x) = xRnx . _ συνχ+συν 2 χ+ημ 2 χ = 1+συνχ = 1 α) Το σύνολο ορισμού της f είναι: (1 +συνχ) 2 (1 +συνχ) 2 1 + συνχ Α : !R.B : (Ο,+οο).Γ : 1R -{Ο}.Δ : (1,+οο) . ' ex ex 2 = = ' · (x )' 2x 3) f'(x) β) Η γωνία που σχηματίζει η εφαπτομένη ε 4) f'(x) = 4(χ3 - 2χ 2 + 1)3 . (χ3 - 2χ 2 + 1)' = της γραφικής παράστασης της f στο ση­ = 4(χ3 - 2χ 2 + 1)3 · (3χ 2 - 4χ) . μείο της ( 1,f(l) ) με τον άξονα χ'χ είναι: 1 π Γ : -. π Δ : -π . (1 + Rn 2 x)' = 2Rnx · (Rnx)' = S) f'(x) = Α : Ο. Β : -. 2.J1 + Rn 2 x 2.J1 + Rn 2 x 6 3 4 . 1 Να δικαιολογήσετε την απάντηση. (Rnx) · � Rnx = =� · Λύ σ η .J1 + Rn 2 x χ · .J1 + Rn 2 x α) Το σύνολο ορισμού της f είναι το διάστημα 9. Έστω η συνάρτηση: f(x) = συν2χ + συνχ . (0, +οο) . Σωστό είναι το Β. α) Να δείξετε ότι: 4f(x) + f"(x) = 3συνχ , β) Η συνάρτηση f είναι παραγωγίσιμη στο για κάθε χ Ε IR (Ο, +οο) με: β) Να βρείτε την εξίσωση της εφαπτομένης f'(x) = (x)'Rnx + x(Rnx) ' = Rnx + χ . ..!.. = Rnx + 1 . ε της γραφικής παράστασης της f στο χ σημείο της ( O,f(O) ) . Έστω ω η γωνία που σχηματίζει η εφαπτομένη γ) Να βρείτε τυς αριθμούς λ Ε IR , για τους ε της Cr στο σημείο (1, f(l)) με τον άξονα οποίους ισχύει: χ ' χ.Τότε: 3 2 λ f'(Ο) + (λ + 1)f(O) + λf"(Ο) + λ = Ο (1). εφω = f'(l) = Rn1 + 1 = 1 => εφω = 1 => ω = � . 4 Λύ ση Σωστό είναι το Δ. Το σύνολο ορισμού της f είναι το R. α) Έχουμε στο IR : 11. Έστω η συνάρτηση: f'(x) = -2ημ2χ - ημχ και f(x) = 2χ3 - 4χ 2 + χ + 2 . f"(x) = -4συν2χ - συνχ . Να βρείτε: Έτσι, έχουμε: 4f(x) + f"(x) = α) Την εξίσωση της εφαπτομένης ε της = 4συν2χ + 4συνχ - 4συν2χ - συνχ = 3συνχ . γραφικής παράστασης της f στο σημείο β) Έστω y = αχ + β η εξίσωση της εφαπτομένης ε. της ( O,f(O) ) . Έχουμε: α = f'(O) = -2ημ0 - ημ0 = 0 , οπότε β) Τις εξισώσεις των εφαπτομένων της y =Ο· χ+β. γραφικής παράστασης της f, οι οποίες Επειδή ε διέρχεται από το σημείο είναι κάθετες στην ε. ( O,f(O) ) = (0, 2) , έχουμε 2 = Ο · 0 + β , δηλαδή Λύ σ η β = 2. Άρα, η εξίσωση της εφαπτομένης ε είναι: Η συνάρτηση f είναι ορισμένη και παραγωγίσιμη y = 2. στο IR με: f'(x) = 6x 2 - 8x + 1 . γ) Έχουμε: f(O) = 2,f'(O) = Ο και f"(O) = -5 . ---

---;=

--;===

ΕΥΚΛΕΙΔΗΣ Β ' λη ' τ.1Π4


Μαθηματικά Γ Λυκείου

α) Έστω y = αχ + β η εξίσωση της εφαπτομένης ε. Έχουμε: α = f'(O) = 1 , οπότε: y = χ + β . Επειδή η ε διέρχεται από το σημείο (O,f(0) ) = (0, 2) , έχουμε 2 = Ο + β , δηλαδή β = 2. Άρα, η εξίσωση της εφαπτομένης ε είναι: y = χ + 2 . β) Ο συντελεστής διεύθυνσης της ε είναι λ = f'(O) = 1 . Η εφαπτομένη της γραφικής παράστασης της f σ' ένα σημείο της με τετμημένη χ0 ε IR έχει συντελεστή διεύθυνσης λ' = f'(x0) . Έτσι, η εφαπτομένη αυτή είναι κάθετη στην ε αν, και μόνο αν: λλ' = -1 <::::> 1 · f'(x0)=-1 <::::> 6χ� - 8χ 0 + 1=-1 <::::> 3χ� - 4χ0 + 1 = Ο <::::> χ0 = 1 ή χ 0 =

Συμπεραίνουμε ότι η f είναι γνησίως φθίνουσα στο διάστημα ( -1] , γνησίως αύξουσα στο [-1, Ι] και γνησίως φθίνουσα στο [1, +οο ) . Επίσης, ότι η f έχει τοπικό ελάχιστο στη θέση χ = -1 , ίσο με ' εχει ' τοπικο' μεγιστο ' ' στη θεση f(- 1) = -21 και οτι χ = Ι , ίσο με f(l) = .!.. . 2 1 3 . Μία συνάρτηση f είναι ορισμένη και παρα­ γωγίσιμη στο IR και για κάθε χ ε IR , ισχύει: 3 χ f'(x) = χ 2 - 1 +1 α) f( -2004) < f( -1821) Σ Λ Σ Λ β) f(lOO) < f(200) Σ Λ γ) f(x) � f(1), για κάθε χ ε IR Άρα, υπάρχουν δύο ζητούμενες εφαπτόμενες, Τις παραπάνω προτάσεις να χαρακτηρίσετε εκείνες στα σημεία της γραφικής παράστασης ως σωστές (Σ) ή λάθος (Λ) δικαιολογώντας της f με τετμημένες 1 και .!_ 3 την απάντησή σας. Όπως στο προηγούμενο ερώτημα, βρίσκουμε Λύση ότι οι εξισώσεις τους είναι: y = -χ + 2 και Έχουμε στο IR : y = -x + 62 · Ο, αν χ 1 27 2 + χ (χ 1 χ 1)(χ + 1) = Ο, αν χ = Ι f'(x) = 2 = 2 + 1 χ + 1 χ 12. Να βρείτε τα διαστήματα μονοτονίας και < 0, αν χ < 1 τα ακρότατα της συνάρτησης: αφού χ 2 + 1 Ο και χ 2 + χ + Ι Ο , για κάθε χ ε IR χ f(x) = χ2 . (γιατί;). +1 -οο ,

�) .

(

3

>

--

Χ

Λύση

f

I

-OC

1

ο

� - -χ

>

{> >

+

συνάρτηση f είναι ορισμένη και παραγωγίσιμη f / στο IR με: 2 2 2 f( l ) f'( x) = (χ)' · (Χ +1)2 - Χ2· (Χ +1)' = Χ +12 - χ ·22Χ = ε λαχ. (χ + 1) (χ + 1) Ο, αν - 1 < χ < 1 Συμπεραίνουμε ότι η f είναι γνησίως φθίνουσα στο 2 1 χ διάστημα ( -οο, -1] , γνησίως αύξουσα στο [Ι, +οο ) = 2 2 = 0, αν χ = -1 ή χ = 1 (χ + 1) < Ο, αν χ < -1 ή χ 1 και έχει ελάχιστο στο χ = 1 . α) Λάθος, γιατί -2004 < -1821 και οι αριθμοί αυ­ -1 1 + οο I I τοί ανήκουν στο διάστημα ( -οο, -1] , στο οποίο χ ο + ο η f είναι γνησίως φθίνουσα, οπότε: f(-2004) f(-1821) . Η

{>

Ι-ΟC

>

x /r � τ.ε.

τ.μ.

ΕΥΚΛΕΙΔΗΣ Β' λη ' τ.ΙΠS

>


Μαθηματικά Γ Λυκείου

β) Σωστό, γιατί 100

<

με πλευρές τα ευθύγραμμα τμήματα ΜΑ και ΜΒ, να είναι το ελάχιστο δυνατό.

200 και οι αριθμοί αυτοί ανήκουν στο διάστημα [ 1, +οο) , στο οποίο η f είναι γνησίως αύξουσα, οπότε f(100) < f(200) .

Λύση

γ) Σωστό, γιατί η f στο χ = 1 έχει ελάχιστη τιμή, Έστω M(x,y) ένα σημείο της ευθείας ε, οπότε ίση με f(l ), οπότε: f(x) � f(1) , για κάθε y = χ + 1 . Το άθροισμα των εμβαδών των δύο αυχ ε JR .

1 4.

Έστω η συνάρτηση: f(x) = χ2 - 2Rnx - 1 . α) Να βρείτε το σύνολο ορισμού της f. β) Να βρείτε τα διαστήματα μονοτονίας και τα ακρότατα της f. γ) Να δείξετε ότι, για κάθε χ > Ο, ισχύει: χ2 � 1 + 2Rnx .

τών τετραγώνων είναι: (ΜΑ) 2 + (ΜΒ) 2 = = (χ - 2) 2 + (y - 6) 2 + (χ - 4) 2 + (y + 2) 2 = = (χ - 2) 2 + (χ + 1 - 6) 2 + (χ - 4) 2 +(χ+ 1+2) 2 = = 4χ 2 - 16χ + 54. Θεωρούμε τη συνάρτηση: f(x) = 4x 2 - 16x + 54 .

{

Η f είναι ορισμένη και παραγωγίσιμη

στο JR με: > 0, αν χ > 2 f'(x) = 8x - 16 = 8(x - 2) = 0, αν χ = 2 < 0, αν χ < 2

Λύση

α) Το σύνολο ορισμού της f είναι το Α = (Ο, +οο ) . β) Η f είναι παραγωγίσιμη στο Α με:

{

1-α..' ---t-

χ

( �) =

f'(x) = 2x - 2 · = 2 χ -

--

J

--j-- ---

� j->Ξ: - �-

> 0, αν χ > 1 χ2 - 1 = 2 -- = 0, αν χ = 1 χ < Ο, αν Ο < χ < 1

λ ε αχ.

�� +-�� -�Ι 1-.;;-�-

+�

f( l ) λαχ.

ε

Συμπεραίνουμε ότι η f είναι γνησίως φθίνουσα στο διάστημα (Ο, 1 ] , γνησίως αύξουσα στο [ 1, +οο) και παρουσιάζει ελάχιστο στη θέση χ = 1 , ίσο με f(l) = 0 . ·,

γ) Από το προηγούμενο ερώτημα συμπεραίνουμε ότι για κάθε χ > Ο, ισχύει: f (χ) � f (1) � χ 2 - 2Rnx - 1 � Ο � χ 2 � 1 + 2Rnx .

Συμπεραίνουμε ότι η f έχει ελάχιστη τιμή για χ = 2, οπότε y = 2 + 1 = 3 . Άρα, το ζητούμενο σημείο είναι: Μ(2,3). 16.

Δίνεται ένα ευθύγραμμο τμήμα ΑΒ μήκους α > Ο και πάνω σ ' αυτό ένα σημείο Μ. Να βρείτε τη θέση του Μ, για την οποία το ά­ θροισμα των όγκων των δύο σφαιρών με διαμέτρους Μ και ΜΒ είναι το ελάχιστο δυνατό. (Υπενθυμίζουμε ότι ο όγκος μιας σφαίρας με ακτίνα R είναι � πR ) 3

3

ΛίJση

.

15. Σ' ένα καρτεσιανό επίπεδο θεωρούμε την Θέτουμε ΑΜ = χ, οπότε Ο � χ � α και ευθεία ε : y = χ + 1 και τα σημεία Α(2,6) και ΜΒ = α - χ . Το άθροισμα των όγκων των υπόψη 8(4,-2). Να βρείτε το σημείο Μ της ευθείας δύο σφαιρών είναι: ε, για το οποίο το άθροισμα των εμβαδών V(x) = π ; + π α ; χ = των δύο τετραγώνων που κατασκευάζονται

j(I j( Ι

ΕΥΚΛΕΙΔΗΣ Β' λη ' τ.1Π6


Μαθηματικά Γ' Λυκείου

= � (3αχ 2 - 3α2 χ + α3 ). 6 Έχουμε, στο διάστημα (0, α):

(

V'(χ) = (6αχ - 3α 2 ) = απ χ -

χ ν ν

'

ο

ο

�)

= 0, αν χ =

α < 0, αν χ < 2

μεγ.

Συμπεραίνουμε ότι το εμβαδόν του τριγώνου αυα , , μεγιστο , , οταν χ J2 , οποτε: του, ειναι =

α +

-�y/ελαχ.

Συμπεραίνουμε ότι το άθροισμα των όγκων των δύο αυτών σφαιρών γίνεται ελάχιστο όταν ΑΜ = � . οπότε ΜΒ = α - � = � , δηλαδή όταν το 2 2 2 Μ είναι το μέσο του τμήματος ΑΒ . ι 7.

Άρα, το ορθογώνιο και ισοσκελές τρίγωνο είναι εκείνο που έχει το μεγαλύτερο εμβαδόν. 1 8.

Από όλα τα ορθσyώνια τρίγωνα με την ίδια υποτείνουσα, ποw tχ.ει το μεγαλύτερο εμβαδό; Λύση

Θεωρούμε ένα ορθογώνιο τρίγωνο με σταθερή υ­ ποτείνουσα α και κάθετες πλευρές χ και y. Έχου­ με: χ 2 + y 2 = α2 => y 2 = α2 _ χ 2 => => Υ = �α 2 - χ 2 (y > 0). Το εμβαδόν του τριγώνου αυτού είναι: Ε(χ) = ! χ �α� - χ 2 , 0 < χ < α . 2 Έχουμε, στο διάστημα (0, α): 1 I 2 2 1 -2χ Ε (χ) = -να = - χ + -χ 2 2�α 2 - χ 2 2 I

-

Ε Ι /-�-- -

α > 0, αν χ > 2

α/2 ο

α

α!-12

Το κόστος παραγωγής χ ραδιοφώνων σε μία ημέρα είναι Κ(χ) = �χ 2 + 105χ + 75 € 4 και η τιμή πώλησης κάθε ραδιοφώνου είναι Τ(χ) = 1 50 - �χ €. Να βρείτε ποια πρέπει 2 να είναι η ημερήσια παραγωγή ώστε να έχουμε το μέγιστο (συνολικό) κέρδος. Στην περίπτωση αυτή, να δείξετε ότι το μέσο κόστος της ημερήσιας παραγωγής είναι το ελάχιστο δυνατό. Λύση

Κατ ' αρχή θα πρέπει: (χ > Ο, Κ(χ) > Ο, τ( χ) > Ο) <::::> <::::>

( χ > Ο, 150 - % χ > Ο) <::::> Ο < χ < 100 .

α) Τα έσοδα από την πώληση των χ ραδιοφώνων είναι:

( %)

Άρα, το (συνολικό) κέρδος είναι: Ρ( χ) = Ε( χ) - Κ( χ) = α < 0, αν χ > J2

%

Ε(χ) = χ · Τ (χ) = χ ι5ο - χ = Ι 5Οχ - χ 2

(

% ) (% χ2 + Ι Ο5χ + 75) =

= ι5οχ - χ 2 = 45χ - � χ 2 - 75 4

ΕΥΚΛΕΙΔΗΣ Β ' λη ' τ.1Π7

ε


Μαθηματικά Γ Λυκείου

> 0, αν χ > 10 2 - 100 75 x K ' (x) = l - = 3 · = 0, αν χ = 10 4 χ2 4χ 2 < 0, αν χ < 10

αν χ < 10 Ρ'(χ) = 45 - χ = 0, αν χ = 10 < 0, αν χ > ι ο I

1

; pl

β)

{

Έχουμε:

Έχουμε:

0

Γ

10

+

ο

/

μ

r-�

μεγ.

χ

1 00

Κμ' Κμ

Συμπεραίνουμε ότι, για να έχουμε το μέγιστο κέρδος θα πρέπει η ημερήσια παραγωγή να είναι 10 ραδιόφωνα. Το μέσο κόστος της ημερήσιας παραγωγής είναι: Κ(χ) l 75 Κ (χ) = = x + 105 + € 4 χ χ

10

ο

Ο

ε λαχ.

1 00 +

/

Συμπεραίνουμε ότι το μέσο κόστος της ημερή­ σιας παραγωγής είναι ελάχιστο όταν η ημερή­ σια παραγωγή είναι ι Ο ραδιόφωνα, δηλαδή ό­ ταν έχουμε το μέγιστο κέρδος.

μ

Π Ρ Ο Β Λ Ν AIAT Α Α Π ' Τ Ν I( A t!J N AI E, P Ι N N z n N 1.

Ο

Γιώργος είναι ιδιοκτήτης ενός χωραφιού τριγωνικού σχήματος με κορυφές τρεις βράχους Α, Β, Γ. Ο Γιάννης δεν θυμόταν ακριβώς τα μήκη των πλευρών του τριγώνου. Όμως θυμόταν ότι: η ΑΒ είναι

το πολύ 1 km, η ΒΓ είναι το λιγότερο 1 km και το πολύ 2 km και η ΓΑ είναι το λιγότερο 2 km και το πολύ 3 km. Με βάση τις πληροφορίες αυτές που έχετε από το Γιάννη να αποδείξετε ότι το χωράφι του Γιώργου δεν μπορεί να έχει εμβαδό μεγαλύτερο από ι τετραγωνικό χιλιόμετρο. Μπορεί να έχει εμβαδό ι τε­ τραγωνικό χιλιόμετρο; 2.

Μια πλατεία έχει σχήμα κυρτού τετραπλεύρου με περίμετρο Π και εμβαδό Ε. Να αποδειχθεί ότι στη πλατεία αυτή μπορούμε να κατασκευάσουμε κυκλικό συντριβάνι ακτίνας � . π

ΕΥΚΛΕΙΔΗΣ Β' λη ' τ.1Π8


Η Σ ΤΗΛΗ ΤΟ Υ ΜΑ ΘΗ ΤΗ

Επιμέλεια: Αντώνης Κυριακόπουλος - Γιάννης Στρατής Αγαπη τ οί μαθ η τ έ ς

Ο Ευκλείδης Β ', κατ ' αρχή, σας εύχεται καλή χρονιά. Σας εύχεται επίσης τη φετινή χρον!ά να αποκτήσε­ τε πολλές και χρήσιμες γνώσεις. Οι γνώσεις όμως αποκτούνται με σωστή καθοδήγηση και πολύ προσωπικό διάβασμα. Φίλοι μας, μοναδικός σκοπός της στήλης αυτής είναι να σας βοηθήσει να κατανοήσετε καλύτερα τα Μα­ θηματικά και να τα αγαπήσετε. Πρέπει όμως να μας βοηθήσετε και εσείς στέλνοντάς μας απορίες, παρατη­ ρήσεις, ασκήσεις που θέλετε να προταθούν για λύση (με τις λύσεις τους και κάθε μία σε ξεχωριστό φύλ­ λο), λύσεις προτεινομένων ασκήσεων της στήλης (κάθε μία σε ξεχωριστό φύλλο) και (γιατί όχι) γνώμες για τα διάφορα άρθρα του περιοδικού μας (σε ξεχωριστό φύλλο). Περιμένουμε λοιπόν τα γράμματά σας ή τα e maίl και τον αριθμό του τηλεφώνου σας με την ένδειξη στο φάκελο: «ΓΙΑ ΤΗ ΣΤΗΛΗ ΤΟΥ ΜΑΘΗΤΗ». -

Λ Υ ΣΕ Ι Σ ΑΣ ΚΉ ΣΕ ΩΝ

1.

(Επροτάθη από το μαθητή Σούμπλη Γεώρ­ γιο, Αθήνα).

Μία ομάδα αποτελείται από πέντε παιδιά, τα οποία είναι τουλάχιστον ενός έτους το καθένα (την ηλικία τους την υπολογίζουμε σε ακέραια έτη). Σε καθένα από τα παιδιά αυτά δίνουμε τόσα ευρώ όση είναι η ηλικία του. Έτσι δώσαμε 14 ευρώ. Να δείξετε ότι τουλάχιστον δύο από τα παιδιά αυτά έχουν την ίδια ηλικία.

w >z�3� w >3� w �4 t>w �4�t>4�t�5 Συνεπώς: χ + ψ + z + w + t 1 + 2 + 3 + 4 + 5 = 15 � x + ψ + z + w + t 15, άτοπο λόγω της (1). Άρα, τουλάχιστον δύο από τα παιδιά αυτά έχουν την ίδια ηλικία. 2.

(Επροτάθη από το μαθητή Σούμπλη Γεώρ­ γιο, Αθήνα).

Στο εσωτερικό ενός τετραγώνου ΑΒΓΔ παίρνουμε τρία σημεία Ε, Ζ και Η έτσι, ώστε το τρίγωνο ΕΑΒ να είναι ισοσκελές με κορυφή το Ε, το τρίγωνο ΖΑΔ να είναι ισο­ σκελές με κορυφή το Ζ και τα τρίγωνα ΑΕΖ και ΕΖΗ να είναι ισόπλευρα. Η ευθεία ΑΕ τέμνει την πλευρά ΒΓ στο σημείο Θ και η ευθείας ΑΖ τέμνει την πλευρά ΔΓ στο Ι. Να δείξετε ότι: 1) ΔΖ = ΓΗ. 2) Τα σημεία Ι, Η και Θ είναι συνευθειακά. 3) Τα σημεία Α, Η και Γ είναι συνευθειακά. ·

Λύ ση ( από τ ον ίδιο μ α θη τή) :

Έστω x,ψ,z,w και t είναι οι ηλικίες των παιδιών αυτών (σε ακέραια έτη). Έστω τώρα ότι οι ηλικίες των παιδιών αυτών είναι ανά δύο διαφορετικές και έστω ότι: χ < ψ < z < w < t . Έχουμε: x + ψ + z + w + t = 14 ( 1 ). Επειδή οι αριθμοί χ, ψ, z, w και t είναι θετικοί α­ κέραιοι, έχουμε: χ�1 ψ>χ�1�ψ>1�ψ�2 z>ψ�2�z>2�z�3

ΕΥΚΛΕΙΔΗΣ Β' λη ' τ.1Π9


Η στήλη του Μαθητή

ΛίJ ση (α π ό τ ο ν ίδιο ν μαθ ητή ) :

1) Τα ισοσκελή τρίγωνα ΕΑΒ και ΖΑΔ προφα­ νώς είναι ίσα (ΠΠΠ). Έτσι, έχουμε: ΕΒ Α = ΕΑΒ = zΑΔ = zΔΑ = 15 ° ' αφού ΕΑΖ = 60° . Φέρνουμε τις ΕΔ και ΕΓ. Επειδή ΕΜ = ΕΒΓ (= 75°), ΕΑ = ΕΒ και ΑΔ = ΒΓ, έπεται ότι τα τρίγωνα ΕΑΔ και ΕΒΓ είναι ίσα (ΠΓΠ), οπότε ΕΔ = ΕΓ. Εξάλλου, τα τρίγωνα ΖΑΔ και ΖΕΔ είναι ίσα, γιατί ΑΖΔ ΕΖΔ (= 150°), ΖΑ = ΖΕ και ΖΔ = ΖΔ. Λ

σο του τμήματος ΙΘ, οπότε ΑΗ _l ΙΘ . Εξάλλου, επειδή τα ορθογώνια τρίγωνα ΑΒΘ και ΑΔΙ είναι προφανώς ίσα, έχουμε ΒΘ = ΔΙ και άρα ΓΙ = ΓΘ. Έτσι, έχουμε και ΓΗ _l ΙΘ . Συνεπώς τα σημεία Α, Η και Γ είναι συνευθειακά (αλλιώς θα είχαμε στο Η δύο κάθετες στην ευθεία ΙΘ). ΑΣΚΗ Σ Ε Ι Σ Γ Ι Λ Λ Υ Σ Η 3.

Α θα-

Οι πραγματικοί αριθμοί α και β πληρούν την ισότητα: α2 + β2 1. Να δείξετε ότι:

=

=

Β

Α

(Προτείνεται από το μαθητή Β ' τάξης ν ά σιο Π α ν αγιιίηου, Αθήνα).

J α 2Ρ + αΡ 2 1 � �.

4.

(Προτείνεται από το μαθητή Β' τάξης νη Μι στ ρ ιώ τη , Αθήνα).

Α ντ ώ ­

Σ ' ένα ημικύκλιο παίρνουμε δύο διαδοχικά τόξα ΑΒ και ΒΓ με ΑΒ > ΒΓ Ονομάζουμε Μ το μέσο του τόξου ΑΓ και φέρ νουμε την κάθετο ΜΔ στη χορδή ΑΒ. Να δείξετε ότι: ΑΔ ΔΒ + ΒΓ. ...-...

..-...

...-...

..-...

-

=

Άρα: ΕΔ = ΑΔ = ΔΓ και επειδή ΕΔ = ΕΓ, έπεται ότι ΕΓ= ΕΔ = ΔΓ και συνεπώς το τρίγωνο ΕΔΓ είναι ισόπλευρο. Άρα ΔΕΓ = 60° και επειδή ΖΕΗ = 60° , έπεται ότι ΖΕΔ = ΗΕΓ . Λόγω αυτού και επειδή ΕΖ = ΕΗ και ΕΔ = ΕΓ, τα τρίγωνα ΕΖΔ και ΕΗΓ είναι ίσα και άρα ΔΖ = Γθ. 2) Φέρνουμε τις ΙΗ και ΗΘ, καθώς και την ΑΗ, η οποία τέμνει την ΕΖ στο Κ. Στο ορθογώνιο τρίγωνο ΑΒΘ, έχουμε 90° - 15 ° = 75° και επειδή ΕΒ Θ = 7 5 ° , έπεται ότι το τρίγωνο ΕΒΘ είναι ισο­ σκελές και άρα ΕΘ = ΕΒ = ΕΑ. Έτσι, το Ε είναι το μέσο του τμήματος ΑΘ. Όμοια, βρίσκουμε ότι το Ζ είναι μέσο του τμήματος ΑΙ. Εξάλλου το τετρά­ πλευρο ΑΕΗΖ προφανώς είναι ρόμβος και άρα το Κ είναι μέσο του τμήματος ΑΗ. Έτσι, έχουμε ΚΕ//ΗΘ και ΚΖ/ΙΙΗ και άρα τα ση­ μεία Ι, Η και Θ είναι συνευθειακά (γιατί, αλλιώς από το Η θα είχαμε δύο παράλληλες προς την ΖΕ). 3) Όπως βρίσκουμε εύκολα, τα τρίγωνα ΖΙΗ και ΕΗΘ είναι ισόπλευρα και ίσα. Άρα το Η είναι μέΛ

Λ

EeB

5.

(Προτείνεται από το μαθητή Β' τάξης τ ρ ω Γ ιαννόγλο υ, Μυτιλήνη).

Δ η μι] =

Έστω ένα τρίγωνο ΑΒΓ με εμβαδό (ΑΒΓ) 1. Να δείξετε ότι: (ρ + ρa )( 4R + ρ) + (ρs + ρ r ) 2 - ρ μ ρ γ + ρ 2 � 10

Λ

6.

=

(Οι ασκήσεις 6, 7 και 8 προτείνονται από το μαθητή Β ' τάξης Αθα νάσιο Ιω άνν ου, Ιστιαία).

Ν α λυθεί η aνίσωση: χ2 + 4χ + 4

1�

7.

-3ι � 9 .

Με lxl � 2 , να απλοποιηθεί η παράσταση:

Να γίνει με ρητό παρονομαστή η παράσταση:

ΕΥΚΛΕΙΔΗΣ Β' λη ' τ.l/80

όπου α * Ο ·


Εκδόσει�

���(�gς�

τη�

ΕΛΛΗΝΙΚΗΣ ΜΑΘΗΜΑΤΙΚΗΣ ΕΤΑΙΡΕΙΑΣ nι ι.ι π-ι\

, \ ΙI "1ι.. 11: \I \·� !1\1 \ l l i<./1>_ I I

... Ift I \�

LOUIS BRAND ΜΑΘΗΜΑΤΙΚΗ ΑΝΑΛΥΣΗ

IUAI"nnt ΣΊΙΣ � � ε!ΙΣΩDΙΣ

t

:(:

Σ'CΟΙΧΕΙQΔΗΣ ΓΡΑΜ..ΚΗ ΑΛΓΕΒΡΑ

Ιii!IIRQ JιΒqJιηιq ιιιιιειι

\1-\HJ Ι \f\ l l k l l

HIIJ \ I O I Ί'\ΦI \

Σ rnιχΕιΩ.\ΙΙl. Γι·ο�η:τι•ι \ \110 ΑΝητι:ι•ιι ΣκσιιιΑ

\::10\0111�1/ f ); I I \! H \ l ltri<H 1 1'1 0) "' Ι t .ll%1 \�\-Η ι \1 I 111\1<>,..<1 Ι I<J\ \1\1111"'''.."'

R J.. S Ι:. ,ι Η ι 11 tι • ,\·Ι ,, ι ιι ι 11 t τ ι r .� , ' " / ,\ f' Q N 11 Ι I / ( .\ ' 9 .J

................ ..... ....... ...

'-'"'"1�ιι οιι�"'""'τι•"Η"Ι'tΙ•

ΕυκλείδηςΑ': Τεύχος 2 ευρώ Συνδρομή (4 τεύχη) 10 ευρώ (4 τεύχη + 2 ευρώ ταχυδρομικά) θ Σχολεία: ευρώ δρχ. ΕυκλείδηςΒ': Τεύχος 2,5 ευρώ Συνδρομή (4 τεύχη) 12 ευρώ (4 τεύχrι + 2 ευρώ ταχυδρομικά) Σχολεία: 10 ευρώ Τεύχος 5 ευρώ ΕυκλείδηςΓ: Συνδρομή (2 τευχη ' ) 10 ευρω' Μαθημ. Επιθεώρηση: Τεύχος 5 ευρώ Συνδρομή (2 τεύχη) 10 ευρώ Αστρολάβος: Τεύχος 5 ευρώ Συνδροι.ο'ι (2 τεύχη) 10 ευρώ ΔελΊίο (Bullelίι): Τεύχος 7,30 ευρώ

Διεθνής Μαθηματικές Ολυμπιάδες 20 ευρώ 1959 1999: Βαλκανικές Μαθημαπκές Ολυμπιόδες 1984-2001: 15 ευρώ Θέματα εξετάσεων στα A.E.I 1976 19θ9: 6 ευρώ Πρακτικά: 1ου Πανελλr'fν{ου Σuνεδρίου & ευρώ 2α.ι θ Πανελλr'fν{ου Σuνεδρίου ευρώ θ , .... ., .... . . ...,.,. .., .. ευρω' 3ou π,..,η,.�"' • '�'• ..νν.ι,. •

• • θ 4αιδου ΓΊι:r.fλλη'νiοι ΣιΜ:δι]α.ι θ ευρώ 6ou Πανελληνίοιι Σuνεδρίου ευρώ θ 7α.ι Πανελληνίοu Σuνεδρίου ευρώ θ 8ou Πcινελλη\iου Σuνεδρίου ευρώ θ • ....... •

ευρώ 9ou Πανελλr'fν{ου Σuνεδρίου 10ou Πανελληνίοu Σuνεδρίου 17 ευρώ

1 1α.ι l'lcιν&λλη'νiου Σuνεδρίου 1 4α.ι Πcινελλη\iου Σuνεδρίου 15α.ι Πcινελλη\iου ΣιΝεδρίου 16ou Πανελλr'fν{ου ΣιΝεδρίου

17 ευρώ 17 ευρώ 17 ευρώ 17 ευρώ

�:���� :εuρω �

....... ,.""' ..u"'"t'"" 19ou Πcινελλη\iου ΣιΝεδρίου 25 ευρώ

20ou Πανελλr'fν{ου ΣιΝεδρίου 2S ευρώ Συνέδριο Hermis '92 (Αγγλικά) 2S ευρώ Συνέδριο Hermis '94 (Αγγλικά) 2 τόμοι 0 τόμος 15 ευρώ

Γραμμική Άλγεβρα

(Gr. Muncres) 6 ευρώ ΗΡΩΝΟΣ ΑΛΕΞΑΝΔΡΕΩΣ (Ονόματα Γεωμετρικών όρων ΓΕΩΜΕτΡΙΚΑ) 20 ευρώ

(Μετρικά · Διόmρα) 20 ευρώ ευρώΔιαλέξεις: Ο τόμος 5 ευρώ Μαθηματική Ανάλυση (Louis Brand) 25 ευ ρώ Διαφορικές Εξισώσεις (Steρhenson) 10 ευρώ Ιστορία Μαθηματικών Loria θ (4 tόμοι) Α, Β, ΓΑ• Γa ο tόμος ευρώ 70 Χρόνια Ε.Μ.Ε. 4 ευρώ Ελλ ηνικη' Μ α θ ηματικη' Βιβλιογραφία 4 ευρώ Στοιχειώδης Γεωμετρία από θ Ανώτερη Σκοπιά ευρώ Θεωρία Αριθμών 17 ευρώ 10 Χρόνια 'ΈΥΚΛΕΙΔΗΣ" ΣΕΙΡΑ Α' (1990-1999) CD 25 ευρώ

Τα nαλαιόnρα τ.ε ύχη όλων των εκδόσεων πωλούνται με τις τρiχοuοες τιμiς του 2004



Issuu converts static files into: digital portfolios, online yearbooks, online catalogs, digital photo albums and more. Sign up and create your flipbook.